Question june

Réussis tes devoirs et examens dès maintenant avec Quizwiz!

A study was conducted of factor dosing strategies for uncomplicated wisdom tooth extraction in adolescent boys with severe factor VIII deficiency. Patients were assigned to each of 2 groups. Preoperatively, patients in the control arm received intravenous factor VIII 50 units/kg, and patients in the experimental arm received 20 units/kg. The rest of the treatment was the same for both groups. The variable tested was intraoperative hemostasis defined as normal vs excessive bleeding. Of the 20 patients in the control group, 5 had excessive bleeding intraoperatively. Of the 20 patients in the experimental group, 10 had excessive bleeding intraoperatively. Of the following, the MOST accurate statement regarding absolute or relative risk for excessive intraoperative bleeding in this study is A.absolute risk of excessive intraoperative bleeding in the control group is 25% B.absolute risk of excessive intraoperative bleeding in the control group is 33% C.relative risk of excessive intraoperative bleeding in the control group is twice as high the experimental group D.relative risk of excessive intraoperative bleeding in the experimental group is 4 times as high as the control group

A. Absolute risk defines the risk or probability of an outcome that an individual would experience during or after a given treatment. Absolute risk is calculated as the ratio of the harmful event in question (outcome) divided by the number of people in the group. The absolute risk of excessive intraoperative bleeding in the control group is equal to 5 affected patients divided by the total number of patients in the control group, 20, for an absolute risk of 25%. The absolute risk of excessive intraoperative bleeding for the experimental group is equal to 10 affected patients divided by 20 patients (or 50%). Thus, Response Choice A is correct. Relative risk of excessive bleeding for the experimental group in this vignette is defined as the ratio of the absolute risk in the experimental group divided by the absolute risk in the control group. In this vignette, the relative risk of excessive intraoperative bleeding is twice as high in the experimental group as in the control group (0.5 ÷ 0.25 = 2). Relative risk compares the probability of one treatment outcome to an alternative treatment outcome. Relative risk is commonly used in population studies in which 2 different groups, exposed and not exposed, are studied for a predetermined harmful outcome. A relative risk of 1 implies no difference between the outcomes of 2 different treatment groups. A relative risk for an experimental group that is greater than 1 implies that the experimental group is more likely to have the studied harmful outcome compared to the control group. Conversely, a relative risk for an experimental group that is less than 1 implies that the experimental group is less likely to have the studied harmful outcome compared to the control group. Three statistical calculations that may be derived from relative and absolute risk include relative risk reduction (RRR), absolute risk reduction (ARR), and number needed to treat (to prevent a harmful event) (NNT). The RRR is the lowering of the risk of bad outcome for the patients receiving the treatment. The RRR is calculated as 1 minus the relative risk. In this vignette, the RRR for the experimental group is -1. A negative value suggests an increase in the relative risk of a harmful outcome in the experimental group compared to control. The ARR is defined as the difference between the absolute risk of an outcome of 2 different treatment groups. In this vignette, the experimental group's absolute risk (50%) is higher than the control group's absolute risk (25%). To calculate the ARR of excessive intraoperative bleeding for the experimental group, the absolute risk of the experimental group is subtracted from the control group (25% - 50% = -25%). A negative value for ARR implies an absolute risk increase, so in this vignette the experimental treatment is not beneficial as compared to the control. The ARR is the reciprocal or inverse of the NNT to prevent harm. However, if the ARR is a negative value, its inverse would be the number needed to treat to cause harm (NNH). The NNT(H) is the number of patients needed to be treated to experience the expected outcome, beneficial or harmful. In this vignette, the ARR is -25%, and its reciprocal or NNH is -4, implying that the NNH is 4 patients. Relative risk and odds ratios are similar and often confused. The calculation of relative risk implies the ratio of people with the studied outcome as a part of the whole group. Odds ratio is calculated as the number of affected individuals divided by the number of unaffected individuals in a group. PREP Pearls Absolute risk is the risk of an outcome in a given group of patients. Absolute risk reduction is the comparison of the risk in the treatment group compared to the experimental group. Relative risk is the ratio of the risk experienced by the experimental group compared to the control group.

A 13-year-old adolescent female underwent biopsy of a painless lump on her left leg, which was diagnostic for rhabdomyosarcoma. Metastatic evaluation revealed multiple pulmonary nodules, 2 left tibial lesions, and bone marrow involvement with rhabdomyosarcoma. Of the following, what pathologic subtype of rhabdomyosarcoma is MOST likely present in this patient? A.alveolar rhabdomyosarcoma, translocation negative B.alveolar rhabdomyosarcoma, translocation positive C.embryonal rhabdomyosarcoma, botryoid variant D.embryonal rhabdomyosarcoma, spindle cell variant

B. Rhabdomyosarcoma represents 3% of all malignant tumors in children, usually presenting as a painless mass. Additional signs and symptoms vary depending on the tumor location. Metastatic disease is present in 20% of patients at diagnosis, and the lung is the most common site. There are 2 histopathologic subtypes of rhabdomyosarcoma, alveolar rhabdomyosarcoma (ARMS) and embryonal rhabdomyosarcoma (ERMS). Alveolar rhabdomyosarcoma comprises 20% of cases of rhabdomyosarcoma, usually presenting in the extremities, trunk, and paratesticular region of older children and adolescents, as in this vignette. Tumors cells are usually described as "small round blue cells" that aggregate together separated by fibrovascular septa. Discohesive areas develop within the aggregates, giving rise to spaces lined by tumor cells. These open spaces resemble pulmonary alveoli, giving rise to the name alveolar rhabdomyosarcoma. Tumors cells in ARMS often have a characteristic translocation of FKHR at 13q14 with PAX 3 at 2q35. Less commonly, they exhibit a translocation of FKHR with PAX7 at 1p36. Translocation-positive ARMS is highly invasive, often presenting with metastatic disease, as in this vignette. Translocation-positive ARMS has a poorer prognosis compared to ERMS. Translocation-negative ARMS is clinically and molecularly identical to ERMS, described below. Given the prognostic significance of these translocations, molecular characteristics are becoming more of a major determinant in risk stratification for therapy options as compared to histology alone. Embryonal rhabdomyosarcoma accounts for the majority of cases of rhabdomyosarcoma (80%). It typically occurs in children younger than 10 years, more commonly in the regions of the head, neck, and genitourinary tract. Anatomical areas that confer a favorable prognosis include the orbit, superficial head and neck, biliary tree, paratesticular region, and vagina. Embryonal rhabdomyosarcoma is associated with a loss of heterozygosity at the 11p15.5 locus. There is expression of the paternal genetic information due to loss of the imprinted maternal gene. This results in the overexpression of insulin-related growth factor-2. Tumor cells may be hyperdiploid, which correlates with a better prognosis. The microscopic appearance of ERMS is similar to developing skeletal muscle with varying degrees of differentiation. There are 2 variants of ERMS, botryoid and spindle cell. The botryoid variant is moderately cellular with a loose myxoid stroma. This variant occurs as polypoid nodules in the vagina, urinary bladder, and bile ducts. The spindle-cell variant develops in the paratesticular region. The tumor cells are arranged in whorls and dense bundles of spindle-shaped cells, which resemble smooth muscle. Both of these ERMS variants have more favorable outcomes compared to non-botryoid or non-spindle cell ERMS. PREP Pearls Alveolar rhabdomyosarcoma usually occurs in the extremities, trunk, and paratesticular region of older children and adolescents and is associated with balanced chromosomal translocations t(2;13)(q35;q14) and t(1;13)(p36;q14). Translocation-negative alveolar rhabdomyosarcoma is clinically and molecularly identical to embryonal rhabdomyosarcoma. Embryonal rhabdomyosarcoma usually occurs in children younger than 10 years, arising in the head and neck region and genitourinary tract and is associated with a loss of heterozygosity at the 11p15.5 locus. The prognosis for embryonal rhabdomyosarcoma is more favorable compared to alveolar rhabdomyosarcoma.

A physician is developing an initiative to decrease the time to administration of factor replacement for hemophilia patients who are brought to the emergency department after head trauma. The average time at the physician's hospital is currently 2 hours. Areas for process improvement are identified on a driver diagram, and a protocol is created for triaging these patients. Of the following, the MOST appropriate SMART (specific, measurable, achievable, realistic, and time-limited) aim for this project is A.over the next 3 months, plan to administer factor replacement within 1 hour in the majority of patients with hemophilia presenting to the emergency department after head trauma B.over the next 6 months, plan to administer factor replacement within 1 hour in at least 80% of the patients with hemophilia who present to the emergency department after head trauma C.plan to administer factor replacement as soon as possible to patients with hemophilia presenting to the emergency department after head trauma D.within 1 month, plan to administer factor replacement within 15 minutes to 100% of patients with hemophilia who present to the emergency department after head trauma

Correct Answer: B View Peer Results Average Correct: 84.05% A quality improvement project should have a SMART aim (specific, measurable, achievable, realistic, and time-limited). Of the response choices listed, only Response Choice B meets all 5 criteria: specific (in at least 80% of patients with hemophilia who present to the emergency department after head trauma), measurable (patients to get factor within 1 hour of presentation), achievable, realistic, and time-limited (over the next 6 months). Response Choice D is not achievable or realistic due to the short 1-month time frame during which factor replacement is expected to be administered within 15 minutes of presentation to the emergency department in 100% of patients. Response Choice A is not specific, measurable, or realistic because there is a vague goal of a "majority" of patients. Response Choice C is not specific, measurable, or time-limited. The Plan-Do-Study-Act (PDSA) cycle is a trial-and-learning methodology used as a model for improvement. Often, multiple PDSA cycles are necessary to successfully effect change. This model is based on 3 questions: What are we trying to achieve? How will we know that the change led to improvement? What changes can we implement for improvement? Once the plan is created (Plan) and the change is implemented (Do), data are collected. Displaying the data on a run chart is a useful way of determining whether the change has led to an improvement (Study). Based on this data, the changes to be implemented in future PDSA cycles are planned (Act). The use of PDSA cycles allows for knowledge to be gained through an iterative process using tests of change to achieve the stated aim. To increase the rate of learning and improvement, it is recommended to test the change on a small scale and collect data sequentially over time. For example, if a quality improvement project is framed for 1 year, 3 months of baseline data are collected and analyzed. A plan is created, and 3 more months of data are collected during the implemented strategy period. This new data can be analyzed and may be the conclusion of the study, or another plan to improve patient care and safety can be created and then implemented over the next 3 months. There are various tools used in quality improvement work. Process mapping (eg, flow diagrams) creates a picture of a process or system. Cause and effect diagrams, also known as fishbone diagrams, can organize the group's ideas on the factors that may be contributing to the problem. Key driver diagrams, which may include primary and secondary drivers, organize information and assist in developing a theory of why the proposals will lead to improvement. Gathering and organizing information using such tools may lead to the development of an effective SMART aim. PREP Pearls A SMART aim for a quality improvement project should be specific, measurable, achievable, realistic, and time-limited. The Plan-Do-Study-Act cycle is a trial-and-learning methodology used as a model for improvement. The cycle is a data-driven, iterative process using small tests of change to achieve the stated aim. Once the plan is created (Plan) and the change is implemented (Do), data are collected. Displaying the data on a run chart is a useful way of determining whether the change has led to an improvement (Study). Based on this data, the changes to be implemented in future PDSA cycles are planned (Act).

A 20-month-old boy has a 2-week history of abdominal pain with distension and lethargy. He appears tired and has a firm, left-sided abdominal mass. Imaging reveals a 6-cm left adrenal mass. Histologic results of a biopsy are consistent with neuroblastoma. Bilateral bone marrow biopsies reveal no evidence of disease. A MIBG (metaiodobenzylguanidine) scan is significant for the left adrenal mass and metastatic lesions in the right fifth rib and left femur. Of the following, the MOST appropriate treatment course is A.chemotherapy with additional chemotherapy based on response B.chemotherapy, surgery, autologous stem cell transplant, radiation, and biologic response modifiers C.chemotherapy and radiation only as determined by MYCN and ALK status D.surgical resection only

Neuroblastoma originates from neural crest cells of the sympathetic nervous system and is the most common extracranial solid malignancy of childhood. Treatment strategies for neuroblastoma are determined by risk group based on the patient's age, extent of tumor metastases (stage), histologic features as determined by the International Pathology Classification (INPC), and genetic mutations, such as MYCN mutation, and DNA ploidy. While there are several staging schemes, the International Neuroblastoma Risk Group Staging System (INRGSS) (http://inrgdb.org/neuroblastoma-information/staging-system) is currently used as part of the Children's Oncology Group risk classification and is based on preoperative imaging. The INRGSS consists of: Stage L1, a localized tumor that does not involve nearby or distant organs or large vessels Stage L2, a locoregional tumor with one or more image-defined risk factors Stage M, distant metastatic disease Stage MS, INRG stage L1 or L2 with metastases confined to skin, liver, and/or bone marrow in children younger than 18 months By the INRGSS criteria, the patient in this vignette, who is older than 18 months and has bony metastases, has stage M disease. Another system, which was established in 1988, was the International Neuroblastoma Staging System (INSS) (https://www.cancer.net/cancer-types/neuroblastoma-childhood/stages-and-groups). It used surgical/pathological staging and had been incorporated into Children's Oncology Group risk classification. In brief, the INSS consisted of: Stage 1: The tumor can be removed completely. Stage 2A: The tumor is localized and cannot be completely removed during surgery. Stage 2B: The tumor is localized and may or may not be completely removed during surgery, but nearby lymph nodes contain cancer. Stage 3: The tumor cannot be removed and has spread to regional lymph nodes but not to other parts of the body. Stage 4: The original tumor has spread to distant lymph nodes, bones, bone marrow, liver, skin, and/or other organs, except for those listed in stage 4S. Stage 4S: The original tumor is localized (as in stage 1, 2A, or 2B) and has spread only to the skin, liver, and/or bone marrow, in infants younger than 1 year. The spread to the bone marrow is minimal, usually less than 10% of cells examined show cancer. By the INSS criteria, the patient's disease is stage 4 or high risk. High-risk neuroblastoma has a poor prognosis, and treatment requires multi-agent chemotherapy, surgical tumor resection, autologous hematopoietic stem cell transplantation, radiation therapy, and biologic response modifiers, which is the most appropriate treatment approach for the patient in this vignette. Biologic response modifiers include anti-GD2 immunotherapy and isotretinoin. Radiation is given to the primary tumor site regardless of the extent of surgical resection, as well as to metastatic sites that do not respond to chemotherapy. Low- and intermediate-risk group designations are based on the other criteria. The INPC (Table) categorizes patients into favorable and unfavorable categories based on histologic features of the tumor taken in context with the patient's age. Less neuronal differentiation is associated with a poor prognosis, as is a high mitosis-karyorrhexis index, which is indicative of unfavorable histology. In certain cases, risk stratification can be further determined by the presence of MYCN amplification and ALK status. In this vignette, given this patient's age (> 18 months) and metastatic disease, he would be classified in the high-risk group regardless of MYCN or ALK status or histologic findings. Low-risk neuroblastoma is commonly treated with surgical resection when feasible, although complete resection is not mandatory. Surgical resection only is an appropriate treatment course for some patients with low-risk neuroblastoma, unlike in this vignette. Some low-risk subgroups can be observed using routine imaging without the need for surgical resection or chemotherapy, such as non-obstructing L1 adrenal tumors. Chemotherapy and radiation are used in instances of life- or organ-threatening tumors, or in cases of tumor recurrence noted during observation of these "low-risk" tumors. Intermediate-risk neuroblastoma is treated with chemotherapy and surgical resection, although complete resection is not always necessary. Subsequent chemotherapy courses are tailored based on initial tumor stage, histology, and response to initial chemotherapy. This treatment approach is suitable for intermediate-risk neuroblastoma, unlike the patient in this vignette. PREP Pearls Treatment strategies for neuroblastoma are determined by risk group based on the patient's age, extent of tumor metastases (stage), histologic features as determined by the International Pathology Classification (INPC), and genetic mutations, such as MYCN mutation, and DNA ploidy. The currently used International Neuroblastoma Risk Group Staging System (INRGSS) is based on preoperative imaging, whereas the International Neuroblastoma Staging System (INSS) had used surgical-based staging. Some cases of low-risk neuroblastoma may be observed with or without surgical resection. Certain subsets of low-risk and intermediate-risk neuroblastoma have been successfully treated with a chemotherapy response-based protocol where the need for additional chemotherapy is determined by imaging after initial chemotherapy courses. High-risk neuroblastoma requires treatment with aggressive chemotherapy, surgical resection, autologous hematopoietic stem cell transplantation, radiation, immunotherapy, and isotretinoin. ABP Content Specifications(s)/Content Area Know the principles of treatment for various stages of neuroblastoma Know the role of surgery in the treatment of neuroblastoma Know the role of irradiation in the treatment of neuroblastoma Know the role of chemotherapy in the treatment of neuroblastoma Know the role of biologic response modifiers in the treatment of neuroblastoma Be able to appropriately monitor the response to treatment of neuroblastoma

A 13-year-old adolescent girl with A-negative blood type receives a transfusion of packed red blood cells for anemia caused by heavy menstrual bleeding at menarche. The transfusion is stopped when the nurse reports that the blood product being transfused is labeled O-positive. The patient is asymptomatic and hemodynamically stable. A direct antiglobulin test, screen for antibodies, and plasma free hemoglobin are ordered. Urinalysis findings are normal. The MOST appropriate next step is to administer A.intravenous immune globulin B.intravenous methylprednisolone C.Rh immune globulin at time of first pregnancy D.Rh immune globulin now

The most severe immediate sequelae of mismatched transfusion is an acute hemolytic reaction. This occurs most commonly with ABO-unmatched transfusion but can occur with Rh(D)-antigen mismatch if the patient has been previously exposed to Rh(D)-positive blood products. A delayed hemolytic transfusion reaction may also occur. This reaction is caused by an anamnestic response with a rise in antibody titer 1 to 2 weeks after transfusion. Since this is the first transfusion for this patient, and she is asymptomatic at this time, neither of these hemolytic reactions are likely. Additionally, as a patient with type A blood, type O blood may be transfused without concern for sequelae. The Rh blood system includes a number of antigens, including D, C, c, E, and e. The terms Rh-positive or Rh-negative refer to the presence or absence of the D antigen. Unlike the ABO blood groups, Rh-negative individuals do not have anti-D antibody unless exposed to D antigen through transfusion or pregnancy. Although it was once reported that 80% of patients without the D antigen can develop anti-D antibody after exposure to a unit of D-positive blood, subsequent data have shown that antibody development only occurs in 22% of patients after exposure to D-positive packed red blood cells. The long-term risk for this patient is the development of anti-D antibodies that could result in hemolytic disease of the newborn in a future D-positive pregnancy. After exposure to a small volume of Rh-positive red blood cells, intramuscular Rh immune globulin (RhIg) is recommended as soon as possible, or as soon as the need is recognized up to 28 days after the exposure. Intravenous RhIg is used for exposure to large volumes. In male patients, the decision to give RhIg is based on whether they are likely to require transfusions in the future. The risk appears to be lower with platelet or plasma transfusion. Therefore, RhIg may not be necessary with D-mismatched platelet transfusion. The half-life of RhIg is 21 days, and the dose is calculated based on the volume of packed red blood cells transfused. Administration of RhIg at the time of the first pregnancy would be too late to prevent antibody formation from this blood exposure, although RhIg may also be indicated at the time of pregnancy for this patient. Intravenous immune globulin and intravenous methylprednisolone are not indicated because there is no evidence of hemolysis or hemodynamic instability. PREP Pearls Antibody development occurs in approximately 22% of persons without the D(Rh) antigen after exposure to D(Rh)-positive packed red blood cells. After exposure of a D(Rh)-negative recipient to a small volume of Rh-positive packed red blood cells, intramuscular Rh immune globulin (RhIg) is recommended. Intravenous RhIg is used for exposure to large volumes. The risk of D(Rh) alloimmunization appears to be lower with platelet or plasma transfusion when compared to packed red blood cell transfusion. Therefore, Rh immune globulin may not be necessary with D(Rh)-mismatched platelet transfusion. After transfusion of D(Rh)-positive red blood cells into a D(Rh)-negative female recipient, Rh immune globulin should be given within 72 hours of exposure, or as soon as the need is recognized up to 28 days after the exposure.

To reduce wait times for patients to receive their chemotherapy during oncology clinic visits, a new procedure is implemented for ordering laboratory tests and writing chemotherapy orders prior to the visit. Six months later, the clinic leadership requests an update. Of the following, the MOST appropriate method of determining whether this change has resulted in an improvement in time to chemotherapy administration is to display the data on a A.driver diagram B.Pareto chart C.run chart D.Shewhart chart

A run chart (Figure ), also known as a trend chart or time series chart, is a simple graphical display of data used to determine whether a change has resulted in an improvement. All of the other response choices are used in the planning of an improvement project. A driver diagram is a tool for organizing and displaying the theory for improvement. A Pareto chart identifies the areas of greatest impact for improvement. A Shewhart chart is used to distinguish between common- and special-cause variations in the system. A run chart is a visual tool to display progress in project measures over time and determine whether the change that was implemented resulted in an improvement and if the improvement was sustained over time. A shift is when there are 6 or more consecutive points either above or below the median. A trend is when 5 or more consecutive points are increasing or decreasing. A run is a series of consecutive points on either side of the median. The number of runs is the number of times that the line connecting the data points crosses the median plus one. An astronomical point is an outlier that is unusually small or large (not just the highest or lowest data point of a data set). A control chart, or process control chart, is a specific type of run chart that shows the typical process variation, or common-cause variation, over time. It typically has a center line, which represents the process mean (or average or median), with 3 process standard deviations above as the upper control and 3 standard deviations below as the lower control. Whereas common-cause variations are inherent in the system and affect all people and outcomes in the system, special-cause variation occurs due to specific circumstances or an unexpected occurrence that does not affect everyone or is not part of the system all the time. There are different measures in an improvement project. An outcome measure looks at the result and reflects the health state of the patient (eg, mean wait time to receive chemotherapy). A process measure assesses the activities carried out by the provider to deliver health services. (eg, percentage of staff who adhered to the new plan). A balancing measure assesses whether the changes intended to improve one aspect of the system have caused new problems in other areas (eg, patients are requiring additional blood draw for laboratory tests not included in the pre-written orders). PREP Pearls A run chart is a visual tool to display progress in project measures over time and determine whether the change that was implemented resulted in an improvement and if the improvement was sustained over time. A control chart, or process control chart, is a specific type of run chart that shows the typical process variation, or common cause variation, over time.

A 5-year-old girl with homozygous (SS) sickle cell disease is seen for her annual transcranial Doppler sonography screening. She is well and without fever or pain. Nonimaging based, continuous wave Doppler, time-averaged mean maximum velocities (in centimeters per second, cm/s) of the circle of Willis are shown: Right Left Middle cerebral artery 212 cm/s 192 cm/s Internal carotid artery 150 cm/s 160 cm/s Anterior cerebral artery 115 cm/s 105 cm/s Posterior cerebral artery 90 cm/s 92 cm/s Of the following, the MOST appropriate next step in her care is to A.initiate chronic transfusion therapy B.initiate hydroxyurea therapy C.repeat transcranial Doppler study in 1 week D.repeat transcranial Doppler study in 12 months

A time-averaged mean maximum velocity (TAMM) of 200 centimeters/second (cm/s) or greater in the middle cerebral artery or internal carotid artery is considered an abnormal nonimaging transcranial Doppler (TCD) sonographic study in children with sickle cell disease (SCD). However, TCD velocities can be influenced by multiple factors such as the expertise of the person performing TCD, concurrent illness causing transient velocity elevation, degree of anemia, and/or recent transfusion causing transient velocity decrease. Furthermore, there are adverse events associated with treating with monthly red blood cell (RBC) transfusions including iron overload and RBC alloimmunization with these risks outweighing benefits when the transfusions are not necessary. Thus, a mildly elevated velocity just above 200 cm/s in the middle cerebral artery on a single TCD, as in this vignette, needs to be confirmed with a repeat TCD in 1 to 2 weeks. When children have a definitive nonimaging TCD velocity of 200 cm/s or greater on initial or repeat studies, they have a stroke risk of up to 10% per year for each of the next 4 years for a total risk of 40% compared to the general population of those with sickle cell risk of stroke of 10% throughout childhood. In sickle cell disease, the standard of care is to initiate chronic monthly RBC transfusions to reduce stroke risk in definitive cases. Repeat TCD at the usual screening interval of 1 year is not an appropriate response to an abnormal TCD result because of the imminent risk of stroke. Although starting treatment with oral hydroxyurea may reduce TCD velocity, it is not recommended as the first-line therapy for primary stroke prevention for children with abnormal TCD (TAMM ≥ 200 cm/s). Prior to the technological development of TCD screening to assess an individual's risk of stroke, approximately 11% of children with phenotypically severe SCD (often referred to as sickle cell anemia), which includes homozygous SS (hemoglobin (Hb), HbSS) or Hb S beta zero thalassemia (HbSβ⁰), experienced an overt stroke by 20 years of age and 25% of patients experienced an overt stroke by 45 years of age. In pediatric age range, ischemic strokes predominate, while hemorrhagic strokes are more frequently seen in patients in their 20s. There are 2 age peaks for stroke prevalence, ages 2 to 5 years and ages 40 to 49 years. Another risk factor for stroke is an absolute reticulocyte count greater than 194 × 103/µL (194 × 109/L) before 6 months of age. Infants with high reticulocytosis and thus a higher hemolytic rate are more likely to have abnormal TCD velocities than infants with lower absolute reticulocyte counts. Primary stroke prevention begins with assessment by TCD before starting interventions in patients deemed high risk prior to actual stroke event. This noninvasive test measures the mean blood flow velocity in the arteries in the circle of Willis and internal carotid artery typically through the temporal bone windows. Focal increased flow suggests arterial stenosis, indicating increased risk of stroke. Children with sickle cell anemia, HbSS or HbSβ⁰, should begin screening with TCD starting at age 2 years and continue at least annually until 16 years of age. In the 'Stroke Prevention Trial in Sickle Cell Anemia' (STOP)-1 trial, chronic RBC transfusion therapy significantly reduced the risk of stroke (to 2%) compared to observation (with 16%) in children with velocities of 200 cm/s or greater. Additionally, the state of California demonstrated that the number of admissions for first stroke in children with SCD dropped with the initiation of transfusion therapy in this population (0.88 events/100 person-years in the years 1991-1998 vs 0.17 events/100 person-years in the year 2000). Up to two-thirds of patients with SCD who experience stroke will have a recurrence. Secondary stroke prevention with chronic transfusion therapy reduces this risk. Chronic transfusion therapy is superior to both observation and hydroxyurea for stroke risk reduction. Additionally, stroke is an indication for subsequent hematopoietic stem cell transplantation. Chronic transfusion therapy aims to lower and maintain the pretransfusion Hb S level at 30% or less and hemoglobin level to approximately 9 g/dL (90 g/L). To achieve a post-transfusion hemoglobin level of not greater than 12.5 g/dL (125 g/L), transfusions are typically given every 3 to 6 weeks. A subset of patients, after a year or more of chronic transfusion therapy for primary stroke prevention, may be eligible to transition to hydroxyurea therapy meeting criteria similar to subjects enrolled in the 'TCD With Transfusions Changing to Hydroxyurea' (TWiTCH) trial. This subset must meet all of the following 3 criteria: Transcranial Doppler velocity normalizes while on transfusion therapy. Brain magnetic resonance angiography shows no significant vasculopathy. There have been no recurrent neurologic events. It is unclear whether patients receiving secondary stroke prevention can ever stop chronic transfusion therapy. In patients who do not have access to blood transfusions because of limited resources or who cannot tolerate blood transfusions (eg, RBC alloantibodies, transfusion reactions), hydroxyurea may be superior to observation. There is minimal evidence for prevention of hemorrhagic strokes in SCD, but chronic transfusion therapy or hydroxyurea are reasonable. PREP Pearls Historically, prior to transcranial Doppler screening, approximately 11% of children with severe sickle cell disease had an overt stroke by 20 years of age. In children with severe sickle cell disease, a time-average maximum mean velocity of 200 centimeters/second (cm/s) or greater in the middle cerebral artery or internal carotid artery is abnormal and increases the risk of stroke up to 40% within the next 4 years. Chronic transfusion therapy is the first-line therapy for primary stroke prevention in children with sickle cell disease who have abnormal transcranial Doppler results.

A 3-year-old boy currently on maintenance therapy for high-risk neuroblastoma develops dry, chapped lips and erythematous, excoriated skin near the corners of his mouth. Of the following, the medication MOST likely to be the cause of his symptoms is A.dinutuximab B.interleukin 2 C.isotretinoin D.sulfamethoxazole-trimethoprim

C. Dinutuximab, interleukin 2, isotretinoin, and sulfamethoxazole-trimethoprim are commonly used agents in maintenance therapy for neuroblastoma. Cheilitis and dry skin are common adverse effects of isotretinoin (13-cis retinoic acid). The most common adverse effect of dinutuximab, the anti-GD2 chimeric monoclonal antibody 14.18, is pain during administration. Other common adverse effects of dinutuximab include fever, hypotension, hyponatremia, edema, and capillary leak syndrome. Adverse effects of interleukin 2, an immune-stimulating agent used to improve the effectiveness of dinutuximab, are similar to those of dinutuximab. Sulfamethoxazole-trimethoprim, given for Pneumocystis jirovecii pneumonia prophylaxis, is typically well tolerated but may cause cytopenias and photosensitivity. Isotretinoin is a synthetic retinoid most commonly used in the treatment of nodular acne. In neuroblastoma, isotretinoin decreases proliferation, decreases MYCN oncogene expression, and induces the differentiation of neuroblastoma cells. In the CCG-3891 study, high-risk neuroblastoma patients randomized to receive isotretinoin after myeloablative chemotherapy and autologous hematopoietic stem cell rescue had a 5-year event-free survival of 50% (± 8%) and 5-year overall survival of 59% (± 8%). These results suggested that treatment with myeloablative chemotherapy and autologous stem cell rescue followed by isotretinoin was superior to other combinations, although the study was not designed to determine a superior treatment. More recently, isotretinoin therapy has been used in conjunction with dinutuximab, which has improved short-term survival in high-risk patients. Cheilitis is a dose-limiting adverse effect of isotretinoin. For this reason, isotretinoin is typically administered in 14-day blocks with a 14-day rest between cycles. Cheilitis is treated with liberal use of topical emollients. Other adverse effects are photosensitivity, hypertriglyceridemia, headaches, dry eyes, and pain. Isotretinoin is a teratogen and is contraindicated in pregnancy (class X). The risk of birth defects is high even with short exposure. All patients, prescribers, wholesalers, and dispensing pharmacists must register in the iPLEDGE risk evaluation and mitigation strategy program (www.ipledgeprogram.com). Isotretinoin only comes in capsules, which can cause problems with dose administration for younger children typically affected by neuroblastoma. PREP Pearls Cheilitis is a common adverse effect of isotretinoin therapy. Isotretinoin is used in maintenance therapy for high-risk neuroblastoma. All patients receiving isotretinoin regardless of sex, as well as prescribers, wholesalers, and dispensing pharmacists must use the iPLEDGE program because of teratogenic risk.

A 5-year-old girl with B-precursor acute lymphoid leukemia has relapsed during maintenance chemotherapy. During her re-induction chemotherapy, it is determined that her only available hematopoietic stem cell product is from a 9/10 human leukocyte antigen (HLA)-matched unrelated donor with an HLA mismatch at DR. Her conditioning chemotherapy is planned, and the best method for cell processing is considered. Of the following, the method of T-cell depletion that will lead to the BEST prevention of graft-vs-host disease with the LEAST risk of viral reactivation is A.cyclophosphamide after stem cell infusion B.ex vivo T-cell depletion with column elution C.in vivo T-cell depletion with alemtuzumab D.irradiation of the stem cell product

CORRECT View Peer Results Average Correct: 35.33% The risk of graft-vs-host disease (GVHD) after hematopoietic stem cell transplant (HSCT) increases with an increasing degree of HLA mismatch between donor and recipient. The incidence of GVHD from a 10/10 HLA-matched donor is approximately 40% to 50%, and the incidence approaches 70% with a 9/10 HLA-matched donor. To decrease the risk of GVHD, several tactics have been used to remove or inactivate T cells before and after stem cell infusion. T-cell depletion decreases the risk of GVHD but increases the risk of viral reactivation of cytomegalovirus, Epstein-Barr virus, and human herpesvirus 6 and increases the risk of Epstein-Barr virus-associated post-transplant lymphoproliferative disease. Ex vivo T-cell depletion is a process where T cells are extracted from a bone marrow product or a peripheral blood stem cell product prior to stem cell infusion into the recipient. Selection techniques can remove all of the T cells, a subset of T cells (α /β or γ/δ), or only the CD34+ stem cells. CD34+ selection uses immunomagnetic beads to isolate the CD34+ cells, followed by column elution to collect the CD34+ selected population. Advances in ex vivo T-cell depletion have led to an improved reduction of T cells in the donated stem cell product. Older ex vivo methods resulted in a 1 log reduction of T cells, and new methods have demonstrated a 5 log reduction. A study from Memorial Sloan Kettering revealed lower rates of acute GVHD with newer CD34+ selection methods compared to older methods (5% vs 18%, P = 0.005), as well as equivalent rates of cancer relapse (18% vs 25%, P = 0.3). This newer method depletes a subpopulation of T cells, known as aß T cells, which cause acute GVHD. This method spares the γδ T cells, which help fight viral infection and promote stem cell engraftment. The patient in this vignette is receiving an HLA-mismatched hematopoietic stem cell transplant. To mitigate the up to 70% incidence of GVHD, T-cell depletion is required. Of the options presented, only ex vivo T-cell deletion with column elution of the stem cell product prior to infusion with the selective removal of αβ T cells alone, will reduce GVHD risk while maintaining the ability to prevent viral reactivation and allow engraftment. Nonselective T-cell depletion by in vivo medications such as alemtuzumab, anti-thymocyte globulin, or cyclophosphamide can lead to higher rates of cancer relapse and/or viral reactivation. In vivo T-cell depletion includes administering agents such as intravenous anti-thymocyte globulin to the HSCT recipient prior to the stem-cell infusion. Anti-thymocyte globulin results in T-cell depletion of the recipient and the donor stem cells. Alemtuzumab is another agent that leads to depletion of T cells and other CD52+ cells (lymphocytes, neutrophils, monocytes, macrophages, and natural killer cells) in the recipient and in the infused donor stem cells. A newer T-cell depletion method used primarily in the haploidentical HSCT setting is the administration of 50 mg/kg of cyclophosphamide on day +3 or +4 after the stem cell infusion to target alloreactive T cells. Tacrolimus and mycophenolate mofetil are also given in this setting as GVHD prophylaxis. A study from the Bone Marrow Transplant Clinical Trials Network (BMTCTN1203) revealed that post-transplant cyclophosphamide, tacrolimus, and mycophenolate had increased efficacy over GVHD control (hazard ratio, 0.74) when compared to standard GVHD prophylaxis of tacrolimus and methotrexate. Irradiation of red blood cells and platelets during HSCT can decrease the risk of transfusion-associated GVHD. Irradiation does not affect the risk of GVHD from a stem-cell source but destroys the stem cells. Therefore, stem cell products are never irradiated prior to infusion. PREP Pearls Ex vivo αβ T-cell depletion of the hematopoietic stem cell product prior to infusion removes the T-cell population that is most likely to cause graft-vs-host disease and leaves the population of γδ T cells, which help with engraftment and viral protection. Nonselective in vivo T-cell depletion with intravenous anti-thymocyte globulin or alemtuzumab may lead to lower rates of graft-vs-host disease but increased rates of viral reactivation.

A 14-year-old adolescent girl with a history of relapsed, high-risk, pre-B-cell acute lymphocytic lymphoma was found to have relapsed again 1 year after an human leukocyte antigen (HLA)-matched related hematopoietic stem cell transplant. Her previous sibling HSCT donor is available to donate stem cells for a second transplant. The patient was given anti-CD19 chimeric antigen receptor T-cell therapy in an attempt to achieve remission. Three months later, she is fatigued with a persistent fever, and has a normal blood pressure. She is without adenopathy or hepatosplenomegaly. She has confirmed adenovirus by serum PCR. Laboratory data are shown: Laboratory Test Result White blood cell count 4,500/µL (4.5 × 109/L) Neutrophils 75% Bands 5% Lymphocytes 15% Monocytes 5% Hemoglobin 12 g/dL (120 g/L) Platelet count 270 × 103/µL (270 × 109/L) IgG 360 mg/dL (3.6 g/L) Of the following, the MOST likely cause of her persistent viremia is A.B-cell aplasia B.cytokine release syndrome C.leukemia relapse D.macrophage activation syndrome

Chimeric antigen receptor T-cell (CAR-T) therapy involves the use of genetically modified T cells with surface receptors that operate independently of HLA-based receptor activation. Anti-CD19 CAR-T therapy is used to target CD19+ leukemia cells in patients with chemotherapy-resistant pre-B-cell leukemia and lymphoma in an attempt to achieve a sustained remission. The duration of cancer remission after CAR-T has been variable and may not obviate the need for subsequent hematopoietic stem cell transplant. The patient in this vignette has no evidence of leukemic relapse. Her persistent viremia and hypogammaglobulinemia is likely due to B-cell aplasia from her recent CAR-T therapy which has targeted host B cells. Intravenous or subcutaneous immunoglobulin infusions may help alleviate viral infection risk. Factors influencing B cell aplasia which can persist for weeks to months after CAR-T therapy, are being studied. Other common toxicities of CAR-T therapy independent of the CAR design or genetic modification include cytokine release syndrome and macrophage activation syndrome. Cytokine release syndrome (CRS) may arise 1-21 days after CAR-T infusion. CRS encompasses a spectrum of symptoms associated with elevation in interleukin (IL) 6 and interferon γ. CRS may range from mild flu-like symptoms to fever, hypotension, hypoxia, neurologic changes, and multiorgan failure. CRS is not common in people who do not respond to CAR-T therapy. Severe CRS symptoms are likely reversible with tocilizumab, an anti-IL-6 receptor antagonist. Glucocorticoids are also used when there is not a prompt response to tocilizumab. Close monitoring and supportive care is essential. Macrophage activation syndrome has been reported in patients receiving CAR-T therapy. This syndrome manifests with pancytopenia, liver dysfunction, hepatosplenomegaly, hyperferritinemia, hypertriglyceridemia, and coagulopathy. Elevated levels of IL-6, IL-10, and interferon γ are common with overlapping features similar to CRS. Additional CAR-T therapy toxicities include tumor lysis syndrome or neurologic changes such as encephalopathy or seizures. PREP Pearls B-cell aplasia, cytokine release syndrome, and macrophage activation syndrome are common toxicities after chimeric antigen receptor T-cell therapy. Cytokine release syndrome arising days to weeks after chimeric antigen receptor T- cell therapy may be treated with supportive care, tocilizumab and/or glucocorticoids depending on the severity of symptoms. B-cell aplasia and hypogammaglobulinemia may last for weeks to months following anti-CD19 chimeric antigen receptor T-cell therapy.

A 6-year-old boy with sickle cell anemia (hemoglobin SS) is receiving chronic transfusion therapy as stroke risk prevention. Earlier today, he received his scheduled packed red blood cell transfusion. Vital signs during the transfusion were stable, including a temperature of 37.1°C. An hour after the transfusion is completed, he develops chills and rigors with a temperature of 38.6°C and a heart rate of 105 beats/min. Respiratory rate and blood pressure remain stable. He does not have any rashes. Results of a urinalysis are normal. Of the following, the MOST likely diagnosis is A.acute hemolytic transfusion reaction B.delayed hemolytic transfusion reaction C.febrile nonhemolytic transfusion reaction D.massive transfusion reaction

Immune-mediated and non-immune-mediated transfusion reactions are complications associated with the administration of blood products. Febrile nonhemolytic transfusion reactions (FNHTRs) are the most common of all non-immune transfusion reactions with an estimated prevalence of 0.1% to 1% of all transfusions. An FNHTR is defined as an otherwise unexplained temperature elevation of more than 1°C above baseline in a patient either actively receiving a blood transfusion or who has received a blood transfusion within the last 3 hours. Symptoms of FNHTRs can include rigors and chills. Normal signs include blood pressure, respiratory status, and urine analysis, as in this vignette. Although acute hemolytic transfusion reactions might also present with fever and occur within hours of a blood transfusion, they are frequently associated with concomitant hemoglobinuria, jaundice, hypotension, and dyspnea. Other signs include disseminated intravascular coagulopathy and acute renal failure. Delayed hemolytic transfusion reactions occur 24 hours to 28 days after a blood transfusion. Patients can present with dark (tea-colored) urine, jaundice, fever, dyspnea, hypertension, and chest, abdominal, or back pain. The hemoglobin level is lower than expected because of the hemolysis of transfused cells. Massive transfusion reactions occur in association with transfusion of large volumes of blood products, defined as 50% of the patient's total blood volume transfused over 3 hours. The clinical presentation is characterized by volume overload as well as signs and symptoms related to hypocalcemia (tingling of the extremities, paraesthesias, and electrocardiographic changes, such as prolonged QT interval). Blood products containing more than 5 × 106 leukocytes per transfusion or products stored for more than 48 hours increase the risk for FNHTR development. These reactions are caused by the interaction between antibodies in the recipient's plasma and the major histocompatibility complex proteins of the human leukocyte antigen system expressed on the cell surface of lymphocytes, granulocytes, or platelets in the transfused products. This immune reaction causes the release of proinflammatory cytokines including interleukin (IL)-1, IL-6, IL-8, and tumor necrosis factor-α. These cytokines are responsible for the characteristic clinical symptoms of FNHTRs. Leukoreduction and volume reduction reduce the risk of allergic or febrile transfusion reactions. Prestorage leukoreduction of donated red blood cells is widely used to prevent these complications. As opposed to FNHTRs, allergic or anaphylactic transfusion reactions are caused by the exposure to soluble substances in the donor plasma that bind to preformed immunoglobulin (Ig) E antibodies on mast cells and basophils in the recipient, resulting in the release of histamine. Allergic transfusion reactions occur during or within 4 hours after a blood product transfusion. The clinical presentation can range from the mild sudden appearance of rash, pruritus, and hives to severe, life-threatening bronchospasm, respiratory distress, and hypotension. One of the most severe allergic transfusion reactions occurs in patients with IgA deficiency who have developed anti-IgA antibodies. Despite IgA deficiency occurring in 1 of 300 to 500 persons, few of these patients develop anti-IgA antibodies, which are usually IgG in type. The presence of anti-IgA antibodies in IgA deficient individuals increases the risk of developing anaphylactic reactions when exposed to blood products containing IgA. Overall, this complication is reported in 1 of 1,200 to 1,600 IgA-deficient patients. Specific precautions are recommended when transfusing products to patients with IgA deficiency and concomitant IgA antibodies. In these patients, frequent screening for IgA antibodies is warranted. Other strategies include the use of washed blood products, especially when transfusing packed red blood cells, to remove as much of the IgA as possible. For IgA-deficient patients requiring immunoglobulin therapy, products with documented low IgA content are needed, and subcutaneous, instead of intravenous, administration is recommended. PREP Pearls A febrile nonhemolytic transfusion reaction is defined as an unexplained temperature elevation of more than 1°C above baseline within 3 hours of a blood transfusion along with normal blood pressure, respiratory status, and urine analysis. Febrile nonhemolytic transfusion reactions result from the interaction between antibodies in the recipient's plasma and human leukocyte antigen (HLA) antigens on the leukocytes and/or platelets in the transfused blood products that causes the release of proinflammatory cytokines. The presence of anti-immunoglobulin (Ig)A antibodies in IgA deficient individuals increases the risk of developing anaphylactic reactions in affected individuals who are administered blood products containing IgA.

An 8-year-old girl has an ependymoma located in her spinal cord at vertebrae C4 to T2. As part of her treatment, she will receive either conformal type of photon therapy called photon intensity-modulated radiation therapy (IMRT) or proton beam radiation therapy. Of the following, the statement that BEST reflects the current knowledge of the cancer control benefit and relative risk of thyroid late effects from each type of radiation therapy is: A.Photon IMRT provides better cancer control and a lower risk for hypothyroidism compared to proton beam radiation therapy. B.Photon IMRT provides worse cancer control and an equivalent risk of thyroid late effects compared to proton beam radiation therapy. C.Proton beam radiation therapy provides equivalent cancer control and a lower risk of thyroid late effects compared to photon IMRT. D.Proton beam radiation therapy provides worse cancer control and a higher risk of thyroid cancer compared to photon IMRT.

In the conformal type of photon therapy or photon intensity-modulated radiation therapy (IMRT), the photons dislodge electrons ahead of the photon beam so that efficacy is lost in an exponential fashion over the distance to the tumor. To provide a tumoricidal dose to a non-superficial tumor, photon IMRT is given in large doses, risking injury to the healthy surrounding tissues from the scatter of electrons or exit dose. Radiation therapists recognize the need to use more conservative dose limits with photon IMRT for the treatment of tumors in and near the central nervous system (CNS) due to neurologic sequelae from radiation necrosis and/or increased risk of second malignancy. Unlike photon IMRT, the energy from proton beam radiation therapy is released within a relatively short distance from where the charged particles come to a stop. This forms an energy absorption dose curve first discovered in 1903 by William Bragg. By modifying the proton beam energy, the 'Bragg peak' may be adjusted to treat the entire volume of the tumor with less ionizing radiation scatter or exit dose to healthy surrounding tissue. Proton beam radiation therapy is increasingly used in childhood cancer treatment, particularly for tumors of the CNS, to avoid surrounding healthy tissue damage. In this vignette, the patient has a large tumor along the spine at about the level of the thyroid gland. The goal is to deliver a tumoricidal dose of radiation to the spinal tumor while minimizing damage to surrounding normal thyroid tissue. A proton beam may be able to irradiate the tumor with less scatter to the thyroid and with less total radiation than may be necessary for treatment with photon IMRT. Photon IMRT in the vicinity of the neck imparts a higher risk of thyroid damage compared to proton beam. Thus, proton beam radiation provides equivalent cancer control with less risk of adverse late effects to the thyroid compared to photon IMRT. Although proton beam therapy delivers less risk of acute damage to surrounding tissues, there is no data at this point that it has superior tumor control compared to photon IMRT. In addition, the long term toxic effects of proton beam therapy to surrounding healthy tissue are yet to be determined. While proton beam radiation therapy has been used most extensively in tumors of and near the CNS, it is also being used to treat other pediatric malignancies. PREP Pearls Proton beam radiation is increasingly being used in the treatment of pediatric central nervous system tumors with subsequently less acute damage to surrounding healthy tissues compared to photon intensity-modulated radiation therapy. The long-term toxic effects of proton beam therapy to surrounding healthy tissue are yet to be determined. Proton beam radiation releases the energy dose within a short distance of the particles' stop, called a Bragg peak, that can be adjusted for the volume of the tumor with less exit dose compared to photon intensity-modulated radiation therapy.

A 2-year-old boy develops a spontaneous intracranial hemorrhage. His birth history is significant for bleeding from the umbilical cord. His complete blood cell count, prothrombin time, partial thromboplastin time, activated fibrinogen level, and thrombin time are normal. Of the following, the BEST test to diagnose his coagulopathy is A.factor VIII activity B.factor XIII activity C.platelet function tests D.von Willebrand factor antigen

In this child with spontaneous intracranial hemorrhage and a history of bleeding from the umbilical stump, factor XIII deficiency is the most likely diagnosis. Although factor VIII deficiency may also lead to intracranial hemorrhage, it would cause significant prolongation of the activated partial thromboplastin time. Platelet function disorders and von Willebrand disease may present with normal screening coagulation tests, however the clinical pattern of bleeding is quite different in both of these disorders. The primary role of factor XIII is fibrin clot stabilization by cross-linking of fibrin monomers. It also plays roles in angiogenesis, wound healing, maintenance of pregnancy, bone metabolism, and cardioprotection. Factor XIII is present in plasma, platelets, and monocytes. It has 2 catalytic A subunits and 2 carrier B subunits. The majority of the A subunit is derived from the bone marrow. The B subunit is synthesized in the liver. Factor XIII circulates in plasma in an inactive form and requires thrombin and calcium for activation. When it is activated, it cross-links γ-chain fibrin molecules and also binds plasminogen to prevent fibrinolysis by plasmin. In addition, it plays a role in platelet adhesion and clot retraction. Factor XIII has a long half-life estimated to be between 9 and 19 days Factor XIII deficiency is an underdiagnosed, rare bleeding disorder, occurring in 1 in 1 million to 3 million people. It is inherited in an autosomal recessive or compound heterozygous manner. Umbilical cord bleeding at birth is seen in more than 80% of patients and is pathognomonic for this disorder. Other common types of bleeding are increased bruising, menorrhagia, and spontaneous intracranial hemorrhage. Intracranial hemorrhage occurs in 40% to 60% of patients in the first 2 decades. In women with severe factor XIII deficiency, recurrent miscarriage is common. Postsurgical course is complicated by bleeding and poor wound healing. Results of routine coagulation tests including prothrombin time, activated partial thromboplastin time, fibrinogen level, and thrombin time are normal in factor XIII deficiency. The diagnosis depends on clinical suspicion and the results of diagnostic tests that include: Clot solubility test Factor XIII activity Factor XIII antigen assay Inhibitor assay Molecular analysis In resource-rich countries, it is recommended to start testing with factor XIII activity since the clot solubility test is not reliable. In resource-poor countries, the initial test is usually the clot solubility test. If factor XIII activity is decreased, an antigen assay is performed to determine whether it is factor XIII-A or factor XIII-B deficiency. In older adults, factor XIII may also be decreased due to autoantibodies. Molecular analysis is performed by amplification and sequencing of factor XIII-A and/or factor XIII-B genes. The most common mutations affect the factor XIII-A gene and are missense mutations. PREP Pearls Bleeding from the umbilical cord is seen in more than 80% of patients with factor XIII deficiency and is highly suggestive of this deficiency. The results of routine coagulation tests are normal in factor XIII deficiency. When factor XIII deficiency is suspected, the best first test is a factor XIII activity assay. Poor wound healing is seen in patients with factor XIII deficiency.

A 16-year-old adolescent boy with severe hemophilia A has recently immigrated to the United States from Egypt. His mother reports that at the age of 8 years, he received a plasma transfusion after a traumatic injury. On surveillance testing, he is positive for hepatitis C IgG. He is clinically stable and has no hepatomegaly. His liver function tests have normal results. Of the following, the MOST accurate statement regarding the impact of the infection on this patient is A.he has a greater than 75% chance of developing cirrhosis B.he is at risk for hepatoblastoma C.he is at risk for hepatocellular carcinoma D.he is not at risk for long-term sequelae

More than 170 million individuals worldwide are infected with hepatitis C virus (HCV). It is a blood-borne infection, and a major portion of HCV infections are transmitted through blood component transfusion. Seroprevalence varies geographically, from approximately 1% to 3% in developed countries to up to 5% to 10% in developing countries (Figure). Individuals may be asymptomatic and not realize they are infected until long-term sequelae arise. Hepatitis C is also one of the major causes of chronic liver disease. Complications of chronic liver disease include cirrhosis and hepatocellular carcinoma. Cirrhosis occurs in 20% to 30% of individuals with HCV within 20 to 30 years after infection. Hepatoblastoma is not associated with HCV infection. Because patients with hematological disorders receive a disproportionate amount of blood components, they are at risk for transfusion-transmitted infections, which include HCV, HIV, and hepatitis B virus. There is no vaccine against HCV. Patients with hemophilia were particularly at risk for HCV infection in the 1970s to 1980s because of contaminated coagulation factor products; the prevalence of HCV infection in this cohort of patients reached almost 90%. Contemporary blood-donor screening and unit testing has greatly reduced the risk for transfusion-transmitted infections in the United States and other developed countries. However, there is still a risk, particularly in countries where the blood supply is less regulated. In patients with cirrhosis, 1% to 4% per year will develop hepatocellular carcinoma. Genotype 1 of HCV, detectable HCV RNA, and elevated liver enzyme levels are independent risk factors for the development of hepatocellular carcinoma. Approximately one-third of hepatocellular carcinoma cases are attributable to HCV. Persons with HCV also have higher mortality from prostate, thyroid, and esophageal cancers. PREP Pearls Transfusion of contaminated blood products is a major source of hepatitis C virus infection. Persons with hepatitis C virus are at risk for development of hepatic cirrhosis and hepatocellular carcinoma. Acute infection with hepatitis C virus is often asymptomatic

A 12-year-old girl has a 7-month history of pelvic pain, constipation, and menstrual irregularity. Menarche occurred at age 10 years. Physical examination reveals a very large palpable abdomino-pelvic mass. Computed tomography reveals a 20.3 cm by 14.0 cm by 28.5 cm enhancing solid and cystic mass arising from right ovary (Figure). The levels of serum ß-subunit of human chorionic gonadotropin and a-fetoprotein are not elevated. On exploratory laparotomy, an irregular right ovarian mass is found. The left ovary is normal. Sections of the tumor reveal solid, cystic, and hemorrhagic areas. Histologic stains reveal a pseudolobular pattern with cellular areas composed of spindle shaped and round to oval cells with vesicular nuclei and a moderate amount of eosinophilic cytoplasm separated by collagenous hypocellular areas. Mitoses and nuclear atypia are not seen. Tumor cells are diffusely positive for smooth muscle actin, vimentin, inhibin, α-inhibin, and CD199, and negative for S100 and desmin. Of the following, the MOST likely diagnosis is A.choriocarcinoma B.embryonal carcinoma C.sclerosing stromal tumor D.yolk sac tumor

Ovarian neoplasms represent 1% of pediatric tumors and are most common between the ages of 10 and 14 years. The World Health Organization classification separates ovarian neoplasms according to the most probable tissue of origin: surface epithelial, germ cell, sex cord-stromal, metastases, and miscellaneous. Two-thirds of all pediatric ovarian tumors are germ cell tumors. Epithelial and sex cord-stromal cell tumors occur less frequently. Sex cord-stromal tumors of the ovary include pure stromal tumors, pure sex cord tumors, and mixed sex cord-stromal tumors (Table). Sclerosing stromal tumor is a pure stromal tumor that is extremely rare and usually occurs in the second or third decade of life. It accounts for only 6% of sex cord-stromal tumors. As in this vignette, sclerosing stromal tumors do not secrete the oncofetoproteins, β-subunit of human chorionic gonadotropin or a-fetoprotein, which are produced in the more commonly occurring malignant ovarian germ cell tumors (eg, yolk sac tumor, embryonal carcinoma, choriocarcinoma, or mixed germ cell tumor). The most common clinical symptoms associated with a sclerosing stromal tumor are chronic pelvic pain, palpable abdominal mass, constipation, urinary frequency, dysuria, fever, amenorrhea, and vaginal bleeding. Sclerosing stromal tumors are thought to be derived from the perifollicular myoid stroma, a population of muscle-specific actin-positive elements from the theca externa. Sclerosing stromal tumors of ovary may be solid or multicystic masses. The diagnosis is made by pathologic examination of the tumor. Ovarian sclerosing stromal tumors have a characteristic microscopic pattern of pseudolobulation. The tumor cells are positive for the immunohistochemical stains for vimentin, smooth muscle actin, a-inhibin, inhibin, and CD199. They are negative for epithelial stains, desmin and S100. A sclerosing stromal tumor should always be considered in young patients with an ovarian mass. These benign tumors can be treated successfully with removal of the mass or oophorectomy. PREP Pearls Ovarian neoplasms represent 1% of pediatric tumors and most commonly occur between the ages of 10 and 14 years. The most common clinical symptoms associated with a pediatric ovarian tumor are chronic pelvic pain, palpable abdominal mass, constipation, urinary frequency, dysuria, fever, amenorrhea, and vaginal bleeding. Two-thirds of all pediatric ovarian tumors are germ cell tumors. Sex cord stromal tumors do not secrete oncofetoproteins, β-subunit of human chorionic gonadotropin and α-fetoprotein.

A 13-year-old adolescent girl with Ewing sarcoma has received 3 cycles of chemotherapy including 2 cycles of vincristine and doxorubicin. During her last cycle of vincristine/doxorubicin, she had excellent nausea control with dexamethasone, ondansetron, and lorazepam but developed syncope and demonstrated prolonged QTc on electrocardiogram. She is now due to receive her second course of ifosfamide and etoposide, a moderately emetogenic combination. Of the following, the MOST beneficial adjustment to her antiemetic regimen is to A.replace dexamethasone with methylprednisolone B.replace lorazepam with scopolamine C.replace ondansetron with dolasetron D.replace ondansetron with granisetron

Prolonged QTc syndrome is a potentially serious complication of the antiemetics classified as serotonin (5-hydroxytryptamine or 5-HT3) receptor antagonists (serotonin blockers). Of these antiemetics, ondansetron is the one most commonly used in children receiving chemotherapy. All members of this drug family have the potential to cause an asymptomatic or symptomatic prolonged QTc interval, particularly in patients receiving cardiotoxic agents that cause arrhythmias. A potentially life-threatening complication of serotonin blocker therapy is torsades de pointes, a ventricular arrhythmia triggered by prolonged QTc interval that may result in syncope and cardiac arrest. Characteristics of patients at high risk for torsades de pointes include female sex, hypokalemia, rapid infusion of the serotonin blockers, prolonged baseline QTc interval, and underlying cardiac dysfunction. Other more common but less severe adverse effects of serotonin blockers include headache, constipation, diarrhea, and dizziness. The patient in this vignette is receiving chemotherapy with alternating cycles of drugs with moderate emetogenic potential (ifosfamide/etoposide) and high emetogenic potential (vincristine/doxorubicin). Serotonin blockers currently include ondansetron, granisetron, and dolasetron. Granisetron has the lowest risk of prolonged QTc interval compared to ondansetron and dolasetron. Therefore, for the patient in this vignette, the best approach is to substitute granisetron for ondansetron and consider electrocardiography monitoring with subsequent doses. Removing serotonin blockers from her antiemetic options may ultimately be necessary if she demonstrates prolonged QTc interval after granisetron. Dolasetron is a potent antiemetic but is associated with a higher incidence of prolonged QTc interval as compared to ondansetron. Both the Italian Medicines Agency and the US Food and Drug Administration have issued warnings about these risks with the injectable form of the drug. Replacement of dexamethasone with methylprednisolone or replacement of lorazepam with scopolamine does not reduce the risk of prolonged QTc interval or arrhythmia and would result in less effective antiemetic therapy. PREP Pearls All serotonin (5-HT3) receptor antagonists (also known as serotonin blockers) are antiemetics with the potential to cause asymptomatic or symptomatic prolonged QTc syndrome, particularly in patients receiving concurrent cardiotoxic agents associated with arrhythmias. Risk factors associated with torsades de pointes, a life-threatening ventricular arrhythmia triggered by serotonin blocker antiemetics, include female sex, hypokalemia, rapid infusion of the serotonin blocker, prolonged baseline QTc interval, and underlying cardiac dysfunction. ABP Content Specifications(s)/Content Area Know the rationale for use, indications, mechanism of action, and toxicity of the serotonin S3 receptor antagonists

A 10-year-old girl is undergoing radiation therapy for treatment of an incompletely resected central nervous system high-grade glioma. As expected, the pathology shows vascular proliferation and areas of tumor necrosis. Her hemoglobin level is 11.2 g/dL (112 g/L). Of the following, the finding in this patient that may REDUCE her response to radiation therapy is A.decreased epidermal growth factor activity B.decreased hemoglobin level C.decreased vascular endothelial growth factor receptor-1 activity D.increased hypoxia-inducible factor 1α activity

Radiation induces ionization of DNA in cells and produces free radicals. The free radicals are oxidized by oxygen, resulting in permanent double-stranded DNA breaks. In the absence of oxygen, the DNA radicals are reduced by compounds containing sulfhydryl groups, and the double-stranded DNA breaks are repaired. Therefore, the presence or absence of molecular oxygen influences the effects of ionizing radiation, which is known as the oxygen effect. Radiation therapy is more effective in normoxic tissues and less effective in hypoxic tissues. Additional hypoxia-related factors that reduce the effectiveness of radiation include upregulation of BCL2 and increased expression of hypoxia-inducible factor 1α (HIF-1α). The upregulation of BCL2 results in changes in the cell death/survival signaling pathway and thus in increased cell survival following radiation. In normal tissues, HIF-1α is hydroxylated and then ubiquitinated for rapid degradation. However, in hypoxic tissues, HIF-1α is stabilized and activated. It induces the expression of genes responsible for the production of proteins including epidermal growth factor, glucose transporter, insulin-like growth factor-2, and vascular endothelial growth factor receptor-1, which are involved in neo-angiogenesis, glycolysis, and invasion/metastasis of tumor cells. The increased (not decreased) expression of these proteins leads to decreased effectiveness of radiation therapy. Tumor tissue may contain well-oxygenated tumor cells and hypoxic tumor cells. The rapid proliferation of tumor cells results in an imbalance of oxygen demand and supply. Tumor-induced neoangiogenesis results in the formation of blood vessels that are abnormal in both structure and function. Tumor cell hypoxia varies depending on the distance of tumor cells from blood vessels. Chronic hypoxic areas develop where tumor cells are exposed to the minimal levels of oxygen necessary for cell survival, and tumor necrosis occurs in areas about 100 μm from tumor blood vessels. An additional factor that influences tumor hypoxia and response to radiation therapy is disease- or treatment-related anemia. Adequate oxygenation is maintained in healthy tissue if the hemoglobin level is greater than 8 g/dL (80 g/L). In fact, normal tissue usually maintains adequate oxygenation with hemoglobin levels between 4 and 8 g/dL (40-80 g/L) by compensating with increased blood flow. In contrast, the oxygen supply to tumor tissue is significantly reduced, and hypoxia develops at hemoglobin levels below 10 g/dL (100 g/L). Tumor tissue cannot compensate with increased blood flow like normal tissue and is more prone to develop tissue hypoxia from anemia leading to a decreased response to radiation therapy. Hemoglobin levels are commonly maintained at 10 g/dL (100 g/L) or greater during radiation therapy. PREP Pearls The presence or absence of molecular oxygen influences the effects of ionizing radiation. Radiation therapy is less effective in damaging hypoxic tumor tissues. In tumor tissue, since hypoxia develops at hemoglobin levels below 10 g/dL (100 g/L), hemoglobin levels are commonly maintained at 10 g/dL (100 g/L) or greater during radiation therapy to improve effective tumoricidal activity.

While awaiting a matched unrelated donor hematopoietic stem cell transplant, a 3-year-old boy with relapsed acute myelogenous leukemia has completed reinduction chemotherapy. His absolute neutrophil count has been less than 500/µL (0.5 × 109/L) for 1 week. He has had persistent candidemia despite removal of the central venous catheter and appropriate antifungal therapy. He is in the intensive care unit and receiving inotropes to treat hypotension. His skin is without rash or sign of infection. Full head/body computed tomography, echocardiography, urine testing, and ophthalmologic examination reveal no focus of infection. The medical care team decides to administer granulocytes in hopes of improving his chance of survival. Of the following, the BEST granulocyte donor for this patient is A.ABO and RhD-blood group compatible B.available for daily apheresis C.pretreated with daily prednisone for 3 days D.pretreated with daily granulocyte colony-stimulating factor for 3 days

Severely neutropenic patients with bacterial infections who do not improve with appropriate antibiotic therapy or who have disseminated yeast/fungal infection may benefit from granulocyte transfusion from an ABO and RhD-compatible blood group donor. Granulocyte concentrates are usually contaminated with red blood cells and therefore should be ABO and Rh cross-matched to the recipient to avoid a transfusion reaction. Recipients of granulocyte transfusions are at risk for developing antibodies to granulocyte and/or human leukocyte antigens. These leukocyte antibodies may decrease the survival of infused granulocytes and lead to both transfusion-related acute lung injury and platelet refractoriness. Type-specific granulocytes may decrease these risks. Granulocyte donors do not undergo daily apheresis due to safety issues and decreasing yield of granulocyte collection. Granulocyte donors are pretreated with one single dose of granulocyte colony-stimulating factor (G-CSF) and a single dose of dexamethasone to stimulate maximum granulocyte release from the marrow. Daily prednisone for 3 days is not routinely used prior to granulocyte collection. Although G-CSF as a single dose prior to granulocyte collection has been used, daily G-CSF for 3 days is not recommended due to toxicity. Prior to apheresis collection, the granulocyte donor is given a combination of G-CSF 300 µg subcutaneously and dexamethasone 8 mg orally to mobilize granulocytes and increase the number of granulocytes collected. The rise in the donor's peripheral neutrophil count is greatest 12 hours after the administration of G-CSF and dexamethasone. Therefore, the apheresis collection is obtained approximately 12 hours after drug administration and usually takes 4 to 5 hours. After G-CSF and dexamethasone stimulation, the collection of 5 × 1010 to 10 × 1010 granulocytes is possible. Neutrophils collected after the administration of G-CSF alone exhibit prolonged survival in the transfused recipient compared to those stimulated by dexamethasone alone, possibly because these cells are released from the marrow compartment early and represent a younger population. When infused into the recipient, these granulocytes result in a measurable increase in the recipient's absolute neutrophil count. The granulocyte fraction of whole blood is collected using continuous flow apheresis blood collection. Apheresis is performed for at least 150 to 180 minutes to process approximately 7 L of whole blood using trisodium citrate as an anticoagulant. Hydroxyethyl starch, a sedimenting agent, is added to create better separation between the red blood cell and white blood cell layers. It is recommended that a single infusion dose of granulocytes contains 2 × 1010 to 3 × 1010 neutrophils. The minimum number of neutrophils required by the AABB Standards for Blood Banks and Transfusion Services is 1 × 1010. Granulocytes are stored at room temperature (22ºC). Since they rapidly lose viability ex vivo, they are infused into the recipient within a few hours of collection and no longer than 24 hours after collection. Granulocyte transfusions are given daily until the infection resolves or the absolute neutrophil count remains above 500/µL (0.5 × 109/L) for 48 hours. There are no published guidelines as to the frequency of collection and total number of donations permissible for normal donors. General guidelines limit granulocyte collections from an individual donor to twice weekly with a minimum of 48 hours between the 2 collections and a maximum of 24 donations per year. Cytomegalovirus (CMV) is an intraleukocytic pathogen, and CMV transmission is another potential risk with granulocyte transfusions. When possible, CMV-seronegative donors are selected for CMV-seronegative recipients. Granulocyte transfusion recipients are monitored for the development of CMV viremia. Granulocyte concentrates are irradiated prior to transfusion to prevent the risk of transfusion-associated graft-vs-host disease. PREP Pearls Severely neutropenic patients with bacterial infections who do not improve with appropriate antibiotic therapy or who have disseminated yeast/fungal infection may benefit from granulocyte transfusion from an ABO and RhD-compatible donor. The granulocyte donor may be given a single dose of granulocyte colony-stimulating factor subcutaneously and/or dexamethasone orally to mobilize granulocytes. It is recommended that a dose of granulocytes contains 2 × 1010 to 3 × 1010 neutrophils. Granulocytes are stored at room temperature (22ºC) and should be infused within a few hours of collection and no longer than 24 hours after collection.

A 15-year-old adolescent boy who received an allogeneic hematopoietic stem cell transplant for acute myeloid leukemia 22 days ago is reporting right upper quadrant abdominal pain. He is 4 kilograms (kg) heavier than his pretransplant weight. His sclerae are icteric. His abdomen is distended, and he has a palpable, tender, enlarged liver. His serum liver function test results are normal except for an elevation in serum total and direct bilirubin levels. Of the following, the medication from his conditioning regimen that is MOST likely associated with this complication is A.antithymocyte globulin B.busulfan C.cyclophosphamide D.melphalan

Sinusoidal obstructive syndrome or veno-occlusive disease (SOS/VOD) is caused by chemical damage to the liver sinusoids presenting as rapid weight gain, ascites, painful hepatomegaly, and direct hyperbilirubinemia, as in this vignette. Busulfan and total body irradiation (TBI) are the most common pre-hematopoietic stem cell transplant (HSCT) conditioning agents associated with SOS, usually within 30 days after HSCT. Antithymocyte globulin, cyclophosphamide, or melphalan are rarely associated with SOS after HSCT. The incidence of SOS/VOD after HSCT has been decreasing with the increasing use of pharmacokinetic monitoring of busulfan levels which helps to achieve appropriate busulfan dosing and by giving ursodiol as prophylaxis. The standard treatment of SOS includes fluid and sodium restriction, diuresis, analgesia, and intravenous colloid solution support. In 2016, the US Food and Drug Administration approved defibrotide, an anticoagulant derived from pig intestine or cow lung, for the treatment of children and adults with SOS after HSCT. Most agents used in HSCT conditioning regimens, such as busulfan, cyclophosphamide, melphalan, and TBI, will result in myelosuppression, temporary hair loss, and gastrointestinal distress within the first several days to weeks after stem cell infusion. Total body irradiation has also been associated with pulmonary and renal toxicities. Potential late effects of TBI include endocrinopathies, cataracts, infertility, and secondary malignancies, depending on the doses, fractionation, and other agents used. Other adverse effects of busulfan include stomatitis, mucositis, electrolyte imbalances, hepatitis, allergic reactions, and lowering of the seizure threshold. Late effects can include infertility and lung complications, including pulmonary fibrosis. Cyclophosphamide, an alkylating agent, is associated with prompt and delayed hemorrhagic cystitis. The risk of this adverse event is reduced with hyperhydration and the coadministration of mesna, which binds the cyclophosphamide byproduct, acrolein. A rare immediate toxicity of cyclosphosphamide is the syndrome of inappropriate diuretic hormone secretion. Reported late effects of cyclophosphamide include infertility, pulmonary fibrosis, and secondary leukemia. Melphalan, a nitrogen mustard, has been associated with acute mucositis, dermatitis, hepatitis, and pneumonitis. Late toxicities can include infertility and secondary malignancy. Antithymocyte globulin leads to significant lymphocyte-selective immunosuppression. During administration, patients can develop allergic complications such as fever, chills, hives, shortness of breath, and anaphylaxis. Within days, patients can experience gastrointestinal distress. Within weeks, transient thrombocytopenia and leukopenia can develop. Antithymocyte globulin is a major cause of viral reactivation of cytomegalovirus, herpes simplex virus, and Epstein-Barr virus within weeks after infusion for HSCT. It has a role in increasing the risk of developing posttransplant lymphoproliferative disease because ATG targets T cells. Rare late effects of ATG include serum sickness, gastrointestinal bleeding, and acute renal failure. PREP Pearls Busulfan pharmacokinetic evaluation with adequate busulfan dose adjustments and prophylactic ursodiol can lower the risk of developing sinusoidal obstructive syndrome (previously referred to as veno-occlusive disease) after hematopoietic stem cell transplant. Total body irradiation (TBI) has also been associated with pulmonary and renal toxicities. Potential late effects of TBI include endocrinopathies, cataracts, infertility, and secondary malignancies, depending on the dose, fractionation, and other agents used. Antithymocyte globulin and other lymphocyte-selective immunosuppressive medications used in hematopoietic stem cell transplant conditioning regimens increase the risk of cytomegalovirus, herpes simplex virus, and Epstein-Barr virus reactivation.

A 6-year-old boy has been diagnosed with an intermediate-risk embryonal rhabdomyosarcoma lesion. The tumor extends from the infratemporal fossa to the left temporal bone, involving the left petrous apex and carotid canal. It cannot be completely resected. At diagnosis, his cerebrospinal fluid and bone marrow studies were negative for metastatic disease. He has now completed 12 weeks of standard chemotherapy. Repeat magnetic resonance imaging shows a residual mass still involving the petrous apex and carotid canal, with a 65% reduction in tumor volume. Of the following, the next BEST step in management is to A.administer an intensified chemotherapy regimen B.irradiate the residual tumor C.re-stage with repeat bone marrow studies D.surgically resect the residual tumor

The current treatment of rhabdomyosarcoma (RMS) involves a combination of systemic chemotherapy and local control of the residual tumor with radiation and/or complete surgical resection to prevent recurrence. The patient in this vignette has intermediate-risk RMS in a parameningeal location, which is not amenable to complete surgical resection because of its location in the carotid canal noted both at diagnosis and after treatment with standard chemotherapy. Thus, radiation is needed to provide local control of the residual tumor. Re-staging with marrow studies and lumbar puncture is not needed during treatment in this vignette unless progression had been observed. This patient has demonstrated tumor reduction in response to initial chemotherapy and does not need different, intensified chemotherapy but will need a continuation of the current chemotherapy regimen after radiation therapy. Rhabdomyosarcoma is the most common soft tissue sarcoma in children, with an incidence of 4.5 cases per 1 million children. Rhabdomyosarcoma occurs in 2 distinct histological subtypes; embryonal (ERMS) and alveolar (ARMS). The embryonal subtype is more common, and occurs more frequently in younger children. The alveolar subtype is associated with the PAX3-FOXO1 or PAX7-FOXO1 translocation and has a worse prognosis compared to ERMS. Rhabdomyosarcoma most commonly arises in the head and neck region, followed by the genitourinary tract, then the extremities. Uncommon sites include the chest wall, perineal region, retroperitoneum, and biliary tract. Favorable sites or primary sites with a better prognosis include the orbit, head and neck (excluding parameningeal), genitourinary tract (excluding kidney, bladder, and prostate), and the biliary tract. Given the infratemporal location of the localized tumor in this vignette, this patient is considered to have parameningeal disease, an unfavorable site that confers an intermediate-risk prognosis and a multimodal treatment approach. Chemotherapy with vincristine, dactinomycin, and cyclophosphamide (VAC) is considered standard chemotherapy for low- and intermediate-risk groups. Irinotecan has also been incorporated into treatment regimens for intermediate-risk RMS. High-risk RMS (any metastatic ARMS or ERMS) often involves therapy intensified in addition to VAC, such as irinotecan, doxorubicin, ifosfamide, and etoposide. Local control for RMS is imperative for a successful outcome and can be achieved with surgical resection and/ or radiation. Complete resection of the primary tumor with a wide, tumor-free tissue margin is recommended whenever possible. Adequate tumor-free margins around the tumor are beneficial unless sacrificing the surrounding healthy tissue results in unacceptable loss of function or form. Resectability is an important predictor of outcome. However, even group III patients (defined as gross residual disease after biopsy only) with localized disease who were treated with chemotherapy and radiation had equivalent outcomes compared to those who underwent surgical resection (5-year failure-free survival of 75% in the Intergroup Rhabdomyosarcoma Study IV) . Rhabdomyosarcoma is a radiosensitive tumor, so if complete resection is not possible, radiation therapy is recommended. Response to induction chemotherapy and correlation with survival rates vary among clinical studies. The Intergroup Rhabdomyosarcoma Study Group and Children's Oncology Group found no association between initial response to chemotherapy and survival, while other studies show a survival benefit based on response assessed by imaging. In contrast, other sarcomas, such as Ewing sarcoma and osteosarcoma, microscopic response to neoadjuvant chemotherapy is predictive of outcome. Positron emission tomography (PET) may be beneficial in assessing response to chemotherapy, but, at this time, therapy is not altered based on PET response alone. Even after multimodal treatment (chemotherapy, radiation, and/or surgery), there can still be macroscopic evidence of RMS on imaging. This is often necrotic tumor or differentiated rhabdomyoblasts. Aggressive surgical resection and/or chemotherapy intensification are not recommended when there is a residual mass that has demonstrated shrinking with standard chemotherapy alone. PREP Pearls Rhabdomyosarcoma is treated with systemic chemotherapy and when necessary, local control with radiation or surgery. Surgical resection is attempted in rhabdomyosarcoma only when the surgeon is able to resect the localized tumor with an adequate margin of healthy tissue, and/or when resection does not compromise form or function. Rhabdomyosarcoma treatment may not result in a complete response on anatomic imaging, and while a residual mass is followed closely, it does not require intensification in chemotherapy if it is otherwise stable or regressing.

A 3-year-old child with high-risk, pre-B-cell acute lymphocytic leukemia in remission is due to receive his cycle 1, day 29 maintenance chemotherapy, which includes standard doses of 6-mercaptopurine, methotrexate, vincristine, and prednisone. His laboratory data are shown: Laboratory Test Result White blood cell count 400/µL (0.4 × 109/L) Absolute neutrophil count 150/µL (0.15 × 109/L) Hemoglobin 10.7 g/dL (107 g/L) Platelet count 68 × 103/µL (68 × 109/L) Peripheral blood smear Negative for blasts Serum chemistry profile Normal Currently available results from genetic assays reveal a heterozygous TPMT mutation and a normal NUDT15. Methotrexate and 6-mercaptopurine are temporarily discontinued until blood count recovery. Of the following, the next step that would provide the BEST chance for cure with the least toxicities is to A.adjust 6-mercaptopurine and methotrexate to achieve an absolute neutrophil count between 500/µL and 1,500/µL B.adjust prednisone to achieve an absolute neutrophil count between 500/µL and 1,500/µL C.permanently discontinue methotrexate D.replace 6-mercaptopurine with 6-thioguanine

Thiopurine methyltransferase (TPMT) metabolizes the thiopurines, 6-mercaptopurine (6-MP, also known as purinethol) and 6-thioguanine, to their inactive methylated forms. Deficiency in TPMT function causes active 6-MP to persist in circulation with the potential for increased toxicities. After repeated daily doses (as occurs in acute lymphocytic leukemia (ALL) chemotherapy), the 6-MP concentration rises to levels that contribute to myelosuppression and hepatotoxicity. In this vignette, the patient's heterozygous genotype for TPMT mutation was unknown prior to the start of maintenance chemotherapy, and he was unintentionally given standard 6-MP dosing. He has had his 6-MP and methotrexate temporarily discontinued until his severe neutropenia and thrombocytopenia resolve. When his absolute neutrophil count and platelet count return to the designated protocol standards, 6-MP (and methotrexate) will need to be restarted at lower doses to prevent recurrent myelotoxicity while maintaining therapeutic levels associated with an absolute neutrophil count between 500/µL and 1,500/µL (0.5-1.5 × 109/L). Although methotrexate metabolism is not affected by TPMT mutations, methotrexate doses are commonly adjusted during maintenance therapy for ALL, along with 6-MP, to maintain efficacy and prevent toxicity. 6-Thioguanine (6-TG), another purine analog, is also metabolized and inactivated by TPMT. WIth deficient TPMT function, 6-TG may become as toxic as 6-MP and therefore substitution of 6-MP with 6-TG would not help this patient. Discontinuing methotrexate would result in inadequate treatment of the patient's leukemia and would not address the inherent toxicity of high doses of 6-MP in the setting of TPMT heterozygosity. Similarly, adjusting prednisone doses would have no impact on myelosuppression. Possible indications for prednisone dosing adjustments include corticosteroid-related osteonecrosis or hallucinations. Nudix hydrolase 15, which is encoded by NUDT15, is involved in the metabolism of both 6-MP and 6-TG by facilitating the final metabolic steps in the hypoxanthine pathway (Figure). Thiopurine myelotoxicity risk based on NUDT15 mutations has been reported in approximately 10% of East Asian and 4% of Hispanic/Native American children. Children who are homozygous deficient for TPMT activity due to mutations in either TPMT or NUDT15, or due to two polymorphisms for NUDT15 (heterozygosity at R139C and R139H) require up to a 90% dose reduction of the thiopurine administered. Children who are heterozygous for TPMT and have had myelosuppression are administered up to a 50% reduction in 6-MP dose, compared to non-affected children. Children who are heterozygous for NUDT15 need to be monitored closely since most might need thiopurine dose reduction to prevent severe myelosuppression. PREP Pearls TPMT genotype and phenotype testing is routinely done in patients with acute lymphocytic leukemia to help determine therapeutic dosing of 6-mercaptopurine while avoiding excessive myelotoxicity. The most common approach to 6-mercaptopurine (6-MP) dose modification for individuals with homozygous or heterozygous TPMT or NUDT15 mutations is to reduce the 6-MP dose to the maximum tolerated dose, which may be a lower dose than the standard dose needed by unaffected patients.

A 4-year-old previously healthy girl presents with fever, bone pain, and easy bruising. Evaluation reveals B-precursor lymphoblastic leukemia. Genetic testing performed prior to chemotherapy shows that she is homozygous for a variant allele associated with markedly reduced thiopurine methyltransferase function. Of the following, the BEST treatment modification to recommend for this patient is A.administration of allopurinol in conjunction with mercaptopurine B.25% decrease in the starting dose of mercaptopurine C.90% decrease in the starting dose of mercaptopurine D.omission of thioguanine

Thiopurines, including mercaptopurine and thioguanine, are a class of purine analog drugs used in the treatment of acute leukemias. Common adverse effects of thiopurines include dose-dependent myelosuppression and hepatotoxicity. The hepatotoxicity risk is higher with thioguanine than mercaptopurine when given long term, so thioguanine is used only in short courses and not as a maintenance therapy drug. Veno-occlusive disease/sinusoidal obstructive syndrome (hyperbilirubinemia, fluid retention and ascites, hepatomegaly), sometimes with portal hypertension, has been associated with thioguanine use. Oral mercaptopurine and thioguanine are prodrugs with low bioavailability. Once ingested, they undergo hepatic and intestinal metabolism along several pathways. Activation along the hypoxanthine mercaptopurine phosphoribosyltransferase pathway results in the formation of the major active metabolites, thioguanine nucleotides (TGNs), which are cytotoxic because of their incorporation into DNA. The 6-TGN metabolites are associated with both myelosuppression and efficacy, whereas the 6-methylmercaptopurine nucleotide (6-MMPN) metabolites are associated with hepatotoxicity. Inactivation of mercaptopurine occurs via methylation by thiopurine methyltransferase (TPMT) and oxidation by xanthine oxidase (XO). Thioguanine is inactivated via the TPMT pathway and a deamination pathway. Unlike mercaptopurine, thioguanine is not dependent on XO. Genetic polymorphisms in TPMT are associated with variable metabolism of both mercaptopurine and thioguanine, leading to increased toxicity and decreased efficacy. Dose modification of thiopurines is therefore required for some polymorphisms. About 90% of the population has normal TPMT activity with 2 functional alleles. Even in these patients, monitoring is needed for potential toxicity of the drug. About 10% of the population is heterozygous for the nonfunctional TPMT allele and is therefore at higher risk for drug toxicity. Patients who are homozygous for the nonfunctional alleles (0.3%) have severe and life-threatening myelosuppression at standard doses because of reduced inactivation of the drug and increased levels of TGNs. Genotyping or phenotyping for TPMT is recommended to identify patients who are at increased risk for thiopurine toxicity. Reduced dosing of thiopurines is recommended for patients who have 1 or 2 nonfunctional TPMT alleles, with disease-specific guidelines sometimes available. A dramatic dose reduction is recommended for homozygous individuals, with a 90% dose reduction often suggested as a starting point. There are over 40 reported variant alleles of TPMT. The wild-type allele associated with normal TPMT activity is designated as TPMT*1. Four variant alleles are responsible for over 90% of the nonfunctional or reduced-function alleles: TPMT*2, TPMT*3A, TPMT*3B, and TPMT*3C. The frequency of these alleles varies by racial/ethnic population. Genetic testing is preferred in leukemia patients, because phenotype (enzyme activity) results can vary according to altered hematopoiesis and chemotherapy effects in leukemia patients. However, genotype testing usually focuses on the most common variants and may miss rare or previously undiscovered variants, so phenotype (enzyme activity) testing is helpful in some individuals. Allopurinol is an XO inhibitor that was designed to increase the bioavailability of mercaptopurine. Mercaptopurine inactivation via the XO pathway is inhibited by allopurinol administration, which can result in increased drug toxicity. However, if allopurinol is administered with reduced doses of mercaptopurine, the hypoxanthine mercaptopurine phosphoribosyltransferase pathway is enhanced and the 6-TGN levels increase. For reasons that are not understood, there is a reduction in 6-MMPN levels with this drug combination as well. Some individuals have altered thiopurine metabolism with high 6-MMPN levels and low 6-TGN levels, and these patients are at risk for both hepatotoxicity and reduced response. In patients with inflammatory bowel disease who have this skewed metabolism, the coadministration of allopurinol and reduced-dose mercaptopurine results in shunting of the metabolism to a more effective and less hepatotoxic pathway. This approach has been replicated in a small number of leukemia patients who have this metabolite profile, but effects on long-term outcomes are not yet known. Allopurinol does not affect thioguanine metabolism since the XO pathway is not involved in thioguanine metabolism. PREP Pearls Thiopurines, including mercaptopurine and thioguanine, are used in the treatment of acute leukemias, and common adverse effects include dose-dependent myelosuppression and hepatotoxicity. Polymorphisms in TPMT are associated with altered thiopurine metabolism and may result in a moderate or severe increase in the risk of myelosuppression from thiopurines. A 90% dose reduction of oral mercaptopurine and/or thioguanine is recommended for individuals who are homozygous for TPMT polymorphisms.

A 7-year-old girl with high-risk acute lymphoblastic leukemia in delayed intensification has developed a diffuse, vesicular and pruritic rash over her trunk and extremities with new lesions appearing over the past 5 days. She is alert and febrile but not tachypneic. Her 15-month-old unimmunized sibling developed a similar rash last week that is resolving. A complete blood cell count and liver function tests are ordered, and a vesicle is swabbed for viral polymerase chain reaction testing. Empiric treatment is started while awaiting the test results. Of the following, the BEST choice for empiric treatment of this patient is A.intravenous immunoglobulin B.oral acyclovir C.oral valacyclovir D.intravenous acyclovir

This immunocompromised patient has varicella infection after exposure to her sibling's primary infection. The diffuse rash suggests primary varicella infection, not varicella zoster reactivation which typically demonstrates a dermatomal distribution. Immunocompromised patients with varicella zoster virus (VZV) or cutaneous herpes simplex virus (HSV) are at high risk for viral dissemination to the lungs (varicella pneumonia), liver, and brain. Varicella pneumonia can occur in the first week of illness with a high mortality rate. Intravenous acyclovir and hospitalization for supportive care are highly recommended for all immunocompromised patients with varicella infection to prevent further viral sequelae. The patient in this vignette is immunocompromised because of her chemotherapy for acute lymphoblastic leukemia. Immunocompromised oncology patients include patients on high-dose corticosteroid therapy for over 14 days, patients diagnosed with acute myelogenous leukemia, patients receiving chemotherapy for solid tumors, and hematopoietic stem cell transplant recipients. Early initiation of intravenous antiviral therapy improves outcomes. Thus, empiric intravenous treatment is needed without waiting for viral testing results. The standard treatment for VZV in an immunocompromised individual at risk for developing disseminated varicella remains intravenous acyclovir. Oral acyclovir has poor bioavailability and is not used to treat primary varicella infection in the immunocompromised patient. Oral valacyclovir has improved bioavailability compared to oral acyclovir and is recommended for use in immunocompetent patients with infection. Varicella zoster immune globulin or other intravenous immunoglobulin preparations are not effective as single agent therapy once the viral exanthem has presented. Intravenous immunoglobulin products have been shown to prevent viral infection in immunocompromised patients when given within 96 hours of exposure to another individual with active varicella infection. In addition, if available, varicella zoster immune globulin can be given up to 10 days after viral exposure to reduce severity or prevent disease, but is recommended within 96 hours whenever possible. Immunocompetent patients with immunity to varicella do not necessarily need treatment for or prophylaxis after exposure to varicella. Patients with decreased cell-mediated immunity are at a high risk for infections with DNA viruses such as VZV and HSV, both from primary exposure as well as reactivation of latent infections. Oral acyclovir, oral valacyclovir, or famciclovir can be prophylactically used to prevent both HSV and VZV reactivation in the immunocompromised population. PREP Pearls As compared to immunocompetent patients, immunocompromised patients are at a high risk of morbidity and mortality from varicella and herpes zoster dissemination to the lungs, liver, and brain. Intravenous acyclovir is recommended to treat varicella infections in immunocompromised patients. Varicella zoster immune globulin or other intravenous immunoglobulin preparations are not effective as a single agent in treating an established varicella infection. However, when administered as early as possible after exposure to active varicella, these agents can help reduce severity or prevent disease progression.

A 15-year-old adolescent boy has sudden onset of dyspnea, chest pain, and swelling of his right calf. Computed tomographic angiography of the chest reveals a right-sided pulmonary embolism. Doppler ultrasonography also confirms the presence of a right popliteal thrombosis. An initial thrombophilia evaluation including factor V Leiden mutation, prothrombin G20210 mutation, quantitative D-dimer, complete blood cell count, and a creatinine level has already been sent. Of the following, the additional laboratory study indicated prior to starting anticoagulation therapy is A.ADAMTS13 activity B.lupus anticoagulant assay C.thrombin time D.von Willebrand factor antigen

When a child is diagnosed with an acute thromboembolic event, especially when seemingly idiopathic or unprovoked, useful testing for inherited and acquired thrombophilic conditions includes the presence of factor V Leiden and prothrombin G20210A mutations, lupus anticoagulant, antiphospholipid antibodies, and deficiencies in protein C, protein S, or antithrombin (Table). The presence of any of these conditions may determine the need for long-term anticoagulation therapy to prevent thrombosis recurrence. However, protein C, protein S, and antithrombin levels may be falsely decreased in the setting of an acute thrombotic episode. Therefore, testing for the three later conditions should be delayed until anticoagulation therapy has been completed. Laboratory evaluation should also include a complete blood cell count to rule out concomitant anemia or thrombocytopenia, which may suggest the need for transfusions prior to the start of anticoagulation therapy. A baseline creatinine level is helpful to determine anticoagulant dosing based on renal function. A pregnancy test (β-human chorionic gonadotropin) is required for female individuals of reproductive age when a vitamin K antagonist is considered for long-term anticoagulation therapy because of the known teratogenic risk. D-dimer and factor VIII activity levels can be obtained at diagnosis and periodically monitored as predictors for thrombosis recurrence to determine the length of anticoagulation therapy for both children and adults (Suggested Reading 2). An initial thrombophilia evaluation does not include ADAMTS13 activity, von Willebrand antigen level, or a thrombin time. In children, more than 90% of venous thromboembolism (VTE) cases are provoked or risk associated, with the most common risk factor being the presence of a central venous catheter. Malignancy, especially acute leukemia treated with asparaginase, is another well-established risk factor. Children with congenital heart disease, nephrotic syndrome, inflammatory bowel disease, and systemic lupus erythematosus are also at high risk for VTE development. The VTE risk is also increased in children with major traumatic injuries, particularly vascular or orthopedic injuries. The clinical presentation of deep venous thrombosis (DVT) is dependent on the affected anatomic occlusion. An upper or lower extremity DVT presents with unilateral acute and painful limb swelling. The presence of a palpable cord in the popliteal fossa or the discomfort observed behind the knee when performing a forced dorsiflexion of the foot (Homan's sign) is commonly observed in the setting of a lower extremity DVT. When the superior vena cava is affected, swelling of the face and neck can occur in conjunction with bilateral periorbital edema. A patient with cerebral sinovenous thrombosis will usually have neurological signs (eg, papilledema and cranial nerve palsy). Cerebral sinovenous thrombosis presents with intense headache, blurred vision, and sometimes seizures. Renal thrombosis is associated with acute hematuria. Patients with renal thrombosis may have concomitant thrombocytopenia and, in severe bilateral cases, uremia. Pulmonary embolism (PE) usually presents with an acute onset of dyspnea associated with pleuritic chest pain. Children with bilateral PE might have hypoxemia. Proximal PEs have been associated with cyanosis and sudden collapse. Interestingly, some patients with PE can be asymptomatic, especially when segmental branches of the lungs are compromised. PREP Pearls Testing for the presence of inherited or acquired thrombophilic conditions is necessary in children presenting with seemingly idiopathic thrombotic episodes. D-dimer and factor VIII activity levels are considered predictors for thrombosis recurrence and have been used to determine the length of anticoagulation therapy. In children, more than 90% of venous thromboembolism (VTE) cases are provoked or risk associated, with the most common risk factor being the presence of a central venous catheter. Patients with pulmonary embolism usually present with an acute onset of dyspnea associated with pleuritic chest pain but some patients with PE can be asymptomatic, especially when segmental branches of the lungs are compromised.

A 2-week-old neonate born at 33 weeks' gestation to consanguineous parents is in the pediatric intensive care unit with bacterial sepsis after recovering from omphalitis. He was born small for gestational age, failed his newborn hearing screen, and has persistent thrush. Flow cytometry data from peripheral blood are shown: Laboratory Test Result CD3+ cell count 20/μL CD4+ cell count 14/μL CD8+ cell count 6/μL CD19+ cell count 9/μL CD56/16+ cell count 11/μL Of the following, the additional hematologic finding that is MOST likely is A.thrombocytosis B.macrocytic anemia C.monocytosis D.neutropenia

With the advent of T-cell receptor excision circle (TREC) newborn screening, patients with severe combined immunodeficiency (SCID) and other disorders with severe T-cell deficiency are often detected prior to the onset of severe infectious and inflammatory complications. However, since TREC newborn screening is variably deployed and many countries lack the infrastructure to support this newborn screening, clinical vigilance for patients with signs and symptoms of severe T-cell deficiency remains paramount. Additionally, some forms of SCID present within the first few postnatal days, limiting the utility of TREC newborn screening in those settings. The patient in this vignette has a T-cell negative (CD3, CD4, CD8), B-cell negative (CD19), natural killer (NK)-cell negative (CD56/16) SCID phenotype complicated by early onset severe bacterial infections including omphalitis and sepsis. Additionally, he has sensorineural hearing loss. His condition is most likely inherited in an autosomal recessive fashion since his parents are consanguineous. These clinical features make reticular dysgenesis (RD) caused by a mutation in AK2 the most likely diagnosis. Reticular dysgenesis accounts for less than 2% of SCID cases. Patients with RD have not only a profound deficiency in the number and function of T cells, but also are well characterized to have severe neutropenia/agranulocytosis leading to the early onset of bacterial infections. Many patients with RD are born premature and small for gestational age. The majority of patients with RD develop bacterial sepsis and/or omphalitis prior to 4 weeks after birth, often in the first week after birth. They are also at risk for disseminated Candida infections. In addition to severe lymphopenia and neutropenia, roughly 45% of patients have normocytic anemia and/or thrombocytopenia. They do not have monocytosis. Bone marrow evaluation may show hypoplasia or hyperplasia and often demonstrates a promyelocytic arrest in the myeloid lineage. These patients have sensorineural hearing loss. Reticular dysgenesis can be cured via hematopoietic stem cell transplant with myeloablative conditioning, which minimizes the risk of graft failure and ensures engraftment of the myeloid lineage as well as the lymphoid lineages. Many patients with SCID have normal absolute lymphocyte counts, consisting of nonfunctional B and/or NK cells. Thus, a normal absolute lymphocyte count does not rule out severe T-cell deficiency. In fact, the most common type of SCID caused by X-linked mutations in the common gamma chain (ILR2G) has a T-B+NK- phenotype. While B cells may be present, they are not functional and do not generate adequate amounts of immunoglobulins, leading to pan-hypoglobulinemia. Flow cytometry must be used to quantify the number of CD3+ T cells. The typical SCID definition requires a CD3+ cell count of less than 300/μL and less than 10% of normal proliferation to phytohemagglutinin, an ex vivo test to assess the functional capacity of T cells. Patients with SCID and other forms of severe T-cell deficiency require routine Pneumocystis jiroveci prophylaxis and regular intravenous immunoglobulin supplementation. Additionally, because of the risk of cytomegalovirus transmission and the risk of transfusion-associated graft-vs-host disease, which has a high mortality rate, all blood products must be leukoreduced and irradiated. Live viral vaccines and well water should be avoided. Mothers should be counseled regarding the risk of cytomegalovirus transmission through breast milk. Additionally, all patients with severe T-cell deficiency must be assessed for the presence of maternal engraftment (ie, the presence of maternally derived T cells in the infant's peripheral blood). PREP Pearls Reticular dysgenesis is a rare form of severe combined immunodeficiency associated with severe neutropenia and sensorineural hearing loss. Some patients with severe combined immunodeficiency (SCID) may have a normal absolute lymphocyte count consisting of nonfunctional B cells and/or natural killer cells, but all patients with SCID have extremely low numbers of T cells. Patients with severe T-cell deficiency require irradiated blood products to protect against transfusion-associated graft-vs-host disease. Hematopoietic stem cell transplant is curative for reticular dysgenesis. A myeloablative conditioning regimen is needed to minimize risk of graft failure.

A 5-year-old boy with standard-risk B-cell acute lymphoblastic leukemia on maintenance therapy is seen in the office for his second episode of direct hyperbilirubinemia and elevation of liver transaminase levels. With his last episode, his oral methotrexate and mercaptopurine were held for 2 weeks with resolution of his symptoms. He was started on a 50% reduced dose of mercaptopurine. His complete blood count showed that his white blood cell count, neutrophils, and platelet count were acceptable to continue therapy. His thiopurine methyltransferase activity genotype was tested in the past and reported as homozygous wild type. Of the following, the MOST appropriate management of his medications after his symptoms resolve is to A.add allopurinol with a 50% reduction in mercaptopurine B.decrease mercaptopurine by 50% C.decrease mercaptopurine by 50% and increase methotrexate by 25% D.discontinue mercaptopurine therapy

A Hepatic toxicity due to mercaptopurine is a known side effect. Daily mercaptopurine therapy is a mainstay of acute lymphoblastic leukemia (ALL) maintenance therapy. Reducing the mercaptopurine dose is a common management strategy, particularly in thiopurine methyltransferase (TPMT) homozygous-deficient patients due to myelotoxicity. However, constant dose reductions of mercaptopurine risks relapse. Increasing the dose of methotrexate to compensate for mercaptopurine may not prevent mercaptopurine-associated hepatotoxicity. Concomitant use of allopurinol with mercaptopurine, the preferred answer, has been shown to increase tolerability of mercaptopurine in inflammatory bowel disease patients and small case series of patients with ALL. Thiopurines, such as 6-mercaptopurine and 6-thioguanine, are prodrugs that are converted to the thioguanine metabolites (6TGN), which can be incorporated into DNA and cause apoptosis via cross-linkage and single-strand breaks. They are converted into the cytotoxic nucleotides via hypoxanthine-guanine phosphoribosyl transferase (HGPRT). Unlike thioguanine's direct conversion to 6TGN, conversion of mercaptopurine requires 2 additional steps via inosine monophosphate dehydrogenase (IMDP) and guanosine monophosphate synthetase (GMPS). HGPRT converts mercaptopurine to 6-thioinosine 5' monophosphate, which is converted by TPMT into 6-methylmercaptopurine nucleotides and 6-methylmercaptopurine (6MMPN/6MMP), which is thought to cause hepatotoxicity. Patients who have significant hepatotoxicity are thought to have an increased ratio of 6MMPN/6MMP vs 6TGN. Allopurinol decreases hyperuricemia through inhibition of xanthine oxidase. It has also been shown to alter thiopurine metabolism. However, concomitant use in inflammatory bowel disease patients has shown that allopurinol has decreased the gastrointestinal and hepatic toxicities of mercaptopurine while improving remission rates. The mechanism of action is thought to be due to inhibition of xanthine oxidase, which prevents the conversion of mercaptopurine to thiouric acid, an inactive metabolite. This leads to an increase in thioxanthine which is thought to inhibit TPMT. Other studies have shown that allopurinol also increases HGPRT activity, thereby decreasing 6MMPN/6MMP production in favor of 6TGN production. Two retrospective chart reviews of pediatric patients with ALL who had gastrointestinal or hepatic toxicities were evaluated for elevated 6MMPN/6MMP due to mercaptopurine. Patients were started on allopurinol. Most patients had resolution of their gastrointestinal or hepatic toxicities with a reduction in 6MMPN/6MMP levels and increase in 6TGN levels. In one study, one patient had a recurrence of the hepatotoxicity and one patient had a relapse of their leukemia. The other study showed no significant side effects with the addition of allopurinol. The limitations of the studies were they were single-center retrospective chart reviews. In addition, many of the patients remained on a reduced dose of mercaptopurine compared to the original despite dose escalation. Larger prospective multi-centered trials will be needed to assess measuring 6MMPN/6MMP and 6TGN levels and starting allopurinol prior to development of toxicities and prevent dose reduction of mercaptopurine. PREP Pearls Thiopurines are prodrugs that are metabolized to 6-thioguanine nucleotides that exert their cytotoxic effects when incorporated into DNA synthesis. Mercaptopurine is also metabolized by thiopurine methyltransferase, leading to 6-methylmercaptopurine and 6-methylmercaptopurine nucleotides. Increased ratio of these metabolites compared to 6-thioguanine nucleotides contributes to increased gastrointestinal and hepatic toxicities. Allopurinol decreases the production of 6-methylmercaptopurine and 6-methylmercaptopurine nucleotides and increases the 6-thioguanine nucleotides, thereby improving the therapeutic effect while decreasing the gastrointestinal and hepatic toxicities.

A previously healthy 11-year-old girl presents with a 2-3 day history of progressively worsening headache, epistaxis, and bruising. Laboratory data are shown: Laboratory Test Result White blood cell count 76,000/µL (76 × 109/L) Hemoglobin 5.9 g/dL (59 g/L) Platelet count 22 × 103/µL (22 × 109/L) INR 1.8 Fibrinogen Low D-dimer Elevated A peripheral blood smear shows myeloblasts with Auer rods, folded nuclei, and cytoplasmic granules. Of the following, the cytogenetic abnormality that is MOST likely to be present in the myeloblasts is A.inv(16) B.t(8;21) C.t(9;22) D.t(15;17)

Acute myeloid leukemia (AML) is classified in the World Health Organization (WHO) classification system according to a combination of morphologic, immunophenotypic, clinical, and cytogenetic/molecular features. The category of "AML with recurrent genetic abnormalities" includes commonly seen variants with prognostic value. The majority of de novo AML cases are associated with chromosomal abnormalities, which frequently have diagnostic, prognostic, and therapeutic significance. The cytogenetic abnormalities and their associated mutated genes which are diagnostic of AML, regardless of bone marrow blast count are: t(15;17); PML-RARA t(8;21); RUNX1-RUNX1T1 inv(16) or t(16;16); CBFB-MYH11 The balanced translocation t(15;17) characterizes acute promyelocytic leukemia (APL). The malignant cells have a characteristic morphologic appearance with a nucleus that may be folded, reniform, or bilobed. Coarse azurophilic granules and Auer rods are usually present. The unique clinical presentation of APL is characterized by promyelocytic infiltration of the bone marrow, disseminated intravascular coagulation (DIC), and fibrinolysis. It has a high rate of early mortality and should be considered an oncologic emergency because of the risk of hemorrhage from DIC, which can worsen during induction of chemotherapy. The PML-RARA fusion gene that occurs as a result of this translocation contains most of the coding sequences of the promyelocytic leukemia gene (PML) and the DNA binding/ligand-binding domains of the α-retinoic acid receptor gene (RARA). The fusion protein has decreased sensitivity to retinoic acid, which may lead to persistent transcriptional repression and prevention of promyelocyte differentiation. Pharmacologic doses of all-trans retinoic acid (ATRA, tretinoin) can overcome this effect and result in terminal differentiation of promyelocytes. Characteristic myeloid blasts and the presence of DIC should prompt consideration of a diagnosis of APL. Because of the high early mortality risk, it is critical to start treatment with ATRA as soon as the diagnosis is suspected. When APL is managed effectively with prompt initiation of ATRA, appropriate chemotherapy, and supportive care, the prognosis of APL is excellent. The t(8;21)(q22;q22) mutation is the most common cytogenetic abnormality in children with AML and confers a favorable prognosis. Distinctive pathologic features of myeloblasts include indented nuclei, basophilic cytoplasm, and easily identifiable Auer rods. Leukemogenesis is thought to occur as a result of altered transcriptional regulation of RUNX1 target genes and activation of new genes that prevent apoptosis or cell differentiation. The inv(16) or t(16;16) mutation in AML is often associated with abnormal eosinophils in the bone marrow and a favorable response to conventional AML chemotherapy. It can also present as an extramedullary myeloid sarcoma. This cytogenetic abnormality results in the juxtaposition of the myosin, heavy chain 11, smooth muscle gene (MYH11) and the core-binding factor, B subunit gene (CBFB). The following subtypes of AML require at least 20% blasts in the bone marrow for diagnosis. These genetic abnormalities confer prognostic value: t(9;11); KMT2A-MLLT3 t(6;9); DEK-NUP214 inv(3) or t(3;3); MECOM (EVI1) t(1;22); RBM15-MKL1 Rearrangements involving 11q are common in patients with a monoblastic or myelomonocytic component to their AML. Over 100 rearrangements have been identified involving 11q23.3, with many different translocation partners identified in AML. These translocations involve KMT2A, and the most common translocation in childhood AML involves KMT2A on chromosome 11 and MLLT3 at 9p21.3. These cases may present with DIC, elevated white blood cell count, and leukemic infiltration of the gingiva or skin. The prognostic impact of MLL rearrangements is not entirely clear. Overall survival for pediatric AML appears to be reduced in the presence of MLL rearrangements but the outcomes differ based on the specific translocation partner. The t(6;9) mutation in AML is associated with basophilia, pancytopenia, and marrow dysplasia. About 70% of cases have FLT3-ITD mutations, and this subtype is associated with an unfavorable prognosis. Abnormalities of 3q in AML are associated with thrombocytosis and atypical megakaryocytes in the bone marrow. The t(1;22) mutation in AML usually involves the megakaryoblastic lineage and occurs more commonly in infants. The latest WHO classification includes 2 provisional entities: AML with t(9;22); BCR-ABL1 (rare but may benefit from treatment with tyrosine kinase inhibitors) and AML with mutated RUNX1 (inherited leukemic predisposition, unfavorable prognosis). Myelodysplasia-related cytogenetic features in AML, such as monosomy 5 or del(5q), monosomy 7 or del(7q), or isochromosome 17q, are also associated with inferior outcomes. PREP Pearls Acute promyelocytic leukemia is characterized by t(15;17), which results in a PML-RARA fusion gene. Acute promyelocytic leukemia frequently presents with disseminated intravascular coagulation and has a high early mortality risk, but prompt institution of all-trans retinoic acid and supportive care for the coagulopathy can result in excellent survival. ABP Content Specifications(s)/Content Area Correlate clinical characteristics with chromosomal abnormalities in acute myeloid leukemia Understand the significance of rearrangements of the ATRA receptor gene in M3 acute myeloid leukemia Abla O, Ribeiro R. How I treat children and adolescents with acute promyelocytic leukaemia. Br J Hematol. 2014;164(1):24-38. doi: http://dx.doi.org/10.1111/bjh.12584 von Neuhoff C, Reinhardt D, Sander A, et al. Prognostic impact of specific chromosomal aberrations in a large group of pediatric patients with acute myeloid leukemia treated uniformly according to trial AML-BFM 98. J Clin Oncol. 2010;28(16):2682-2689. doi: http://dx.doi.org/10.1200/JCO.2009.25.6321 Wang M, Bailey N. Acute myeloid leukemia genetics: risk stratification and implications for therapy. Arch Pathol Lab Med. 2015;139(10):1215-1223. doi: http://dx.doi.org/10.5858/arpa.2015-0203-RA

A 7-year-old boy with severe aplastic anemia has received twice weekly platelet transfusions to maintain his platelet count at or above 10 × 103/µL (10 × 109/L) during immunosuppressive therapy. Yesterday, for a platelet count of 9 × 103/µL (9 × 109/L) he received a 10-mL/kg platelet transfusion from a donor with blood type A. The patient is blood type O. Today's platelet count is 9 × 103/µL (9 × 109/L). He has no fever or bleeding. He has some scattered petechiae on his face and chest. His spleen is not enlarged. Of the following, the MOST appropriate platelet product to be transfused is A.ABO-matched B.cross-matched C.HLA-compatible D.HLA-matched

Although a typical platelet transfusion dose of 10 mL/kg is expected to increase platelet counts by 30 [1] to 50 × 103/µL (30-50 × 109/L) for 2 to 3 days in pediatric patients affected by hypoproliferative thrombocytopenias, less robust responses are common. Varied definitions of platelet refractoriness exist, including some based on the corrected count increment (CCI). The following formula, where BSA indicates the body surface area, is used to calculate the CCI: CCI = ([posttransfusion platelet count - pretransfusion platelet count (/L)] × BSA (m2)/platelets transfused (1011) The number of platelets transfused can usually be obtained from the issuing blood bank or estimated; a random donor platelet unit contains about 0.7 × 1011 platelets, and a typical apheresis unit contains about 4 × 1011 platelets. One definition of refractoriness is a CCI less than 5 × 103/µL (50 x 109/L) on at least 2 occasions based on platelet counts drawn 10 to 60 minutes after transfusion using the freshest available ABO-matched platelets. Alternatively, a recent American Society of Clinical Oncology guideline suggests an expected platelet increment in children of at least 3,500/m2 of BSA for each transfused random donor platelet unit. Platelet count measurement within this time period immediately following transfusion may help differentiate the cause of refractoriness. An immune etiology is suggested by poor recovery immediately after a transfusion, whereas non-immune causes tend to demonstrate some initial, immediate rise in platelet count but poor subsequent platelet survival (ie, return to pretransfusion platelet count by the next day). The primary immune cause is alloimmunization, most often due to anti-HLA antibodies that arise after exposure to class I HLA expressed on donor platelets. Alloantibodies against human platelet antigens on the platelet surface are detected less frequently and thought to play a less clinically significant role. Non-immune causes occur more commonly than immune causes and include splenomegaly, sepsis, disseminated intravascular coagulation, bleeding, graft-vs-host disease, and sinusoidal obstruction syndrome. Splenomegaly can clinically mimic alloimmunization by causing a poor recovery immediately posttransfusion. If platelet alloimmunization is suspected, as in this vignette, first transfusing the freshest available (< 48 to 72 hours old), ABO-matched platelets is recommended. Donor platelets have a limited shelf-life of 5 days. Transfusing fresher platelets maximizes the duration and extent of the platelet count increase. Since platelets naturally express ABO blood group antigens on their surface, transfusing ABO-matched platelets limits potential interaction with circulating isohemagglutinins (anti-A and anti-B antibodies), which can accelerate platelet clearance. Transfusing ABO-matched platelets increases platelet recovery by about 20% compared with non-ABO-matched platelets. If the CCI measured 10 to 60 minutes after such transfusion with fresh, ABO-matched platelets remains low and other non-immune causes have been ruled out, platelet alloimmunization is likely, and efforts should be undertaken to obtain more antigenically compatible platelet units. Three primary approaches (cross-matched platelets, HLA-matched platelets, and HLA-compatible platelets) are currently available and usually coordinated with the transfusion medicine service: Cross-matched platelets can be identified after incubating patient serum with donor platelets to assess for compatibility. This option is most expeditious if appropriate expertise and resources are available at the local blood center. The patient can be HLA typed to identify a pool of HLA-matched donors, from whom HLA-matched platelets can be collected by apheresis. This approach is resource-intensive and time-consuming because of the need for HLA-typing and identification and recruitment of a limited donor pool. Anti-HLA-antibody testing can be performed on the patient serum to identify HLA-specific alloimmunity. Donors lacking the recipient's HLAs can provide HLA-compatible platelets for the patient. This strategy, also known as antibody specificity prediction, permits identification of a larger donor pool than the HLA-matching strategy and does not require patient HLA-typing. Platelet count should be promptly reassessed within an hour posttransfusion to ascertain the success of the chosen strategy. Even the use of cross-matched or HLA-matched platelets does not always result in improved transfusion response. Although other approaches to mitigate alloimmune-mediated platelet refractoriness, such as intravenous immune globulin, anti-Rh(D) globulin, splenectomy, and continuous infusion of platelets, have been reported, strong evidence to support their efficacy is lacking. Clinical studies have demonstrated that leukoreduction of platelet products reduces platelet alloimmunization risk by two-thirds or greater. Consequently, more than 85% all red blood cell and platelet products transfused in the United States are now leukoreduced. PREP Pearls Leukoreduction of donor platelets is currently the most effective strategy for minimizing a patient's risk of developing platelet alloimmunization. Using the freshest available ABO-matched platelets for transfusion is a key first step in managing a patient with suspected platelet refractoriness. Transfusion of more patient-specific platelet products, such as cross-matched platelets, HLA-matched platelets, or HLA-compatible platelets, may mitigate platelet refractoriness caused by alloimmunization.

A 15-year-old adolescent girl has finished 6 months of daily low-molecular-weight heparin therapy with complete resolution of an unprovoked occlusive left proximal femoral vein thrombus. Protein C, protein S, and antithrombin levels were in the normal range. Prothrombin G20210A and factor V Leiden mutations were not identified. At a follow-up visit 11 months after diagnosis she reports a 1-month history of tingling and swelling in the left leg that worsens after prolonged standing. The left calf is 3 cm larger than the right calf and has a capillary refill time of less than 2 seconds. Bipedal pulses are +2. Of the following, the clinical aspect that MOST LIKELY predisposed this patient to develop her current condition is A.choice of anticoagulant B.duration of treatment C.extent of initial vein occlusion D.female sex

CORRECT View Peer Results Average Correct: 74.06% Post-thrombotic syndrome (PTS) is a common complication of deep vein thrombosis (DVT). Risk factors at diagnosis for the development of PTS include a delay in anticoagulation treatment, a limb DVT, elevated serum D-dimer and plasma factor VIII levels, complete venous occlusion by thrombus, and number of segments of the vein occluded. Risk factors at completion of 3 to 6 months of anticoagulation therapy include lack of resolution of DVT. Duration of treatment, patient sex, and choice of anticoagulant have not been identified as risk factors for the development of PTS. Venous thromboembolism is becoming more commonly recognized in children, particularly in hospitalized pediatric patients. Children who experience a DVT may develop PTS as a result of venous outflow obstruction and venous valvular incompetence secondary to thrombosis-induced valve damage. Post-thrombotic syndrome occurs a minimum of 6 months after a DVT has resolved in approximately 25% of pediatric patients who have a DVT of an upper or lower extremity, although prevalence estimates range widely from 0% to 70%. Symptoms and signs of PTS include swelling, pain, skin discoloration, and venous stasis ulcers. Many cases of PTS are mild. Diagnosis is usually made by history and physical examination and aided by validated scores such as the Manco-Johnson instrument or modified Villalta instrument (Figure). Prevention of PTS relies on prompt recognition and treatment of the DVT, as delay in treatment has been associated with PTS development. There is evidence that catheter-directed thrombolysis in proximal leg DVT may decrease the incidence of PTS. Compression stockings at the time of DVT diagnosis do not appear to have an impact on PTS development. Once PTS is present, there is no curative therapy. Compression stockings may mitigate symptoms. Research is focused on identifying improved biomarkers for the development of PTS, which may lead to therapeutic strategies. Inflammatory markers, such as a positive dilute Russell viper venom test, elevated interleukin-6 level, and elevated C-reactive protein level, have been implicated as potential markers for disease risk. PREP Pearls Post-thrombotic syndrome occurs in approximately 25% of pediatric patients within a minimum of 6 months after resolution of a deep vein thrombosis of the extremities. Symptoms and signs of post-thrombotic syndrome include swelling, pain, skin discoloration, and venous stasis ulcers. Prevention of post-thrombotic syndrome relies on prompt recognition and treatment of deep vein thrombosis. Catheter-directed thrombolysis may decrease post-thrombotic syndrome in proximal leg deep vein thrombosis. Risk factors at deep venous thrombosis (DVT) diagnosis for the development of PTS include a delay in anticoagulation treatment, a limb DVT, elevated serum D-dimer and plasma factor VIII levels, complete venous occlusion by thrombus, and number of segments of the vein occluded. Risk factors at completion of 3 to 6 months of anticoagulation therapy include lack of resolution of DVT.

A 16-year-old boy with sickle cell anemia and a history of deep vein thrombosis is receiving long-term monthly transfusion therapy due to abnormal transcranial Doppler ultrasonography results noted on routine evaluation several years ago. He is being seen in the clinic before his monthly transfusion. He receives daily oral hydroxyurea, daily subcutaneous deferoxamine, and daily oral rivaroxaban. During the last week, he has noted a high-pitched ringing in his ears that is most bothersome when he is trying to sleep. Otherwise, he states he has been feeling well and denies any recent pain or illness. Of the following, the MOST likely cause of his new symptoms is A.deferoxamine B.hydroxyurea C.long-term transfusions D.rivaroxaban

CORRECT View Peer Results Average Correct: 84.38% Deferoxamine is used in patients with iron overload, usually because of frequent transfusions. Deferoxamine complexes with iron and makes it water soluble so that it can be eliminated. Deferoxamine is usually given intramuscularly or via slow subcutaneous infusion because rapid intravenous administration can cause hypotension and shock as well as urticaria. However, even when administered slowly, if patients are receiving it for a long time or at high doses, complications have been reported, particularly with vision and hearing. Patients may develop tinnitus, as described in the vignette, and high-frequency sensorineural hearing loss. They also can develop vision problems, ranging from impaired night or peripheral vision to vision loss and optic neuritis. Most visual and auditory adverse effects are reversible when use of the medication is stopped if they are noted early, so hearing and vision testing is recommended for patients treated with deferoxamine for prolonged periods. Other complications reported with deferoxamine include growth retardation, particularly in children with lower ferritin levels using high deferoxamine doses. Lowering the dose of deferoxamine can often improve growth rate. Deferoxamine has also been reported to potentially increase the risk of Yersinia enterocolitica and Yersinia pseudotuberculosis as well as mucormycosis infection. Finally, high doses of vitamin C given with deferoxamine has been associated with heart dysfunction, but vitamin C supplementation in vitamin C-deficient patients receiving deferoxamine increases deferoxamine's chelation effectiveness without adverse effects. Limiting vitamin C supplementation to the recommended daily dosing typically minimizes the risk of cardiac adverse effects, but if cardiac concerns are noted, vitamin C supplementation should be stopped. Adverse effects of hydroxyurea are primarily myelosuppression and gastrointestinal concerns, including nausea, vomiting, diarrhea, constipation, and anorexia. Tinnitus has not been reported with hydroxyurea use, although headaches, dizziness, and fatigue may occur. Rivaroxaban is generally well tolerated, and its main adverse effect is bleeding complications, including gastrointestinal bleeding and intracranial bleeding. Complications from long-term transfusions include transfusion-associated graft-vs-host disease and iron overload. Tinnitus is associated with iron deficiency but not typically with iron overload. PREP Pearls Prolonged use of high-dose deferoxamine has been associated with hearing and vision complications. If caught early, these adverse effects will typically resolve once deferoxamine use is stopped. Deferoxamine may cause decreased growth, especially when used at high doses. High-dose vitamin C supplementation when used with deferoxamine has been associated with cardiac dysfunction, which typically resolves when vitamin C supplementation is stopped. ABP Content Specifications(s)/Content Area Know the adverse side effects of deferoxamine (hearing loss, vision changes, growth retardation) Suggested Readings Derin S, Mehmet Azik F, Topal Y, et al. The incidence of ototoxicity in patie

A 17-year-old boy with severe aplastic anemia, who is 3 months postallogeneic hematopoietic cell transplant, is seen in the office with a concern of lymph node enlargement. He also reports unintentional 20-lb weight loss during the past month. He has painless, firm, nonmobile posterior cervical lymphadenopathy, with the largest node measuring approximately 2 cm in diameter. An excisional biopsy of the lymph node reveals atypical cells that are positive for CD19, CD20, and Epstein-Barr encoding region in situ hybridization. Of the following, the MOST likely risk factor for the condition affecting this patient is A.antithymocyte globulin in conditioning B.diagnosis of severe aplastic anemia C.use of human leukocyte antigen (HLA)-matched sibling donor D.use of peripheral blood graft source

Correct Answer: A The clinical and biopsy findings described in this vignette are consistent with a diagnosis of Epstein-Barr virus (EBV)-associated monomorphic B-cell type posttransplant lymphoproliferative disorder (PTLD). Posttransplant lymphoproliferative disorder encompasses a wide spectrum of diseases, which are usually associated with EBV, ranging from a benign illness such as infectious mononucleosis to a malignant and often fatal lymphoma. It is a rare complication of hematopoietic cell transplantation (HCT) with a cumulative incidence of 1% to 2% at 10 years after HCT. It most commonly occurs in the first year after HCT, with the highest incidence being within the first 3 months. Epstein-Barr virus, a member of the herpesvirus family, is a ubiquitous pathogen among the general population and infects more than 90% of individuals by the time they reach adulthood. Typically, EBV has a tendency to cause latent infection of the B-lymphocytes, and it can remain in a nonpathogenic status within memory B-cells, evading immune response for a prolonged period. In immunocompetent individuals, EBV-specific cytotoxic T-cells control its spread. However, individuals with impaired T-cell mediated immunity, specifically HCT recipients in the early post-HCT period, are at a high risk of unchecked growth of EBV-infected B-lymphocytes driven by the ability of the virus to induce cellular proliferation and inhibit programmed cell death. Given the role of T-cell reconstitution in controlling spread of EBV, the risk factors for developing PTLD among allogeneic HCT recipients includes the use of in vivo or ex vivo T-cell depletion of the graft before HCT. Anti-thymocyte globulin (ATG) is an equine- or rabbit-derived antibody against T-lymphocytes and is commonly used as a serotherapy for in vivo T-cell depletion. Additionally, it is often used for the management of severe aplastic anemia and posttransplant graft-vs-host disease (GVHD). Prior studies have consistently shown ATG to be a significant risk factor for EBV-PTLD; therefore, routine post-HCT longitudinal surveillance for EBV through quantitative polymerase chain reaction (PCR) testing is often instituted among HCT recipients exposed to ATG. It is important to note that the method of T-cell depletion plays a role in predicting the risk of PTLD in HCT recipients. While alemtuzumab, which is a monoclonal antibody against CD52, is often used as a serotherapy for in vivo T-cell depletion, it is not always associated with higher risk of PTLD because it removes both T- and B-lymphocytes. Other risk factors for PTLD among HCT recipients include underlying diagnosis of immunodeficiency, use of human leukocyte antigen (HLA)-mismatched donor, use of reduced intensity conditioning coupled with prolonged immune suppression, and evidence of acute or chronic graft-vs-host disease posttransplant requiring additional immunosuppression. Additionally, the use of umbilical cord blood instead of a peripheral blood graft source has also been associated with a higher risk of PTLD after allogeneic HCT due to delayed immune reconstitution. PREP Pearls Posttransplant lymphoproliferative disorder after allogeneic hematopoietic cell transplantation (HCT) is typically seen within the first year post-HCT and most commonly associated with Epstein-Barr virus reactivation. Risk factors for posttransplant lymphoproliferative disorder among hematopoietic cell transplantation recipients include underlying immunodeficiency, use of human leukocyte antigen-mismatched donor, in vivo or ex vivo T-cell depletion, reduced intensity conditioning, umbilical cord blood donor, and evidence of acute or chronic graft-vs-host disease after transplant. Routine longitudinal surveillance for Epstein-Barr virus (EBV) DNA-emia is recommended after allogeneic hematopoietic cell transplantation for individuals at a high risk for EBV-posttransplant lymphoproliferative disorder.

A fellow is developing a drug to treat a chronic viral infection. This drug does not treat the acute phase of infection but prevents the infection from becoming chronic. Of the following, the statement that MOST accurately describes the effect of the new drug on the population is that the disease incidence will A.decrease, and prevalence will increase B.decrease, and prevalence will stay the same C.increase, and prevalence will decrease D.stay the same, and prevalence will decrease

Disease incidence refers to how many new cases of a disease are diagnosed during a given period. In this vignette, the new drug does not prevent the disease, so the same number of people should continue to get the disease. Therefore, disease incidence will not change. Disease prevalence refers to how many people have a disease at any given point in time. This would include anyone who just got the disease (new cases) as well as anyone who already had the disease (ongoing or chronic cases). In this vignette, the drug cures patients with chronic infections. Therefore, this drug would decrease the prevalence of the disease. PREP Pearls Incidence of a disease is the number of new cases of a disease diagnosed in a given period (eg, 10 new cases per year). Prevalence of a disease is the number of people who have a disease at any time no matter when they contracted the disease (eg, 37 million people are living with HIV). ABP Content Specifications(s)/Content Area Distinguish disease incidence from disease prevalence Suggested Readings Centers for Disease Control and Prevention. Measures of risk. In: Principles of Epidemiology in Public Health Practice: An Introduction to Applied Epidemiology and Biostatistics. 3rd ed. Centers for Disease Control and Prevention; 2012. Accessed July 15, 2022. https://www.cdc.gov/csels/dsepd/ss1978/lesson3/section2.html#:~:text=Prevalence%20refers%20to%20proportion%20of,during%20a%20particular%20time%20period. National Institute of Mental Health. What is prevalence? Accessed July 15, 2022. https://www.nimh.nih.gov/health/statistics/what-is-prevalence#:~:text=Incidence%20is%20a%20measure%20of,they%20first%20developed%20the%20characteristic

An 11-year-old boy with B-precursor acute lymphoblastic leukemia in remission for 21 months is being treated with maintenance chemotherapy, including intrathecal methotrexate. He has a generalized tonic-clonic seizure lasting 5 minutes. Computed tomography of the head shows no evidence of intracranial hemorrhage, but magnetic resonance imaging of the brain shows patchy enhancement in the parietal lobes bilaterally. On lumbar puncture, the cerebrospinal fluid has 3 red blood cells and 47 white blood cells per microliter with blasts. The bone marrow aspiration is negative for leukemia by morphology and molecular testing. Of the following, the MOST appropriate treatment is to A.add cranial radiation to the current maintenance treatment regimen B.switch to an intensive relapse chemotherapy regimen with cranial radiation C.switch to a relapse chemotherapy regimen with allogeneic hematopoietic stem cell transplant D.switch from single- to triple-agent intrathecal chemotherapy in the current maintenance regimen

During treatment for acute lymphoblastic leukemia (ALL), seizures are an infrequent event that may occur due to leukemic infiltration of the brain or, more commonly as a complication of chemotherapy. Central nervous system (CNS) leukemia relapse may present with neurologic symptoms such as seizure or headache, or can be asymptomatic and discovered in the cerebrospinal fluid (CSF) taken during routine lumbar punctures for administering intrathecal chemotherapy. Treatment-related complications that may present with seizures include: methotrexate leukoencephalopathy, cerebral venous sinus thrombosis due to asparaginase, posterior reversible encephalopathy syndrome related to corticosteroids or other chemotherapy, and CNS infections in the setting of immunosuppression. A new seizure in a patient with ALL is an oncological emergency. Once the patient has been stabilized and the airway protected, imaging is needed. Computed tomography of the head can help to assess for potential intracranial hemorrhage. Magnetic resonance imaging of the head can provide detail of potential white and gray matter changes, vascular changes, inflammation, or lesions. Diagnostic lumbar puncture is needed to evaluate for infection or leukemia relapse. Since the CNS is a sanctuary site for leukemia, CNS-directed chemotherapy is mandatory in patients without CNS leukemia at diagnosis. Without CNS prophylaxis, more than half of ALL patients would experience relapse in the CNS. In the past, CNS prophylaxis included chemotherapy and cranial radiation. Cranial radiation is now given only as treatment to high-risk individuals with CNS leukemia at diagnosis (or after traumatic lumbar puncture) because of radiation-related potential late effects of neurocognitive impairment and second malignancy. Prophylactic CNS-directed chemotherapy with intrathecal chemotherapy and intravenous/oral agents that penetrate the blood-brain barrier has replaced prophylactic cranial radiation. For CNS prophylaxis in ALL, triple-agent intrathecal chemotherapy consisting of cytarabine, hydrocortisone, and methotrexate, has not demonstrated a benefit compared to single-agent intrathecal methotrexate. Despite CNS prophylaxis, leukemia relapse in the CNS occurs in up to 5% of patients, with a higher rate of CNS relapse seen in T-cell ALL patients as compared to B-lineage ALL patients. A leukemia relapse in the CNS can manifest as either a chloroma in the CNS, noted as an abnormality on detailed neurological examination and imaging, or as leukemic blasts in the CSF with no imaging abnormalities. The patient in this vignette has isolated leukemia relapse in the CNS. Isolated CNS relapse is defined as 5 or more white blood cells per microliter of CSF with blasts present or biopsy-proven leukemia recurrence in the CNS with no morphologic involvement in the bone marrow or other site. Prognosis and therapy for isolated CNS relapse depend on the duration of the first complete remission (CR1). Relapse that occurs greater than 18 months after CR1 is associated with an approximately 70% overall survival when treated with reinduction chemotherapy and cranial radiation. For the patient in this vignette with an isolated CNS relapse after 21 months of CR1, the preferred treatment is a switch to an intensive relapse chemotherapy regimen with cranial radiation. Some patients with CNS relapse have concurrent subclinical leukemia in the bone marrow detectable by molecular testing but not morphology. In addition, since isolated CNS relapse may be a precursor of eventual bone marrow relapse, intensive systemic chemotherapy is needed to achieve remission and cure. Thus, changing the intrathecal therapy from single-agent to triple agent chemotherapy or adding cranial radiation to the current maintenance treatment regimen would not be adequate therapy for isolated CNS relapse. In isolated CNS relapse, relapse chemotherapy regimens involve agents that achieve CNS penetration, such as intravenous, high-dose cytarabine and high-dose methotrexate, in addition to intrathecal chemotherapy. Relapse chemotherapy also includes epipodophyllotoxins and some alkylating agents that are not typically given in ALL therapy regimens upfront. This intensive combination of chemotherapy agents are given over 6 to 12 months prior to cranial radiation with 18 to 24 Gy. Giving the intensive blocks of chemotherapy prior to radiation allows for better tolerance of the chemotherapy, thereby avoiding the neurotoxicity observed when intensive CNS-directed therapy is given after cranial radiation. The approximate duration of relapse therapy is 2 years. Allogeneic hematopoietic stem cell transplant is not indicated in isolated CNS relapse greater than 18 months after CR1. Early isolated CNS relapse that occurs less than 18 months after CR1 is associated with an approximately 45% event-free survival. While some groups support the use of hematopoietic stem cell transplant after a second complete remission (CR2) in patient who have an early isolated CNS relapse, the optimal therapy is controversial and not standardized. PREP Pearls During treatment for acute lymphoblastic leukemia, seizures are infrequent events that can occur due to leukemic infiltration of the brain or, more commonly as a complication of chemotherapy such as methotrexate leukoencephalopathy. Acute lymphoblastic leukemia relapse isolated to the central nervous system occurring greater than or equal to 18 months after the first complete remission exhibits an approximately 70% overall survival rate when treated with an intensive, systemic, relapse chemotherapy regimen and cranial radiation. Acute lymphoblastic leukemia relapse isolated to the central nervous system occurring less than 18 months after the first complete remission exhibits an approximately 45% event-free survival rate and optimal treatment has not yet been determined.

The leader of a hospital-acquired venous thromboembolism prevention team is reviewing the literature regarding risk factors in order to develop best practices. A meta-analysis comparing different risk factors and risk-assessment models is identified (Figure 1). Of the following, the factor MOST WEAKLY associated with the risk of hospital-acquired venous thromboembolism from the data derived from case-control studies is A.intensive care unit admission B.mechanical ventilation C.presence of a central venous catheter D.prolonged hospitalization

Forest plots can be used in meta-analyses to visually represent data compared across studies, the relative study sizes, confidence intervals, and the overall conclusions. Forest plots compare the outcomes for studies and provide the combined statistical analysis as an overall measure. The confidence interval is represented by the length of the line for each study and the size of the study by the size of the diamond. The combined statistic differentially weights the input of the studies. In the forest plot in the vignette (Figure 1), intensive care unit admission showed the highest overall association with hospital acquired venous thromboembolism, with a combined odds ratio of 2.14 (95% CI, 1.97-2.32). However, this confidence interval crosses with the confidence interval for the presence of a central venous catheter (95% CI, 1.99-2.25). Prolonged hospitalization, while a significant risk factor in this meta-analysis, resulted in a combined odds ratio of only 1.03 (95% CI, 1.03-1.03). Thus, prolonged hospitalization is most weakly associated with the risk of hospital-acquired venous thromboembolism in this meta-analysis. A meta-analysis allows for the comparison of the outcomes of published studies through a quantitative, formal, systematic method. Meta-analyses are a subset of systematic reviews, however, they are superior in the hierarchy of clinical evidence due to the rigorous statistical analysis of the existing data (Figure 2). The characteristics of a systematic review include a clearly stated set of objectives with predefined eligibility criteria for studies, a systematic search that attempts to identify all eligible studies, a clear methodology, land an assessment of the risk of bias of the studies. Systematic reviews also have a synthesis of the findings of the studies used and may or may not also contain a meta-analysis. Meta-analyses assess the strength of evidence presented regarding a disease and treatment. They are designed to determine if an effect exists, and if so, whether the effect is positive or negative. They also examine the heterogeneity between studies, which can be informative. The most common measures of effect are the risk ratio (relative risk) and the odds ratio. It is important in a meta-analysis that all included studies meet strict inclusion criteria and that the quality of the studies is carefully appraised. Specific methods used in a meta-analysis are heterogeneity analysis, sensitivity analysis, and evaluation of publication bias. The I2 statistic is a measure of heterogeneity within a specific study and between studies, with a lower number indicating less heterogeneity. A funnel plot can aid in identifying publication bias, as journals tend to publish studies with positive findings over studies with negative results. An example of funnel plot can be found in Haidich AB. Often, authors of meta-analyses will attempt to include unpublished data, if available. Studies are weighted based upon the size of the study and the variance of the results. Both systematic reviews and meta-analyses should be reported using the PRISMA (Preferred Reporting Items for Systematic Reviews and Meta-Analyses) guidelines. A high-quality meta-analysis can provide a clearer understanding of the available data and generate hypotheses that can lead to new avenues of research. However, care must be taken to ensure the quality of the included studies. PREP Pearls A high-quality meta-analysis uses rigorous statistical methods according to PRISMA (Preferred Reporting Items for Systematic Reviews and Meta-Analyses) guidelines to systematically compare published studies on a topic. Forest plots can be used to visually represent the data compared across studies, the relative study sizes, confidence intervals, and overall conclusions.

A 9-year-old boy with severe hemophilia B has been on prophylactic factor therapy for 7 years consisting of twice-weekly doses of an intravenous, recombinant, short-acting factor concentrate. Eighteen months ago, he switched to weekly, intravenous, recombinant, long-acting factor concentrate (50 units/kg). He continues to have no episodes of spontaneous bleeding with this regimen. He tripped and fractured his left forearm during baseball practice today. An open reduction of this fracture is needed. His last dose of factor replacement was approximately 12 hours ago. Of the following, the BEST next step(s) in management is(are) to A.measure a factor IX activity level now and preoperatively give a dose of his long-acting concentrate to reach 100% activity B.measure a factor IX activity level now and preoperatively give a dose of a short-acting concentrate to reach 100% activity C.preoperatively give additional long-acting concentrate at a dose of 50 units/kg of body weight D.preoperatively give a short-acting concentrate at a dose of 100 units/kg of body weight

Hemophilia B is an X-linked, congenital deficiency of factor IX resulting from a variety of mutations. Prophylactic administration of factor IX concentrate reduces spontaneous joint bleeds, leading to an overall improved quality of life. Since prophylactic doses do not raise the clotting factor to the normal hemostatic range, additional factor replacement is needed for hemostasis prior to major surgery or following trauma associated with excessive bleeding as in this vignette. Factor IX concentrates are available as short-acting and long-acting products. Short-acting products are either plasma-derived or recombinant, providing a factor product with a similar half-life as endogenous factor IX. Long-acting products are recombinant with additional modifications (eg, fusion with albumin, Fc fusion, and PEGylation) to yield a longer half-life. Although evidence-based recommendations are limited, the 2013 World Federation of Hemophilia treatment guidelines provide an approach for adequate hemostasis in patients with hemophilia who require a major surgical procedure (ie, one that requires hemostatic support for more than 5 consecutive days). In this vignette, the patient requires major surgery and will need additional factor replacement to achieve hemostatic levels. In a health center that can obtain a rapid factor IX activity level, the appropriate dose of his current factor replacement can be calculated to raise his level to near 100% for good hemostasis while reducing the risk of hypercoagulability and cost. The best next step for the boy in this vignette is to measure his factor IX activity level now and give a dose of long-acting concentrate to reach 100% activity. Although long-acting factor IX concentrates are capable of achieving close to 100% factor IX activity for minor surgery with a single preoperative dose, in this vignette, major surgery will require multiple doses to achieve continuous hemostatic levels to promote adequate bone healing. The empiric administration of arbitrary doses of long-acting factor concentrate (50 units/kg of body weight) or short-acting concentrate (100 units/kg of body weight) after having received routine factor replacement 12 hours prior, may put the patient at risk for hypercoagulability and increased financial expense. Administration of a dose of short-acting concentrate to reach 100% factor IX activity may provide a safe hemostatic level of factor IX, but the use of 2 different product types simultaneously is not currently recommended. Hemophilia B may be amenable to gene therapy but is not available yet for pediatric patients. In adult patients treated with gene therapy in clinical trials, sustained factor IX activity levels of up to 50% of normal have been reported. Most of the trials thus far have used the adeno-associated virus or lentivirus vectors to introduce the corrective gene. The disadvantage of current gene therapy modalities is that both viral vectors are highly immunogenic. Approximately 40% of adults have demonstrated neutralizing antibodies to the adeno-associated virus rendering them unresponsive. In addition, the smaller size of a patient requires smaller doses of viral gene therapy to be given more frequently than in larger individuals. The virus transfects the hepatocyte cytoplasm with the gene. The transfected hepatocytes may not proliferate; or, if they do proliferate, they may not duplicate these episomes. Therefore, repeated infusions of the viral vector are needed to keep up with an individual's hepatic growth and may increase the risk of neutralizing antibody development. Another treatment currently in clinical trials for hemophilia B is coagulation inhibitor blockers. The antisense molecule against antithrombin (fitusiran) blocks the production of antithrombin. This causes the coagulation system to be severely disinhibited and recalibrate at a level consistent with the procoagulant pathway. As the ambient level of antithrombin produced by patients given fitusiran drops precipitously, factor VIII and factor IX can activate large amounts of Xa thereby increasing thrombin levels. Typically there is only a small amount of thrombin produced by the factor IX-deficient patient due to inhibition by the normal amount of antithrombin. Other agents that block tissue factor pathway inhibitor are in earlier stages of research. PREP Pearls Long-acting factor IX concentrates may provide adequate hemostasis for minor surgery with a single preoperative dose. Short-acting and long-acting factor IX concentrates have demonstrated similar hemostatic efficacy after trauma and as hemarthrosis prophylaxis.

A 7-year-old boy is scheduled to receive craniospinal irradiation and vincristine for the treatment of average-risk medulloblastoma. Of the following, the acute hematologic effect of radiation injury that is MOST likely to occur first is A.anemia B.lymphopenia C.neutropenia D.thrombocytopenia

Lymphocytes are the most radiosensitive cells in the peripheral blood. The lymphocyte count decreases dramatically within hours following large-volume marrow irradiation, defined as total body, craniospinal, whole abdomen, full thoracic, or pelvic irradiation. Local irradiation may also cause pronounced lymphopenia. The lymphopenia is likely caused by intermitotic or apoptotic cell death in circulating lymphocytes. After the initiation of large-volume marrow irradiation, the absolute neutrophil count typically decreases at 7 to 10 days followed by a drop in the platelet count within 2 to 3 weeks. The absolute neutrophil count and platelet count usually plateau at 35% to 50% of pretreatment levels and recover within 4 to 6 weeks of completing radiation therapy. Of the circulating peripheral blood cells, the red blood cells have the longest circulating life span and are most resistant to the damaging effects of irradiation. Anemia rarely occurs with local radiation. However, when large-volume marrow irradiation is given, the erythrocyte count declines over 2 to 3 weeks, reaching a nadir at approximately 3 weeks following irradiation. The red blood cell count drops the slowest and recovers the quickest after irradiation compared to other blood cells. Hematopoietic stem cells (HSCs) are one of the most sensitive tissues to radiation-induced damage. Radiation damage to HSCs is caused by the following mechanisms: increased production of reactive oxygen species and induction of oxidative stress; DNA damage; activation of apoptotic cell death; enhanced cell senescence; and promotion of HSC differentiation. Manifestations of acute radiation injury to HSCs include bleeding, infection, and bone marrow failure. The hematologic effects of irradiation and the ability of the bone marrow to recover are dependent on the dose of radiation and the volume of bone marrow within the irradiated field. The effects may be acute or chronic. The acute depletion of bone marrow components results from the direct effect of radiation on the HSC compartment, whereas chronic bone marrow aplasia is the result of endothelial damage leading to an impaired vascular supply with subsequent fibrosis in the bone marrow stroma. PREP Pearls Large-volume marrow irradiation, defined as total body, craniospinal, whole abdomen, full thoracic, or pelvic irradiation, produces lymphopenia within hours of exposure. After the initiation of large-volume marrow irradiation, the absolute neutrophil count typically decreases by 7 to 10 days followed by a drop in the platelet count within 2 to 3 weeks. Anemia rarely occurs after local irradiation. However, after large-volume marrow irradiation, the erythrocyte count declines over 2 to 3 weeks, reaching a nadir at approximately 3 weeks following irradiation

A full-term neonate was noted at 4 hours after birth to have a hematoma at the site of his intramuscular vitamin K shot. Apgar scores were 7 and 8 at 1 and 5 minutes, respectively. Jaundice was noted, and the total serum bilirubin level at 29 hours of age is 33.3 mg/dL (570 µmol/L). The rest of his physical examination findings are normal. Maternal blood type is O positive. Laboratory data are shown: Laboratory Test Result Hemoglobin 11.6 g/dL (116 g/L) Platelet count 26 × 103/µL (26 × 109/L) White blood cell count Normal Differential Normal Creatinine 1.3 mg/dL (115 µmol/L) Reticulocytes 4.3% Peripheral smear Thrombocytopenia, 7-8 schistocytes/HPF Prothrombin time Normal Activated partial thromboplastin time Normal Fibrinogen Normal Of the following, the test MOST likely to reveal the underlying disorder in this patient is A.ADAMTS-13 activity B.blood culture C.direct antiglobulin test D.renal ultrasonography

Microangiopathic hemolytic anemia, presenting with thrombocytopenia and schistocytes on the peripheral blood smear, can be caused by a number of underlying etiologies. Disseminated intravascular coagulation is a common cause, but when presenting in a neonate, congenital causes should also be considered. The patient in this vignette has normal coagulation parameters, making disseminated intravascular coagulation less likely. A disintegrin and metalloproteinase with thrombospondin type 1 motif, member 13 or ADAMTS13 level, if low, will suggest congenital thrombotic thrombocytopenic purpura (TTP). While the creatinine level is abnormally elevated, renal ultrasonography is unlikely to reveal the underlying cause of this patient's clinical condition. A direct antiglobulin test will reveal autoimmune or alloimmune hemolytic anemia, but in these disorders, spherocytes rather than schistocytes would be present on the peripheral smear, and these disorders are less likely to result in marked thrombocytopenia. The hallmarks of TTP are thrombocytopenia, microangiopathic hemolytic anemia, and end-organ damage, most notably in the brain, heart, and kidneys. However, the classic pentad of TTP including fever, neurological symptoms, and renal dysfunction is not required to make the diagnosis. Congenital TTP (previously known as Upshaw-Schulman syndrome) is a rare recessively inherited disorder caused by a deficiency of the von Willebrand factor-cleaving protease ADAMTS13. Von Willebrand factor is released from the endothelium as ultra-large multimers, which are then cleaved by ADAMTS13 into smaller sizes that do not initiate microvascular thrombosis. Thrombotic thrombocytopenic purpura is most commonly caused by an acquired antibody against ADAMTS13. However, a small portion of patients with TTP have homozygous or compound heterozygous mutations in ADAMTS13 that result in constitutional deficiency. ADAMTS13 is located on chromosome 9q34 and expressed in the liver. Mutations leading to congenital TTP have been identified throughout the gene. The prevalence of congenital TTP is unknown, although estimates range from 0.5 to 4 cases/million. Because some patients may not be recognized, the prevalence may be higher than anticipated. Patients may be misdiagnosed as having immune thrombocytopenia or Evans syndrome. Genetic testing for ADAMTS13 mutations is available for definitive diagnosis of congenital TTP. Patients with congenital TTP may present with symptoms at birth or later in life. There may be some relationship between the type of ADAMTS13 mutation and age of presentation. Hyperbilirubinemia, as seen in the vignette, has been reported in infants with congenital TTP. One-third to one-half of patients with congenital TTP will have their first acute TTP episode before age 5 years. Episodes may be triggered by events such as pregnancy or infection, which result in high circulating levels of von Willebrand factor. Neurological sequelae are common in congenital TTP, with up to one-third of untreated patients having ischemic events leading to behavioral disorders, sensorineural hearing loss, and seizures. Patients may have a relapsing/remitting course requiring regular infusions of plasma which is the treatment of choice for congenital TTP because it replaces ADAMTS13. Some patients who have allergic reactions to plasma have been treated successfully with a plasma-derived factor VIII product with a high concentration of ADAMTS13. The hereditary TTP registry, established in 2009, collects genetic information and long-term clinical data on patients with congenital TTP. PREP Pearls The hallmarks of thrombotic thrombocytopenic purpura are thrombocytopenia, microangiopathic hemolytic anemia, and end-organ damage, most notably in the brain, heart, and kidneys. Congenital thrombotic thrombocytopenic purpura (previously known as Upshaw-Schulman syndrome) is a rare recessively inherited disorder caused by a deficiency of the von Willebrand factor-cleaving protease ADAMTS13. Plasma infusion, which replaces ADAMTS13, is the primary treatment for congenital thrombotic thrombocytopenic purpura.

A 9-year-old boy has 6 café-au-lait macules that measure more than 5 mm in greatest diameter and freckles in the axillae and groin. Of the following, the malignancy for which he is at increased risk is A.acute lymphoblastic leukemia B.osteosarcoma C.rhabdomyosarcoma D.Wilms tumor

Neurofibromatosis type 1 (NF1) is an autosomal dominant genetic disorder with complete penetrance and variable expression that affects approximately in 1 in 4,000 individuals. It results from mutations in NF1, which is located on band 17q11.2 and encodes neurofibromin. These mutations cause decreased production of neurofibromin or loss of function, both of which result in the clinical features of NF1. Neurofibromin stimulates intrinsic GTPase activity in the ras p21 family. The Ras protein activates cell signaling pathways including mTOR, MAPK, and SCF/c-kit and is involved in the pathogenesis of benign and malignant tumors. Although genetic testing is available, the diagnosis is usually made based on clinical features. The National Institutes of Health criteria for the diagnosis of NF1 are met if 2 or more of the following criteria are found in a child: Six or more café-au-lait macules over 5 mm in greatest diameter in prepubertal individuals and over 15 mm in greatest diameter in postpubertal individuals Two or more neurofibromas of any type or one plexiform neurofibroma Freckling in the axillary or inguinal region Optic glioma Two or more Lisch nodules (iris hamartomas) A distinctive osseous lesion such as sphenoid dysplasia or thinning of long bone cortex with or without pseudarthrosis A first-degree relative with NF1 based on the above criteria Children with NF1 often develop benign neurofibromas. They are at increased risk (relative risk, 4.0) for developing malignant tumors that include optic pathway glioma, other central nervous system gliomas, malignant peripheral nerve sheath tumor, pheochromocytoma, rhabdomyosarcoma, gastrointestinal stromal tumors, and non-lymphoblastic leukemia. Optic pathway glioma is the most common malignancy and occurs in 15% of children with NF1. There is no increased risk in patients with neurofibromatosis for acute lymphoblastic leukemia, osteosarcoma, or Wilms tumor. PREP Pearls Typical features of neurofibromatosis type 1 include café-au-lait macules, axillary and/or inguinal freckling, Lisch nodules, neurofibromas, skeletal dysplasia, and intellectual disability. Children with neurofibromatosis type 1 are at increased risk for developing optic pathway glioma, other central nervous system gliomas, malignant peripheral nerve sheath tumor, pheochromocytoma, rhabdomyosarcoma, gastrointestinal stromal tumors, and non-lymphoblastic leukemia. Optic pathway glioma occurs in 15% of children with neurofibromatosis type 1.

A 5-year-old boy with high-risk neuroblastoma who received high-dose chemotherapy and autologous stem cell rescue infusion 10 days ago develops diffuse, watery, foul-smelling diarrhea. His highest absolute neutrophil count since autologous transplant was 200/µL (0.20 × 109/L). He is taking daily subcutaneous granulocyte colony-stimulating factor, prophylactic antibiotics (levofloxacin), and antifungal medications. He has had no sick contacts. Of the following, the MOST likely cause of his diarrhea is A.Clostridium difficile infection B.cytomegalovirus infection C.graft-vs-host disease D.rotavirus infection

The child in this vignette underwent high-dose chemotherapy followed by autologous hematopoietic stem cell transplant (HSCT) (autologous stem cell rescue). He is within 30 days after HSCT and his stem cells have not engrafted. During this period, bacterial and fungal infections are common due to mucositis and neutropenia. Infections can be prevented with prophylactic antibiotics and antifungal agents. However, prolonged antibiotic use decreases the heterogeneity of healthy intestinal microbial flora, leading to Clostridium difficile overgrowth and infectious gastroenteritis. Diarrhea within 30 days of autologous HSCT could also be an adverse effect of chemotherapy or a viral infection. However, C difficile infection is the most common cause of diarrhea, arising in up to 10% of immunocompromised patients within 1 week of antibiotic exposure. Cytomegalovirus (CMV) reactivation can arise in patients who were seropositive before HSCT due to the prolonged immunosuppression. After autologous HSCT, an individual is less immune suppressed compared to an individual undergoing an allogeneic HSCT, making CMV colitis in this vignette less likely. In addition, the typical onset of CMV reactivation is a median of 31 days (range, 21-54 days), unlike in this vignette. Typically, CMV titers are monitored weekly, and treatment is initiated at institution-specific titer thresholds depending on the time of onset and severity of symptoms. Acute graft-vs-host disease often manifests as watery diarrhea, skin rash, and/or elevation in serum liver function tests. However, given that stem cells in autologous HSCT are self in origin, graft-vs-host disease is not observed. Rotavirus gastroenteritis is typically acquired within the community from contact with infected individuals, as in day care settings. Although many pediatric patients undergoing HSCT may have previously received the rotavirus vaccine, their memory T cells and B cells are ablated after chemotherapy. Rotavirus vaccine is a live oral vaccine. Vaccinated individuals can shed rotavirus in their stool for approximately 1 week after vaccination, placing immunocompromised contacts at risk. Rotaviral infection presents as watery diarrhea. To differentiate rotavirus infection from other causes of infectious diarrhea, stool viral, bacterial, and C difficile cultures need to be obtained. PREP Pearls The risk of infection prior to engraftment after autologous hematopoietic stem cell transplant is high, with bacterial and fungal infections being the most common. Prophylactic antibiotics may help prevent bacterial infections after high-dose chemotherapy and autologous hematopoietic stem cell transplant, but they can lead to Clostridium difficile overgrowth resulting in infectious diarrhea. Graft-vs-host disease is a complication seen with allogeneic hematopoietic stem cell transplant but not autologous hematopoietic stem cell transplant.

A 5-year-old boy with aplastic anemia is scheduled to start his conditioning chemotherapy for a matched unrelated donor hematopoietic stem cell transplant. He has had more than 10 packed red blood cell transfusions in his lifetime. He has a serum ferritin level of 1,500 ng/mL (3,370 pmol/L). Hepatic T2* magnetic resonance imaging revealed a liver iron content of 12 mg/g dry weight. His serum liver transaminase levels are elevated, but his serum bilirubin level, prothrombin time, and activated partial thromboplastin time are normal. An antifungal prophylactic agent is prescribed to start on day -1 of hematopoietic stem cell transplant. Of the following, the BEST prophylactic antifungal agent for this patient is A.amphotericin B B.fluconazole C.micafungin D.voriconazole

Prolonged neutropenia resulting from severe aplastic anemia as well as the myelosuppressive conditioning regimen for allogeneic hematopoietic stem cell transplant (HSCT) increases this patient's risk of opportunistic fungal infection, particularly from yeast (eg, Candida species) or mold (eg, Aspergillus species, mucormycosis). Invasive infection with Aspergillus species has been associated with a mortality rate of up to 80%. The choice of anti-mold prophylactic therapy is dependent on this patient's limited hepatic metabolism caused by iron overload from packed red blood cell transfusions. Echinocandins (eg, micafungin, caspofungin) are effective against both yeast and mold and do not interfere with hepatic cytochrome P450 enzymes, unlike azoles. In a 2004 randomized clinical trial by van Burik et al involving over 1,000 patients, micafungin was more effective than fluconazole at preventing invasive fungal infection (80% vs 73.5%). A 2012 meta analysis by Ethier et al compared echinocandins to fluconazole prophylaxis and found that echinocandins had a decreased risk of mortality associated with invasive fungal infections in patients undergoing chemotherapy or allogeneic HSCT. Although micafungin, amphotericin, and voriconazole are equally effective at treating any invasive fungal infection due to yeast or mold, the clinical indications for prophylaxis differ. Because amphotericin B has the common adverse effects of fever, chills, and renal dysfunction, it is rarely used as a prophylactic agent. Voriconazole, a potent azole, inhibits cytochrome P450 enzyme activity, leading to elevation in immunosuppressive drug levels, as well as liver transaminitis. Voriconazole is rarely used as a prophylactic agent for allogeneic HSCT because of the potential worsening of immunosuppression and hepatotoxicity. Fluconazole is an azole indicated for antifungal prophylaxis when neutropenia is expected to be brief, as experienced after autologous HSCT or after allogeneic HSCT for patients who do not have hepatic impairment. Fluconazole also affects the cytochrome P450 system, interfering with drug metabolism and resulting in hepatotoxicity but to a lesser extent than voriconazole. Fluconazole is an effective prophylactic and treatment for yeast (eg, Candida species) but is less efficacious against molds (eg, Aspergillus species). PREP Pearls Prophylaxis with micafungin is indicated for prolonged neutropenia after allogeneic hematopoietic stem cell transplant because of the increased risk of invasive fungal infection from yeast (eg, Candida species) or mold (eg, Aspergillus species). Fluconazole is an azole indicated for antifungal prophylaxis when neutropenia is expected to be brief, as experienced after autologous HSCT or after allogeneic HSCT for patients who do not have hepatic impairment. Echinocandins (eg, micafungin, caspofungin) are effective against both yeast and mold and do not interfere with hepatic cytochrome P450 enzymes, unlike azoles.

A 21-year-old man with hypodiploid acute lymphoblastic leukemia is in remission. Because of the high risk of relapse with conventional chemotherapy alone, he is referred for hematopoietic stem cell transplantation. He does not have any siblings that are human leukocyte antigen (HLA) matches, but he does have a 10/10 HLA-matched unrelated donor in the bone marrow registry. Of the following, the MOST appropriate conditioning regimen for this patient is A.bleomycin, etoposide, adriamycin, melphalan B.busulfan, melphalan C.cyclophosphamide, total body irradiation D.fludarabine, melphalan, alemtuzumab

Selecting a conditioning or preparative regimen for allogeneic hematopoietic stem cell transplant (HSCT) is dependent on the extent of myeloablation or immunosuppression needed as part of curative treatment. The factors that help determine the intensity of conditioning regimen include: disease type, performance status/organ function, stem cell donor source, degree of HLA mismatch of the stem cell product, and a history of prior HSCT. A common pediatric myeloablative regimen includes full-dose busulfan and cyclophosphamide. Another common myeloablative regimen includes total body irradiation (TBI) at cumulative doses of 1,200 cGy or higher and cyclophosphamide. A myeloablative conditioning regimen results in pancytopenia with peripheral blood count recovery time ranging from 14 to 28 days depending on the stem cell source. Stem cells from peripheral blood apheresis engraft the quickest, followed by bone marrow, and then cord blood. The patient in this vignette has high-risk leukemia and will require myeloablation to minimize the risk of leukemic relapse. A randomized study (Bunin et al showed that patients receiving matched unrelated donor HSCTs had better event-free survival when using a TBI-based regimen as compared to a busulfan- or chemotherapy-based regimen without TBI. A reduced-intensity conditioning regimen results in a shorter period of time to stem cell engraftment compared to myeloablative therapy. Common pediatric reduced-intensity regimens prior to HSCT that are immunosuppressive include fludarabine, low-dose busulfan, or cyclophosphamide at lower doses than myeloablative regimens. Response Choices A and B are nonmyeloablative conditioning regimens commonly used prior to autologous HSCT as consolidation chemotherapy for patients with solid tumors. Busulfan and melphalan have been used to treat high-risk neuroblastoma after multiple rounds of induction chemotherapy and surgical resection of macroscopic tumor. A regimen of bleomycin, etoposide, adriamycin, and melphalan has been used to treat relapsed/refractory lymphoma. The combination of fludarabine, melphalan, and alemtuzumab has been used as a reduced-intensity regimen for HSCT to treat nonmalignant diseases such as sickle cell disease or thalassemia major. The combination of fludarabine and alemtuzumab makes this regimen particularly immune suppressive, while melphalan will provide some mild myeloablation. Alemtuzumab and a similar agent, thymoglobulin, are directed at reducing T cells to prevent graft-vs-host disease. PREP Pearls The hematopoietic stem cell transplant conditioning regimen needed to eradicate disease and minimize toxicity is dependent on the patient's disease, performance status/organ function, stem cell donor source, and degree of HLA mismatch of the stem cell product. Myeloablative conditioning regimens prior to allogeneic hematopoietic stem cell transplant for treatment of relapsed or high-risk acute lymphoblastic leukemia typically include cyclophosphamide with total body irradiation or with busulfan. Reduced-intensity or nonmyeloablative conditioning regimens in hematopoietic stem cell transplantation may include fludarabine as well as alemtuzumab or thymoglobulin to provide immune suppression rather than myeloablation.

A 9-year-old girl is being treated for medulloblastoma and spinal metastases. She is scheduled to start a 4-week course of craniospinal radiation next month. Due to the high-risk nature of her disease, possible chemotherapy options to give during the next month (concurrent with radiotherapy) are considered. Of the following, the agent most likely to enhance the effect of the radiation therapy is A.carboplatin B.cyclophosphamide C.doxorubicin D.vincristine

Radiation modifiers act selectively to sensitize tumors to radiation (radiation sensitizers) or protect normal tissues from radiation toxicity (radioprotectors). Radiation sensitizers enhance the ability of radiation to kill cancer cells but do not increase the negative effect of radiation on healthy surrounding tissue, thereby improving the therapeutic ratio for radiation. In this setting, radiation may have improved efficacy without increased toxicity. Several chemotherapy agents have been identified to be radiation sensitizers. A radiation sensitizer acts synergistically with radiation, such that the number of cancer cells killed by both therapies together is more than the sum of the cancer cells killed by each modality alone. Radiation causes DNA damage, and the majority of radiation sensitizers work by inducing additional independent DNA damage or inhibiting DNA repair. Many compounds are under study as potential radiation modifiers, but relatively few agents have been validated to be radiosensitizers, particularly in children. Agents that have shown a radiosensitizer effect include: Platinum agents (eg, carboplatin, cisplatin, oxaliplatin) Temozolomide 5-Fluorouracil and capecitabine Gemcitabine Paclitaxel Doxorubicin and other agents may cause radiation recall (sunburn-like rash in the radiation field) if given during or shortly after radiation therapy. Cyclophosphamide and vincristine are not known radiation modifiers. PREP Pearls Radiation sensitizers enhance the ability of radiation to kill cancer cells but do not increase the negative effect of radiation on healthy surrounding tissue, thereby improving the therapeutic ratio for radiation. Examples of radiation sensitizers include platinum agents, temozolomide, 5-fluorouracil, gemcitabine, and paclitaxel.

A 12-year-old Nigerian boy presents with bilateral, painless, bulky cervical lymphadenopathy, slowly increasing over the past year. He is growing well and has not had fevers, weight loss, or night sweats. He is otherwise asymptomatic. An excisional biopsy shows large histiocytes with emperipolesis (presence of intact hematopoietic cells within the cytoplasm of the histiocytes), that stain positive for S100 and fascin, and negative for CD1a and CD207. A diagnosis of Rosai-Dorfman disorder is made, and positron emission tomography-computed tomography does not show any additional sites of disease. Of the following, the BEST management strategy for this patient is A.combination chemotherapy B.glucocorticoids C.observation D.radiation therapy

Rosai-Dorfman disease (RDD), also called Rosai-Dorfman-Destombes disease or sinus histiocytosis with massive lymphadenopathy, is a rare histiocytic disorder affecting approximately 100 individuals in the United States each year. It is a clinically heterogeneous disease that usually presents in childhood or young adulthood. It is more commonly seen in males and in individuals of African ancestry. The signs and symptoms of RDD can vary significantly depending on the sites and extent of disease. Classically, RDD presents with bilateral and painless cervical lymphadenopathy that is often massive. Mediastinal, axillary, and inguinal lymphadenopathy may also be present, and the number of involved nodal groups appears to be prognostic. Extranodal disease is not uncommon, occurring in more than 40% of cases. Extranodal sites include the skin (10% of cases), central nervous system (< 5% of cases), nasal cavity/paranasal sinuses (11% of cases, more common in individuals of Asian descent), bone (5%-10% of cases), kidneys (4% of cases, associated with a 40% mortality), intrathoracic region (2% of cases), and gastrointestinal tract (< 1% of cases). Pathologically, RDD is characterized by sinus expansion of large histiocytes. Nodal disease may also have a large number of plasma cells with varying proportions of immunoglobulin (Ig)G4+/IgG+ plasma cells. Emperipolesis, the presence of an intact cell within another cell's cytoplasm, is classically found but can be focal and is not required for diagnosis. Extranodal disease often shows fewer histiocytes, more fibrosis, and less emperipolesis. The large histiocytes seen in RDD stain positive for S100 and fascin. CD68, CD163, and CD14 are variably expressed. The lack of expression of CD1a and CD207 can distinguish RDD from Langerhans cell histiocytosis. Histiocytosis with emperipolesis may occur as a reactive process, particularly in the setting of malignancy. Therefore, a diagnosis of RDD requires that at least 10% of a biopsy specimen demonstrates the histopathological RDD features explained above. The pathogenesis of RDD is not well-understood. It is sometimes associated with autoimmune or neoplastic disease. Approximately 10% of cases have been seen concurrently with autoimmune diseases such as systemic lupus erythematosus, idiopathic juvenile arthritis, and autoimmune hemolytic anemia. With regard to malignancy, RDD may occur at cancer diagnosis, during, or after completion of therapy. It has been described most frequently in association with lymphoma, but it has also been reported in association with myelodysplastic syndrome, hematopoietic stem cell transplant for acute leukemia, and cutaneous clear-cell sarcoma. Rosai-Dorfman disease may also be seen in the setting of other histiocytic disorders. Because of the wide clinical spectrum and the variable pathologic features of RDD, the diagnosis is challenging to make. The evaluation of patients with RDD includes imaging studies to evaluate sites of disease, as well as evaluation for other associated conditions such as autoimmune disease or malignancy. Because RDD lesions are fluorodeoxyglucose (FDG)-avid, positron emission tomography-computed tomography is sometimes used for staging. Symptoms of RDD can be slowly progressive or may wax and wane over a course of many years. However, RDD is generally associated with good outcomes. Nodal/cutaneous disease is often self-limited. Extranodal disease, particularly with kidney, liver, or lung involvement is associated with a less favorable prognosis. HIstorical clinical studies have reported mortality rates of 7% to 12%, usually resulting from disease complications or infections. Management strategies for RDD are tailored to the individual patient. There is no established standard of care because the clinical manifestations are variable, and published evidence is scarce. Observation is a reasonable strategy in many cases, as 20% to 50% of patients with nodal and/or cutaneous disease will have spontaneous remissions. Surgical excision has a role in single-site disease and in cutaneous disease, and debulking may be needed in some cases of large obstructive or compressive lesions. Glucocorticoids are often helpful in relieving symptoms associated with nodal disease, although the response is variable. Chemotherapy is used in refractory or relapsed cases with variable response. Vinca alkaloids, low-dose antimetabolites such as mercaptopurine and methotrexate, and nucleoside analogs such as cladribine and clofarabine, have shown efficacy. Radiotherapy is modestly effective and is primarily used for emergent management or palliation of refractory RDD. If systemic therapy is needed, the optimal duration of therapy is not known. A common approach is to plan 6 to 12 months of therapy, with periodic response assessment with imaging, followed by observation. The patient in the vignette has classical symptoms of nodal RDD affecting limited sites, and he does not have constitutional symptoms or signs of obstruction or compression. Since many patients with nodal disease will have self-limited disease and spontaneous remission, observation is the best next step. Chemotherapy, glucocorticoids, and radiotherapy are used primarily in refractory or relapsed cases or in patients who have significant morbidity related to their disease. PREP Pearls Rosai-Dorfman disease is a rare histiocytic disorder that classically presents with massive lymphadenopathy, although the clinical presentation can vary widely. Rosai-Dorfman disease can be associated with autoimmune or neoplastic disease. Rosai-Dorfman disease is self-limited in many cases and generally has a good prognosis.

A 3-year-old girl with acute lymphoblastic leukemia presents with severe mouth pain after receiving high-dose methotrexate 4 days ago. Her fluid intake over the last 24 hours is significantly decreased, and she cannot eat due to pain. She has multiple oral ulcers. Of the following, the BEST treatment for her pain is A.acetaminophen, 10 mg/kg, oral B.ibuprofen, 10 mg/kg, oral C.ketorolac, 0.5 mg/kg, intravenous D.morphine, 0.1 mg/kg, intravenous

The child in this vignette is having pain severe enough to limit oral fluid intake, indicating she is unlikely to swallow oral pain medications, such as oral acetaminophen, ibuprofen, or nonopioid analgesic/opioid combinations. Thus, oral analgesics are not an option. Additionally, since she is experiencing mucositis secondary to methotrexate, she may also have developed renal dysfunction from methotrexate toxicity or dehydration, making nonsteroidal anti-inflammatory medications such as ibuprofen and ketorolac less desirable due to their renal effects. For these reasons, intravenous morphine is the best choice of analgesic for this patient. Pain is not uncommon for children with cancer and may be secondary to the disease, treatments, procedures, or generalized distress (Table 1). Untreated pain can result in poor quality of life, increased anxiety with future procedures, and even posttraumatic stress disorders. Children with sickle cell disease experience repeated pain events secondary to vaso-occlusive events/tissue ischemia and may develop chronic pain caused by avascular necrosis of the femoral or humeral head. Pain management begins with proper pain assessment performed frequently to assess effectiveness of pain relief treatment so that the plan of care can be adjusted accordingly. Assessment includes noting behavioral changes (particularly in young nonverbal children), physiologic changes (heart rate, blood pressure), restrictions in physical and social activities, and self-reports of pain. The pain assessment tool needs to be age- and developmentally- appropriate. Common self-report pain scales include: Numeric rating scale: Severity is ranked on a scale of 0 (no pain) to 10 (worst possible pain). Valid for use with children 8 years of age and older. Visual analogue scale: Patient is asked to place a mark across a 10-cm line at a point that corresponds to level of pain intensity. Typically, one end is labelled "no pain" and the other end is labeled "severe pain." The score is obtained by measuring the distance between "no pain" and the patient's mark in millimeters. Faces pain scale: Multiple versions exist. Used mostly in young children. Children choose the face with the expression that most closely resembles how they feel. Comprehensive treatment of pain includes pharmacologic and nonpharmacologic management. When possible, pain medication should be administered orally. Intramuscular administrations are to be avoided. As in this vignette, parenteral medications are preferred to intramuscular medications when the oral route is not available. Nonopioid analgesics like acetaminophen and ibuprofen are useful treatments for mild to moderate pain in children (Table 2). Acetaminophen has analgesic and antipyretic properties but does not have anti-inflammatory effects like ibuprofen. Ibuprofen has limited use in children receiving active treatment for cancer due to inhibition of platelet aggregation, potential gastrointestinal mucosal damage, and renal toxicity. Opioid medications are used to treat moderate to severe pain. Dosing is based on the amount needed to be effective. Switching to opioids is recommended when analgesic control with other modalities and medications is inadequate or has significant adverse effects. Adverse effects of opioid medications include nausea, vomiting, pruritus, constipation, and respiratory depression. Patients with severe pain benefit from a scheduled opioid dosing regimen with breakthrough doses available as needed. This can also be achieved by using a patient-controlled, analgesia, intravenous pump where the patient (or parent/nurse in the case of young children) can request and receive a dose of opioid medication on demand. Patient-controlled analgesia can be used with or without continuous infusion of the opioid, depending on whether a steady dose of opioid is needed for pain control. Morphine is the most frequently used opioid in patient-controlled analgesia; hydromorphone and fentanyl are also commonly used. Topical and local anesthetics can be used to prevent and manage postoperative pain. Topical formulations are particularly helpful for procedures involving needles. PREP Pearls Pain is not uncommon for children with cancer and may be secondary to the disease, treatments, procedures, or generalized distress. Pain assessment includes noting behavioral changes, physiologic changes, and use of an age- and developmentally- appropriate self-reported pain scale. When possible, pain medication can be administered orally while intramuscular administrations are to be avoided.

A 10-year-old girl who was recently diagnosed with acute myeloid leukemia is starting induction therapy. Chemotherapy is being administered via a subcutaneous central venous port-a-cath (port) accessed with an appropriate-sized port needle 2 days ago. Immediately upon starting the chemotherapy infusion she has severe pain, burning, swelling, and erythema in the skin surrounding her port site. Of the following, the chemotherapeutic agent that is MOST likely to be responsible for these signs and symptoms is A.clofarabine B.cytarabine C.daunorubicin D.sorafenib

The child in this vignette is having symptoms consistent with extravasation, the escape of a medication or fluid from a blood vessel into the surrounding tissue. She has experienced extravasation of chemotherapy secondary to dislodgement of the port needle from the septum of the reservoir compartment (or portal) during infusion of daunorubicin. Anthracyclines (doxorubicin, idarubicin, daunorubicin, mitoxantrone) are intravenous, vesicant chemotherapy agents that can cause extravasation injury when inadvertently infused outside of an intact blood vessel. Therefore, it is recommended that anthracyclines be administered via a functioning central venous catheter (CVC). Clofarabine, cytarabine, and sorafenib are not considered vesicants. Extravasation of vesicant medications has the potential to cause severe tissue damage and lasting injury. The incidence of extravasation of vesicant chemotherapy is unknown. Risk factors can be patient- or infusion-related (Table 1). Risk of injury due to extravasation is increased when vesicant chemotherapy is administered through peripheral intravenous access compared to a CVC. Although the risk of extravasation decreases when vesicant medications are delivered via a CVC, it is not zero. Classification of vesicant chemotherapies based on damage potential is listed in Table 2. The damage potential is secondary to the length of the retention of the vesicant in the tissue affected by extravasated chemotherapy. The liposomal form of doxorubicin reduces vesicant damage compared to doxorubicin by decreasing the diffusion of doxorubicin into the surrounding tissue. Extravasation of vesicants presents with signs and symptoms including pain, swelling, erythema, skin induration and discoloration, blister formation, and tissue necrosis. The severity of extravasation injury can be graded per the latest Common Terminology Criteria for Adverse Events (CTCAE). Because of the potential severity of irreparable tissue damage, most institutions have vesicant-specific protocols for the immediate management of chemotherapy extravasation. Steps can be taken to prevent extravasation. Education and training of nurses and physicians, including knowledge about the vesicants' risks and the appropriate administration of each chemotherapy agent, is essential. Procedures for administering vesicants need to be standardized. Administration of vesicants are given through a CVC whenever possible, particularly for continuous intravenous infusion. If central venous access is not available, peripheral venous access can be used. Recommendations concerning peripheral venous administration include: avoidance of small/fragile veins, especially in the dorsum of the hand, antecubital fossa, and radial/ulnar aspects of the forearm; avoidance of multiple attempts in the same vein; and avoidance of placement in veins of limbs with impaired circulation. A peripheral venous catheter can be placed in the largest vein possible with the smallest adequate venous cannula size. A clear dressing is placed over the peripheral access site and inspected prior to the administration of all intravenous agents and particularly vesicants. A butterfly needle is not recommended for administration of vesicant chemotherapy due to the mobility of the device. Patients and families can also be educated on the signs and symptoms of extravasation and the importance of immediate reporting. PREP Pearls Extravasation into skin and subcutaneous tissue of intravenous vesicant chemotherapy can manifest with pain, swelling, erythema, skin induration and discoloration, blister formation, and tissue necrosis. Anthracyclines (doxorubicin, idarubicin, daunorubicin, mitoxantrone) are the most common intravenous chemotherapy vesicants that can cause extravasation injury. Because of the potential severity of irreparable tissue damage, most institutions have vesicant-specific protocols for the immediate management of extravasation.

A previously healthy 17-year-old adolescent boy who has had worsening morning headaches for the past several weeks has developed new-onset seizures. Brain magnetic resonance imaging shows a large, expansile, infiltrative lesion with characteristics suggestive of a high-grade glioma. A gross total resection is obtained, and the pathology is consistent with glioblastoma multiforme without methylguanine methyltransferase (MGMT) overexpression. Based on their own research, the patient and family wish to proceed with radiotherapy and concomitant, followed by adjuvant, temozolomide. Of the following, the study that is MOST important to serially monitor in this patient is A.complete blood cell count B.echocardiography C.pulmonary function testing D.serum creatinine level

Surgery and irradiation are the mainstays of therapy for pediatric high-grade gliomas such as glioblastoma multiforme (GBM) and anaplastic astrocytoma. Based on small but measurable increases in survival observed in randomized trials, temozolomide is approved by the Food and Drug Administration for use in adults (with newly diagnosed GBM and refractory anaplastic astrocytoma) concurrent with and as an adjuvant with radiation therapy. However, the utility of temozolomide in pediatric patients with high-grade gliomas is less well-defined, as a Children's Oncology Group study (ACNS0126) did not show clear benefit. Although some small pediatric studies suggest oncolytic activity in other relapsed and refractory solid tumors (generally in combination with other agents such as temsirolimus, irinotecan, and vincristine), temozolomide has limited current frontline pediatric use. However, temozolomide does offer a reasonable oral therapy option with 100% bioavailability based on limited demonstrations of antitumor activity in children and adolescents. In the case of this older adolescent with newly diagnosed GBM, proceeding with standard chemoradiotherapy that includes temozolomide, especially given the tumor's lack of methylguanine methyltransferase (MGMT) overexpression, is a reasonable treatment option. Temozolomide, an imidazotetrazine, is an orally administered, small molecule, lipophilic prodrug alkylating agent, able to cross the blood-brain barrier. It does not require hepatic metabolism for activation, but instead hydrolyzes spontaneously at physiologic pH to an active metabolite that subsequently dissociates to form a methyldiazonium cation. This reactive cation methylates purine bases, primarily at the N7 position of guanine but also at N3-adenine and O6-guanine. The O6-guanine adduct accounts for temozolomide cytotoxicity, through the generation of DNA strand breaks that occur when cellular mismatch repair enzymes attempt to excise this moiety, ultimately leading to apoptosis. Temozolomide appears to work in an additive manner to radiation therapy by enhancing radiation-induced DNA strand damage. Methylguanine methyltransferase is a naturally occurring enzyme that can demethylate guanine at the O6 position, thus reversing temozolomide cytotoxicity, and is a primary mechanism of drug resistance. The MGMT expression level is typically low in normal brain tissue, but varies by as much as 300-fold between different gliomas. Methylation of the MGMT promoter is a primary epigenetic determinant of enzyme expression. In adult studies, MGMT promoter methylation status from tumor tissue has correlated with response to alkylating agent chemotherapy as well as to survival (ie, methylated MGMT promoter signifies lower enzyme expression and activity, correlating with improved outcomes) and is a commonly assessed pathologic parameter. Deficiency or defect in the mismatch repair machinery within tumor cells is yet another mechanism for resistance to temozolomide, as the normal apoptotic response to methylated adducts is impaired. The dose-limiting toxicity of temozolomide is myelosuppression, which is noncumulative, but cytopenias may be significant and are commonly encountered. The frequency of grade 3/4 hematologic events include: lymphopenia 55%, thrombocytopenia 25%, neutropenia 20%, and leukopenia 11%. In combination with affected patients' other immune risk factors, which may include steroid use, radiation, and the cancer diagnosis itself, lymphopenia increases risk for Pneumocystis jirovecii infection and appropriate prophylaxis is indicated. Due to the myelosuppressive effects of trimethoprim/sulfamethoxazole, some experts recommend prophylaxis with inhaled or intravenous pentamidine during chemoradiotherapy involving temozolomide. Severe pancytopenia, aplastic anemia, myelodysplastic syndrome, and acute leukemia (both myeloblastic and lymphoblastic) have been described in association with temozolomide use, albeit rarely. Complete blood cell counts need to be obtained throughout the treatment course. Since fatal and severe hepatotoxicity has also been reported, regular liver function tests are recommended. Although hypersensitivity pneumonitis has been reported, it is a rare occurrence and regular follow-up of pulmonary function is not required. Routine echocardiography or renal function assessments are not necessary because temozolomide has no recognized cardiotoxic or nephrotoxic effects. Nausea, vomiting, and fatigue are very common adverse effects. Some experts recommend administering an antiemetic prior to each temozolomide dose. Temozolomide is teratogenic, and pregnancy testing is recommended in females of reproductive age. PREP Pearls Temozolomide is part of the standard of care for treating adults with high-grade gliomas, but its utility in treating children with such tumors has not been clearly demonstrated. Tumor cell methylguanine methyltransferase (MGMT) expression status, which can be assessed as methylation status of the MGMT promoter, is an important prognostic parameter and may predict response to alkylating agents like temozolomide. Since myelosuppression is the dose-limiting toxicity for temozolomide, peripheral blood cell counts are routinely followed while on therapy.

A 15-year-old adolescent girl has a 3-week history of fever, weight loss, easy bruising, and bone pain. She has bruises, petechiae, pallor, and splenomegaly. Laboratory data are shown: Laboratory Test Result White blood cell count 140,000/µL (140 × 109/L) Peripheral lymphoblasts 68% Hemoglobin 9.7 g/dL (97 g/L) Platelet count 35 × 103/µL (35 × 109/L) Bone marrow evaluation shows 82% B lymphoblasts and the presence of BCR-ABL1. Of the following, the finding MOST specific and supportive of a diagnosis of Philadelphia-positive acute lymphoblastic leukemia in this patient is A.bone marrow blast percentage greater than 20% B.peripheral blast percentage greater than 20% C.p190 BCR-ABL1 fusion protein D.p210 BCR-ABL1 fusion protein

The Philadelphia (Ph) chromosome results from translocation t(9;22)(q34;q11) of the proto-oncogene ABL1 on chromosome 9 to the breakpoint cluster region gene (BCR) on chromosome 22. Three hybrids of the BCR-ABL1 fusion gene have been identified, encoding protein isoforms p210, p190, and p230, all of which have increased tyrosine kinase activity. This aberrant activity leads to downstream effects that ultimately result in increased proliferation, inhibition of differentiation, and resistance to cell death leading to the development of leukemia. The Ph chromosome is seen in almost all cases of chronic myeloid leukemia (CML) and in 3% to 5% of children and 25% of adults with acute lymphoblastic leukemia (ALL, primarily of B-cell origin). The Ph chromosome is rarely seen in patients with acute myeloid leukemia or mixed-phenotype acute leukemia. The hallmark of CML is marked proliferation of mature or maturing granulocytes, including neutrophils, basophils, and eosinophils. Three clinical stages have been described: the chronic phase, accelerated phase, and blast crisis. Chronic myeloid leukemia in lymphoid blast crisis clinically resembles ALL. Both CML in blast crisis and ALL have a high percentage of blasts in the bone marrow. They also can have a high percentage of circulating blasts, as seen in the vignette. However, the p190 BCR-ABL1 fusion protein is commonly seen in Ph+ ALL but is very rare in CML. Chronic myeloid leukemia is almost always characterized by the p210 fusion protein. About 85% of patients with CML present in the chronic phase. Many of them (20%-50%) are asymptomatic at diagnosis with an abnormality discovered on routine blood tests. Symptomatic patients most commonly present with fatigue, weight loss, sweating, abdominal distension or discomfort, and bleeding. Frequent findings include splenomegaly, leukocytosis (median white blood cell count [WBC] of approximately 100,000/µL [100 × 109/L]), anemia, and thrombocytosis (15%-30% have platelet counts > 600 × 103/µL [600 × 109/L]). Extramedullary involvement is unusual unless blast crisis is present. The peripheral blood smear shows all of the WBCs in maturing neutrophil progression, ranging from myeloblasts to mature neutrophils. A classic finding is a "leukemic hiatus" or "myelocyte bulge," in which myelocytes are more common than the more mature metamyelocytes. Absolute basophilia is always present, and absolute eosinophilia can be seen in most cases. There are very few peripheral blasts in chronic-phase CML, usually less than 2%, and dysplasia is not usually seen. In the accelerated phase of CML, progressive impairment of neutrophil differentiation is seen, and higher leukocyte counts are usually present. Dysplasia may also be seen. A blast percentage of 10% to 19% in the peripheral blood or bone marrow indicates accelerated-phase disease. In blast crisis, there is uncontrolled proliferation of blasts, with a blast percentage of 20% or more. Blasts can be myeloid or lymphoid, with lymphoid blast crisis occurring in 30% of cases and predominantly of B-cell origin. If untreated, the disease may progress through all 3 phases, or it can go directly from the chronic phase to blast crisis. Progression to blast crisis likely occurs as a result of genomic instability, the development of additional cytogenetic and molecular abnormalities, and additional activation of downstream signaling pathways. The presentations of Ph+ ALL and Ph- ALL in children are similar, although high WBC count and central nervous system involvement are more common in Ph+ ALL. It can be difficult to distinguish Ph+ ALL from CML blast crisis if there is no history of preexisting suggestive hematologic abnormalities or a diagnosis of CML. However, it is important to try to distinguish the 2 entities because CML in lymphoid blast crisis would warrant allogeneic hematopoietic stem cell transplant (HSCT) as initial therapy. Frontline therapy for Ph+ ALL is chemotherapy and tyrosine kinase inhibition, with HSCT reserved for patients who have a poor response to frontline therapy or relapse. The location of the genomic breakpoints for BCR-ABL1 is variable, resulting in different protein isoforms. The main regions of breakpoint clustering on BCR occur as follows: Between exons 12 and 16, called the major breakpoint cluster region (M-BCR) At the first exon of BCR, called the minor breakpoint cluster region (m-BCR) Between exons 19 and 20, called the micro breakpoint region (µ-BCR) In CML, the genomic breakpoint almost always occurs in M-BCR, leading to a 210-kD fusion protein (p210 isoform). The breakpoint in Ph+ ALL can occur in the M-BCR region (10%-20% of pediatric cases) or more commonly the m-BCR region, the latter resulting in a smaller 190-kD fusion protein (p190 isoform). The p190 isoform seen in ALL is associated with strong transforming activity and a shorter latency than p210 in animal models. Although the p190 fusion protein has been seen in CML, it is very uncommon. Neutrophilic-chronic myeloid leukemia, representing less than 1% of CML cases, is characterized by a breakpoint in µ-BCR, which results in a 230-kD fusion protein. The presence of p210 alone is not enough to diagnose CML because some patients with Ph+ ALL have this isoform rather than the more common p190 isoform. However, the presence of p210 in the setting of an ALL presentation may prompt further investigation to try to ensure that CML is not the actual diagnosis. While there is not a definitive diagnostic test, the following scenarios can suggest CML: Discordance between cytogenetic results and flow cytometry results after the initiation of treatment (persistence of BCR-ABL1 without a proportionate leukemic clone evident on flow cytometry) The presence of a population of lymphoid blasts but also a second (likely smaller) population of neutrophils showing the various maturation stages described above, possibly including the "myelocyte bulge" or basophilia. If CML in blast crisis is present, BCR-ABL1 will be present in both the blasts and the neutrophil population. In Ph+ acute leukemia, the fusion gene would only be present in the blasts. PREP Pearls Chronic myeloid leukemia in lymphoid blast crisis clinically resembles Philadelphia chromosome-positive acute lymphoblastic leukemia and it is important to distinguish between the two because treatment is different. Chronic myeloid leukemia is almost always characterized by the p210 BCR-ABL1 fusion protein. The p190 BCR-ABL1 fusion protein is commonly seen in Philadelphia chromosome-positive acute lymphoblastic leukemia, although the p210 fusion protein is seen in a minority of patients.

A 4-year-old boy has leukocytosis, neutrophilia, and a complicated history including recurrent skin infections, severe periodontitis, and intellectual disability. He is admitted for gastrostomy tube placement because of poor weight gain. Laboratory data are shown: Laboratory Test Result White blood cell count 27,500/μL (27.5 × 109/L) Absolute neutrophil count 25,300/μL (25.3 × 109/L) Hemoglobin 11.2 g/dL (112 g/L) Platelet count 449 × 103/μL (449 × 109/L) Erythrocyte sedimentation rate Normal His mother states that he has always had an elevated white blood cell count and that he has a "funny blood type." Blood bank testing reveals the absence of A, B, and H antigens on the red blood cells. Of the following, the type of flow cytometry analysis MOST likely to confirm the suspected diagnosis is A.CD11/CD18 B.CD40 ligand C.sialyl Lewis X D.signaling lymphocyte activation molecule-associated protein

The boy in this vignette has leukocytosis, neutrophilia, recurrent skin infections, severe periodontitis, intellectual disability, and growth retardation, which is suggestive of leukocyte adhesion deficiency type II (LAD II). The Bombay phenotype (lack of expression of the A, B, and H antigens on red blood cells) is a distinguishing feature of LAD II, a functional neutrophil disorder caused by defective fucosylation of macromolecules leading to the absence of fucosylated glycoproteins. Hence glycans that incorporate fucose, such as sialyl Lewis X (SLeX [CD15a]) and the H antigen, are unable to be expressed on the surface of myeloid cells. This deficit leads to the inability of the phagocytes to bind selectins on the endothelial surface of blood vessels to mediate margination and rolling. Flow cytometry for SLeX is most likely to confirm the diagnosis of the boy in this vignette. Leukocyte adhesion deficiency type II should be differentiated from LAD I, which is caused by a defect in β2 integrins leading to a lack of expression of CD11/CD18, because the treatment options differ. While both conditions present with chronic neutrophilia in the range of 10,000 to 40,000 cells/μL (10-40 × 109/L) that worsens during periods of infection and inflammation, patients with LAD II generally have fewer and less severe infections because phagocytes retain the ability to adhere and transmigrate via β2 integrins. Unlike LAD I, LAD II does not generally present with omphalitis. Patients with LAD I also have increased risk of colitis. Moderate to severe periodontitis and impaired wound healing occur with both conditions, but intellectual disability and the Bombay blood phenotype are uniquely associated with LAD II. Patients with LAD I are generally managed with supportive care including antibiotics and optimal dental hygiene. Patients with a severe phenotype are also considered for hematopoietic stem cell transplant. Similarly, patients with LAD II are managed with antibiotics and optimization of dental hygiene, but in addition may try fucose supplementation. It is important to initiate fucose supplementation early, ideally prior to the onset of neurologic impairment. However, the evidence for fucose supplementation is limited by the rarity of LAD II, and the results have been variable. The interaction between CD40 ligand (CD40L), a surface glycoprotein (CD154) on activated T cells, and the CD40 surface molecule on B cells, monocytes, and macrophages starts immunoglobulin class switching (from IgM to IgG, IgA, and IgE) needed for immune competence. A CD40L deficiency or X-linked hyperimmunoglobulin-M1 is associated with a primary T-cell immunodeficiency manifesting with bacterial sino-respiratory infections (including Pneumocystis jiroveci infections), protozoan gastrointestinal infections, cytomegalovirus infections, and enteroviral meningoencephalitis. Mutations in SH2D1A that produces signaling lymphocyte activation molecule-associated protein results in X-linked lymphoproliferative syndrome, which is associated with a potentially lethal mononucleosis illness based on the inability to control Epstein-Barr virus proliferation. PREP Pearls Leukocyte adhesion deficiency type II is associated with neutrophilia, poor growth, mental impairment, and the Bombay phenotype (a lack of expression of the A, B, and H antigens on red blood cells). Fucose supplementation can be used as a supportive adjunct for patients with leukocyte adhesion deficiency type II. Significant periodontitis occurs with both types I and II leukocyte adhesion deficiency.

A 12-year-old boy developed a petechial rash and headache. Laboratory evaluation reveals a white blood cell count of 120,000/µL (120 × 109/L) with circulating blast cells, a reduced fibrinogen level, and prolonged prothrombin time and activated partial thromboplastin time. Cytogenetic evaluation of the blast cells reveals chromosome translocation t(15;17)(q22;q11-12). Of the following, the MOST likely leukemia diagnosis is A.megakaryocytic B.monocytic C.myelomonocytic D.promyelocytic

The child in this vignette has the clinical and cytogenetic findings of acute promyelocytic leukemia (French-American-British [FAB] subtype M3). The coagulopathy and translocation t(15,17) are characteristic while hyperleukocytosis is more common in the agranular/hypogranular variant of promyelocytic leukemia. Hyperleukocytosis is a marked elevation of leukemia cells in the peripheral blood greater than 100,000/µL (100 × 109/L). Hyperleukocytosis occurs in 10% to 20% of children with acute myeloid leukemia (AML) and is more common with myelomonocytic leukemia (FAB subtype M4), monocytic leukemia (FAB subtype M5), or acute promyelocytic leukemia (FAB subtype M3). Hyperleukocytosis may result in leukostasis, which is characterized by a marked leukemic blast cell count and symptoms of impaired tissue perfusion. The mechanism of damage that leads to decreased tissue perfusion is thought to be caused by increased blood viscosity due to leukemic blast cell aggregates. Leukemic blast cells also have a higher rate of oxygen consumption and deplete oxygen in obstructed tissues. The increased expression of cellular adhesion molecules on blast cells results in increased blood viscosity and leukostasis. Children with AML have higher rates of leukostasis likely due to the fact that myeloblasts are larger in size and have a higher expression of adhesion molecules compared to lymphoblasts. Leukemic blast cells secrete cytokines that cause damage to vascular endothelial cells leading to thrombosis and hemorrhage. The vasculature in the lungs and central nervous system seems especially prone to leukostasis. Children with hyperleukocytosis and clinical signs of leukostasis have increased early mortality caused by respiratory failure or intracranial hemorrhage. In addition to pulmonary and central nervous system complications, other clinical manifestations of hyperleukocytosis with leukostasis include acute renal failure, priapism, myocardial infarction, bowel infarction, and limb ischemia. Because the early mortality rate is 9% to 17%, early recognition and prompt treatment of leukostasis is critical. Children with newly diagnosed AML and hyperleukocytosis, with or without clinical signs of leukostasis, need to be promptly treated with cytoreductive chemotherapy along with prophylaxis or treatment for tumor lysis syndrome (TLS) and correction of coagulopathy. Blood products, especially packed red blood cells, are reserved for life-threatening emergencies only because they may increase blood viscosity and the complications from leukostasis. Hydroxyurea is an oral medication used for cytoreduction when a definitive diagnosis has not been established. Hydroxyurea is effective in reducing hyperleukocytosis associated with both myeloid and lymphoblastic acute leukemia. Leukocytapheresis is a temporary method of leukoreduction that removes peripheral blood leukemic blast cells and reinfusing plasma, red blood cells, and platelets. There are no randomized clinical trials evaluating leukocytapheresis in the treatment of hyperleukocytosis. A Children's Oncology Group (COG) study of children with AML and hyperleukocytosis suggested that leukocytapheresis did not reduce induction mortality. Historically, leukocytapheresis had been used to manage TLS associated with AML. This same COG study revealed that TLS is no longer an indication for using leukocytapheresis since the effectiveness of rasburicase for the treatment of hyperuricemia in TLS has been well established. Additionally, other retrospective studies have not shown a correlation between leukocytapheresis and decreased mortality. There are no evidence-based criteria for starting or stopping leukocytapheresis. It may be considered as a temporizing measure for children with hyperleukocytosis and clinical signs of leukostasis, but it should not delay the initiation of cytoreductive chemotherapy. Leukocytapheresis is contraindicated in children with acute promyelocytic leukemia (FAB subtype M3) because it may worsen the associated coagulopathy. PREP Pearls Hyperleukocytosis is a marked elevation of leukemia cells in the peripheral blood greater than 100,000/µL (100 × 109/L). Hyperleukocytosis may result in leukostasis, which is characterized by a marked leukemic blast cell count and symptoms of impaired tissue perfusion. Leukocytapheresis may be considered as a temporizing measure for children with hyperleukocytosis and clinical signs of leukostasis, but it should not delay the initiation of cytoreductive chemotherapy. Leukocytapheresis is contraindicated in children with acute promyelocytic leukemia (FAB subtype M3) because it may worsen the coagulopathy associated with this type of acute myeloid leukemia.

A 2-year-old boy with severe factor VIII deficiency is on a prophylaxis regimen with a recombinant factor VIII product, 35 units/kg given 3 times per week. He has had 2 spontaneous left ankle hemarthrosis over the past 3 months. Laboratory tests drawn 8 hours after his last prophylactic dose of recombinant factor VIII show factor VIII activity less than 1% and a factor VIII inhibitor level of 22.5 Bethesda units. Of the following, the BEST next step in management for this patient is to A.use recombinant activated factor VII for treatment of bleeding B.start immune tolerance induction with daily high-dose factor VIII C.substitute recombinant factor VIII with plasma-derived factor VIII for prophylaxis and treatment of bleeding D.use recombinant factor VIII at increased doses for prophylaxis and treatment of bleeding

The child in this vignette with hemophilia A (factor VIII deficiency) has developed an inhibitory antibody or inhibitor to factor VIII as measured by the Bethesda assay. Given his high inhibitor titer, he is unlikely to respond to increased doses of factor VIII replacement with either recombinant or plasma-derived factor for the treatment of bleeding. Since plasma-derived factor could also be neutralized by the inhibitory antibody, substituting plasma-derived factor for recombinant factor at the same dose would not lead to a sufficient increase in factor VIII activity. Using a bypass agent, such as recombinant activated factor VII, for treatment of bleeding can be an appropriate strategy to provide hemostasis, however this does not address the issue of the high inhibitor titer. Although it was previously thought that patients with lower inhibitor titers at the start of ITI had a higher likelihood of successful immune tolerization than patients with higher titers, there is newer evidence that having a titer ≥ 10 BU at the start of immune tolerization induction (ITI) start does not influence outcome when ITI is initiated within 1 month of inhibitor detection. Thus, immediate ITI should be the preferred option in patients with newly identified inhibitors. In patients with hemophilia A, the estimated incidence of inhibitor development is 20% to 33%, whereas in hemophilia B (factor IX deficiency), the incidence is only 1% to 6%. Inhibitor development is much more common in severe (< 1% factor VIII activity) and moderate (1%-5% factor VIII activity) hemophilia than in mild (> 5% factor VIII activity) disease. In patients with severe hemophilia A, inhibitor development tends to occur at a young age, with the risk greatest during the first 50 exposures to factor VIII and decreasing after 200 treatment days, though inhibitors may develop at any age. An inhibitor may be detected when a screening inhibitor assay is performed during an annual comprehensive clinic visit or when a patient does not respond to prophylaxis and/or treatment doses of factor replacement as expected. The inhibitor titer is measured using the Nijmegen-Bethesda assay. The original Bethesda assay is a factor neutralization assay, where 1 Bethesda unit (BU) is defined as the amount of inhibitor that results in 50% residual factor activity of a defined test mixture. The Nijmegen modification standardizes the pH and protein concentration of the test mixture, thereby decreasing artifactual deterioration and improving specificity. In patients with a low inhibitor titer (< 5 BU), high doses of factor VIII replacement can override the effect of the neutralizing antibodies for treatment of bleeding episodes. For patients with high inhibitor titers (> 5 BU), immune tolerance induction (ITI) is the standard of care to eradicate the inhibitor. Regimens for ITI in hemophilia A include daily or several days per week infusions of variable doses of factor VIII, administered for several weeks to months to stop production of the neutralizing antibodies. Bypass agents include activated prothrombin complex concentrates, recombinant activated factor VII, and emicizumab-kxwh. Activated prothrombin complex concentrates contain trace amounts of factor VIII. Continued exposure to trace amounts of factor VIII can cause an anamnestic response, leading to increased inhibitor titers. Elimination of any factor VIII product exposure results in a gradual decrease in inhibitor titer; this decrease may take years in patients with very high inhibitor titers (> 100 BU). Recombinant activated factor VII, which does not contain factor VIII, may be useful as prophylaxis and treatment, while preventing an anamnestic response. Emicizumab-kxwh, indicated for prophylaxis only, is a subcutaneously administered bispecific monoclonal antibody that binds to factor X and activated factor IX, thus mimicking the mechanism of action of factor VIII. PREP Pearls Factor inhibitors are measured in Bethesda units (BU), where 1 BU is the amount of inhibitor that results in 50% residual factor activity. In hemophilia A patients with an inhibitor, immediate immune tolerance induction (ITI) within 1 month of inhibitor detection should be the preferred option in patients with newly identified inhibitors, regardless of the inhibitor titer at the start of ITI. In hemophilia A patients with an inhibitor, continued exposure to factor VIII even in the trace amounts found in bypass agents can increase the inhibitor titer.

A 3-year-old boy has acute onset of petechiae and mild thrombocytopenia. Previous episodes of petechiae had been difficult to detect due to an eczematous rash. His mother reports a history of recurrent otitis media and pneumonia requiring antibiotics, although he has not received any medications in the past 4 months. He has a family history of an uncle who died of lymphoma. Laboratory Test Result White blood cell count 7,000/µL (7 × 109/L) Hemoglobin 11 g/dL (110 g/L) Platelet count 110 × 103/µL (110 × 109/L) Mean platelet volume 6 fL He has a normal prothrombin time, activated partial thromboplastin time, and von Willebrand disease panel. The platelet function assay reveals an abnormal (or prolonged) response to epinephrine and adenosine diphosphate. Platelet aggregation studies confirm decreased aggregation to adenosine diphosphate and epinephrine (normal primary wave only), a blunted response to collagen, and a normal response to ristocetin. Of the following, the MOST LIKELY platelet disorder in this patient is A.Bernard-Soulier syndrome B.dense granule deficiency C.Glanzmann thrombasthenia D.platelet-type von Willebrand disease

The clinical presentation of eczema, microthrombocytopenia, immunodeficiency, and family history of malignancy is indicative of Wiskott-Aldrich syndrome, which is caused by a mutation in WAS. This disease may also express platelet function abnormalities in the form of dense granule deficiency. In the absence of medications, vascular abnormalities, or platelet dysfunction, an individual with a platelet count greater than 100 × 103/µL (100 × 109/L) does not typically demonstrate petechiae. Therefore, further investigation as to the cause of petechiae is recommended. Dense granule deficiency demonstrates decreased platelet aggregation to adenosine diphosphate (ADP) and epinephrine (with a normal primary wave only, but no secondary wave), a blunted response to collagen, and a normal response to ristocetin (normal primary and secondary waves), as seen in the patient in this vignette. Individuals with Bernard-Soulier syndrome have macrothrombocytes, variable degrees of thrombocytopenia, and a bleeding tendency characterized by no platelet response with ristocetin yet normal responses to ADP, epinephrine, and collagen. Glanzmann thrombasthenia patients have a bleeding tendency with normal platelet size and platelet count, abnormal or decreased primary platelet aggregation responses to ADP, epinephrine, and collagen, but a normal response to ristocetin. Platelet-type or pseudo-von Willebrand disease arises from a mutation in the glycoprotein 1b receptor that causes increased binding with von WIllebrand factor. Patients with this condition typically have normal platelet size, intermittent thrombocytopenia, and increased platelet agglutination with ristocetin. Dense granule deficiency may be part of a platelet storage pool disorder (also known as δ storage pool disorder) or be co-inherited with diseases such as Wiskott-Aldrich syndrome, Hermansky-Pudlak syndrome, and Chediak-Higashi syndrome. Deficiency of dense granules may be acquired as a manifestation of systemic lupus erythematosus, myelodysplastic syndrome, myeloproliferative disorders, or acute leukemias. It may be caused by a single genetic defect with a variable inheritance pattern and penetrance. Dense granule deficiency is associated with mucocutaneous bleeding and excessive bleeding after surgery or injury. Platelet aggregation studies document the decreased secondary wave in response to ADP and epinephrine caused by decreased or absent stores of non-metabolic ADP, which is usually present in the dense granules. The diagnosis can be confirmed by electron microscopic measurement of the number of dense granules. A normal, unaffected platelet has between 3 and 8 dense granules. Gray platelet disorder is associated with decreased or absent α-granules, which results in a gray appearance on Wright-stained peripheral blood smears. Inheritance may be autosomal recessive, dominant, or X-linked. Clinically, it manifests with macrothrombocytopenia and mild to moderate mucocutaneous bleeding. In adulthood it may progress to splenomegaly and myelofibrosis. Platelet aggregation assays reveal nonspecific abnormalities. Diagnosis is confirmed by electron microscopic measurement of the number of absence of α-granules or flow cytometric analysis of α-granule release. Rarely, an individual may have a deficiency of both dense and α-granules. Management of excess bleeding due to platelet storage pool diseases involves the use of desmopressin for mild to moderate bleeding, antifibrinolytics for mucosal bleeding, and platelet transfusions prior to surgery. Affected individuals need to avoid aspirin and nonsteroidal anti-inflammatory drugs. Women with menorrhagia may benefit from oral contraceptives. PREP Pearls Platelet dense granule deficiency may be inherited as δ storage pool disorder, co-inherited with Wiskott-Aldrich syndrome, Chediak-Higashi syndrome, or Hermansky-Pudlak syndrome, or secondary to systemic lupus erythematosus, myelodysplastic syndrome, myeloproliferative disorders, or acute leukemias. Gray platelet disorder is an inherited decrease or absence of platelet α-granules, manifesting with macrothrombocytopenia and mild to moderate mucocutaneous bleeding, progressing to splenomegaly and myelofibrosis in adulthood. Management of excess bleeding due to platelet storage pool diseases includes desmopressin for mild to moderate bleeding, antifibrinolytics for mucosal bleeding, and platelet transfusions prior to surgery.

A previously healthy 3-year-old girl is seen for evaluation of pallor. She was treated with antibiotics for pharyngitis 4 weeks ago, which has resolved. She eats a healthy diet, and her physical examination findings are normal. Laboratory data are shown: Laboratory Test Result White blood cell count 9,700/µL (9.7 × 109/L) White blood cell count differential Normal Hemoglobin 9.7 g/dL (97 g/L) Mean corpuscular volume 96 fL Platelet count 536 × 103/µL (536 × 109/L) Red blood cell distribution width 16.7% Reticulocyte count 1.5% Peripheral blood smear Macrocytosis, no nucleated red blood cells, and normal-appearing neutrophils, lymphocytes, and platelets No significant levels of parvovirus B19 IgM and IgG are detectable. Iron, alanine aminotransaminase, aspartate aminotransferase, bilirubin, and lactate dehydrogenase levels are normal. Results of hemoglobin electrophoresis are shown: Laboratory Test Result Hemoglobin A1 90.1% Hemoglobin A2 2.1% Hemoglobin F 7.8% Of the following, the MOST helpful next test to diagnose this patient is A.bone marrow aspirate and biopsy B.erythrocyte adenosine deaminase activity level C.RPS and RPL mutations D.serum vitamin B12 and folate levels

The constellation of macrocytic anemia, reticulocytopenia, and an elevated fetal hemoglobin level is suggestive of Diamond-Blackfan anemia (DBA). Although the majority of children with DBA present in the first year of age, approximately 10% are diagnosed at older ages. Only 50% of patients with DBA will have congenital anomalies that include craniofacial anomalies (ie, cleft lip/palate), upper extremity (ie, thumb) anomalies, genitourinary (ie, kidney) anomalies, and cardiac abnormalities. The short stature of DBA patients is not considered part of the constellation of congenital anomalies but may be a secondary phenomena caused by chronic anemia, iron overload, or corticosteroid use. Elevated platelet counts have also been reported in patients with DBA. Elevated erythrocyte adenosine deaminase (eADA) levels are seen in 75% to 90% of patients with DBA. The macrocytosis is most likely due to the elevated fetal hemoglobin level. Vitamin B12 and folate deficiency are unlikely given that the patient in the vignette has no past medical or family history, has normal growth parameters, a regular diet, and normal segmented neutrophils. Hemolytic anemias may also present with macrocytosis due to reticulocytosis. However, the reticulocyte count for the child in the vignette is not elevated, and patients with hemolytic anemias typically have elevated lactate dehydrogenase levels and indirect hyperbilirubinemia, which are also not present in the child in this vignette. An international clinical care conference for DBA established diagnostic criteria in 2008. Patients with classic DBA meet the following 4 major diagnostic criteria: Age younger than 1 year Macrocytic anemia Reticulocytopenia Decrease in erythroid precursors on bone marrow evaluation Patients who do not meet all diagnostic criteria should have findings in the major and minor supporting criteria for DBA. Major supporting criteria include the following: Mutations in ribosomal proteins and other newer known mutations A positive family history Minor supporting criteria include the following: Elevated eADA Congenital anomalies of DBA Elevated fetal hemoglobin level Elevated erythropoietin level No evidence of other inherited bone marrow failure When a bone marrow evaluation is performed, the morphology typically reveals normal cellularity with a significant decrease in erythroid precursors. Mutations in the genes, RPS and RPL, encoding the small or large subunits in ribosomal proteins are seen in about two-thirds of patients diagnosed with DBA. GATA1 and EPO mutations have also been identified in some patients with DBA. However, there is a 20% to 30% chance that no genetic mutation may be isolated. A bone marrow evaluation with a paucity of red blood cell precursors, an elevated eADA level, or positive genetic testing for ribosomal protein mutations would all support the diagnosis of DBA. Checking an eADA level would satisfy the criteria for a diagnosis of DBA in this patient as she would have 3 diagnostic criteria and 3 minor criteria. If the eADA was not elevated, the next step would be a bone marrow evaluation. Genetic testing takes several weeks to confirm a DBA diagnosis, and the results are positive in only 70% to 75% of patients with clinical disease. Therefore, ordering an eADA level is the most appropriate next step. Long-term follow-up of patients with DBA has demonstrated an increased risk of malignancies. Analysis of the Diamond-Blackfan Anemia Registry shows an estimated cumulative risk of 13.7% of developing cancer by 45 years of age, with a relative risk of 4.8 times compared to the general population. Patients with DBA are at increased risk of hematologic malignancies, such as myelodysplastic syndrome and acute myelogenous leukemia, similar to other inherited bone marrow failure syndromes. Patients with DBA are also at increased risk of solid tumors, particularly colon carcinoma and osteosarcoma. The risk appears to increase by 40 years of age for acute myelogenous leukemia/myelodysplastic syndrome and by 30 years of age for solid tumors. The risk of malignancy does not appear to be associated with any specific treatment. However, patients with DBA who receive a hematopoietic stem cell transplant appear to have an even higher risk of malignancy. The risk of malignancy is not associated with a particular DBA genotype. Somatic mutations in ribosomal proteins and haploinsufficiency in ribosomal proteins have been demonstrated in a variety of malignancies. Animal models have shown that ribosomal proteins act as tumor suppressor genes, suggesting that mutations may increase the risk of malignancy. Given the recent illness for the child in the vignette, transient erythroblastopenia of childhood (TEC) is a possible diagnosis. The age of the patient, recent possible viral illness causing pharyngitis, and the reticulocytopenia are part of the clinical manifestations of TEC. However, thrombocytosis, macrocytosis, and an elevated fetal hemoglobin level are not part of the constellation of symptoms for TEC, so another etiology for anemia should be suspected. Patients with TEC have normal eADA levels. PREP Pearls Diamond-Blackfan anemia is an inherited bone marrow failure syndrome that causes a decrease in erythropoiesis. Macrocytosis, elevated hemoglobin F level, and elevated erythrocyte adenosine deaminase activity level are seen in patients with Diamond-Blackfan anemia. Diamond-Blackfan anemia is a cancer predisposition syndrome.

A 3-year-old girl has newly diagnosed B-precursor acute lymphoblastic leukemia. Her 28-year-old mother is currently being treated for breast cancer, a maternal aunt had osteosarcoma at age 15 years, and her maternal grandmother died from adrenocortical carcinoma. Of the following, the MOST likely genetic characteristic of her leukemia is A.hyperdiploid B.low hypodiploid C.presence of the BCR/ABL fusion gene D.trisomy 4

The family in this vignette has a history concerning for a cancer predisposition syndrome. Given the cancers present in the family (B-precursor acute lymphoblastic leukemia [ALL], osteosarcoma, breast cancer, and adrenocortical carcinoma at relatively young ages), the most likely cancer predisposition syndrome is Li-Fraumeni syndrome. Leukemia was initially identified as an associated cancer in Li-Fraumeni syndrome, but subsequent studies suggested that the rate of leukemia was not higher than the general population. However, more recent data has shown that persons with Li-Fraumeni syndrome are at a higher risk of developing low hypodiploid ALL (30 to 39 chromosomes) compared to the general population. Favorable ALL genetic features, such as hyperdiploidy or trisomy 4, are not associated with Li-Fraumeni syndrome or other cancer predisposition syndromes, nor is the BCR/ABL fusion gene. In families with multiple members with cancers at young ages, a familial cancer predisposition syndrome needs to be considered. Li-Fraumeni syndrome is an autosomal dominant cancer predisposition syndrome, characterized by early development of sarcomas (eg, rhabdomyosarcoma), adrenocortical carcinoma, choroid plexus carcinoma, glioblastoma multiforme, and breast cancer as well as other tumors. The same individual may develop multiple primary tumors. The criteria for diagnosis and determining when testing should be undertaken for Li-Fraumeni syndrome has been described by Ruijs et al. Li-Fraumeni syndrome is most commonly caused by a germline TP53 mutation, which may be inherited or arise de novo. The p53 protein acts as a tumor suppressor through control of the cell cycle and DNA repair. Other activities regulated by p53 include glycolysis and mitochondrial respiration. Interestingly, individuals and mice with TP53 mutations show increased mitochondrial oxidative metabolism and improved exercise endurance compared to those with wild-type p53. This difference in fatty acid metabolism may also contribute to tumorigenesis because of increased oxidative damage. A variety of TP53 mutations, ranging from point mutations to complete gene deletions, have been identified. The type of mutation affects the clinical presentation of the syndrome. Missense mutations not only inactivate p53, but also result in enhanced oncogenesis, leading to earlier cancer onset in those families. Additional genetic variability also influences age at tumor development. For example, a polymorphism in the main negative regulator of p53, MDM2, also results in cancer development at a younger age in individuals with Li-Fraumeni syndrome. As the ability to detect mutations has improved, the relationship of p53 to tumorigenesis has been broadened to include other forms of cancer. For example, Wilms tumor has also been found to be strongly associated with TP53 germline mutations. Some data have suggested that surveillance for cancer in families with Li-Fraumeni syndrome with regular full body scanning may decrease mortality through early detection and treatment of cancer once it develops. If oxidative damage is shown to be linked to tumorigenesis, treatments that target mitochondrial activity may become options for cancer prevention in affected individuals. PREP Pearls Li-Fraumeni syndrome is a rare, autosomal dominant cancer predisposition syndrome. Cancers associated with Li-Fraumeni syndrome include adrenocortical carcinoma, breast cancer, brain tumors, hypodiploid acute lymphoblastic leukemia, soft tissue sarcomas, and osteosarcomas. Most individuals with Li-Fraumeni syndrome have a germline mutation in TP53, a tumor suppressor gene

A 6-year-old girl is evaluated for concerns of new onset pallor, fatigue, and "yellow eyes". Her parents report that she had a cold 10 days ago, but has been otherwise well. On examination, she has scleral icterus, tachycardia, a systolic ejection murmur, and splenomegaly. Laboratory Test. Result White blood cell count. 23,000/µL (23 ×109/L) Hemoglobin. 5.4 g/dL (54 g/L) Mean corpuscular volume. 104 fL Platelet count. 460 × 103/µL (460 × 109/L) Absolute reticulocyte count. 220 × 103/µL (220 × 109/L) Indirect bilirubin. 0.25 mg/dL (4.2 µmol/L) Total bilirubin 0.29 mg/dL (5 µmol/L) Direct antiglobulin test Polyspecific. 4+ IgG. 4+ C3 2+ Of the following, the MOST likely diagnosis is A.cold agglutinin disease B.hereditary spherocytosis C.paroxysmal nocturnal hemoglobinuria D.warm autoimmune hemolytic anemia

The girl in this vignette has evidence of hemolytic anemia (reticulocytosis, indirect or unconjugated hyperbilirubinemia, and anemia). The result of a positive direct antiglobulin test (DAT, also known as direct Coombs) is consistent with immune-mediated hemolytic anemia, eliminating hereditary spherocytosis and paroxysmal nocturnal hemoglobinuria from the differential diagnosis list. In this case, the DAT is positive for IgG which is most consistent with warm autoimmune hemolytic anemia. Warm autoimmune hemolytic anemia is mediated by polygenic IgG autoantibodies, and when hemolysis is brisk, complement (C3) autoantibodies can accompany IgG antibodies (as shown in the vignette). The diagnosis of cold autoimmune hemolytic anemias, such as cold agglutinin syndrome or paroxysmal cold hemoglobinuria, is aided by a positive DAT for C3 but negative for IgG, unlike in this vignette. The DAT is used to detect anti-IgG antibodies or anti-C3 antibodies bound directly on red blood cells (RBCs). Three different reagents are used: polyspecific (pool of IgG and C3), monospecific IgG, and monospecific C3. In general, the polyspecific reagent is used to screen for bound IgG and/or C3. If positive, the monospecific reagents are used. In this test, the antiglobulin reagents are added directly to washed red blood cells (to remove excess unbound IgG/C3 in the serum) and visually observed for agglutination. Agglutination is graded from 0 (none) to 4+ (solid button). The DAT is used to assess for possible hemolytic transfusion reactions (particularly delayed hemolytic transfusion reactions), hemolytic disease of the newborn, and autoimmune hemolytic anemia. The indirect antiglobulin test (IAT) detects antibodies circulating in plasma. In this test, serum is exposed to red blood cells with known antigens and is observed for agglutination. The IAT is the antibody screen performed as part of the type and screen procedure to identify any unexpected RBC antibodies. If antibodies are detected, it is important to identify the specific antibody and determine its clinical significance. Many alloantibodies are not clinically significant; they do not cause in vivo destruction of RBCs. If the patient has an alloantibody on screening or a history of alloantibody, an IAT is used again to cross-match blood, ensuring that patient serum and donated RBCs are compatible. PREP Pearls The direct antiglobulin test assesses for immune-mediated hemolysis resulting from delayed hemolytic transfusion reaction, hemolytic disease of the newborn, and autoimmune hemolytic anemia. The indirect antiglobulin test, which is the antibody screen portion of the type and screen, is used to detect any unexpected red blood cell antibodies. A direct antiglobulin test positive for IgG alone or IgG in combination with C3 is most consistent with warm autoimmune hemolytic anemia. A direct antiglobulin test positive for C3 but negative for IgG is most consistent with a cold autoimmune hemolytic anemia such as cold agglutinin syndrome or paroxysmal cold hemoglobinuria. ABP Content Specifications(s)/Content Area Be able to interpret the results of direct and indirect antiglobulin tests Suggested Readings Parker V, Torney CA. The direct antiglobulin test: indications, interpretation, and pitfalls. Arch Pathol Lab Med. 2017;141(2):305-310. doi: http://dx.doi.org/10.5858/arpa.2015-0444-RS Reid ME, Lomas-Francis C. Erythrocyte antigens and antibodies. In: Kaushansky K, Lichtman MA, Prchal JT, et al, eds. Williams Hematology. 9th ed. New York, NY: McGraw-Hill Education; 2016:2329-2352.

A 5-month-old male infant has a history of prolonged bleeding after circumcision, bleeding from the umbilical stump, and easy bruising. Laboratory data are shown: Laboratory Test Result Platelet count 250 × 103/µL (250 × 109/L) Prothrombin time > 100 s Activated partial thromboplastin time > 100 s Thrombin time > 100 s Mixing study (1:1 mixture) Complete correction Reptilase time Prolonged Of the following, the MOST likely diagnosis for this patient is A.afibrinogenemia B.heparin contamination in the sample C.prothrombin deficiency D.type 3 von Willebrand disease

The infant in this vignette has the clinical and laboratory features of afibrinogenemia. Prothrombin deficiency may cause prolongation of the prothrombin time and activated partial thromboplastin time but should not prolong the thrombin time (TT). Type 3 von Willebrand disease does not prolong the prothrombin time. The TT reflects the conversion of fibrinogen to fibrin and is prolonged by fibrinogen deficiency or functionally defective fibrinogen. The TT is prolonged when functional fibrinogen levels are below 100 mg/dL (2.9 µmol/L). The TT is also prolonged by the presence of heparin and fibrin degradation products in the plasma. In blood samples containing heparin, a substance derived from snake venom (reptilase) is used instead of thrombin in the TT test. The snake venom substance has a similar action to thrombin but unlike thrombin it is not inhibited by heparin. The prolonged reptilase time rules out heparin contamination in the sample. Fibrinogen (factor I) is synthesized in the liver. The normal circulating plasma level of fibrinogen is 200 to 400 mg/dL (5.9-11.8 µmol/L) with a half-life of 3 to 4 days. Fibrinogen is converted to fibrin during coagulation. Fibrinogen is an acute phase reactant and levels may increase 20 fold in response to inflammation. Bleeding does not usually occur unless the fibrinogen level falls below 100 mg/dL (2.9 µmol/L). Disorders of fibrinogen are rare and may be inherited or acquired. Fibrinogen defects may be quantitative, qualitative, or both. Afibrinogenemia is inherited in an autosomal recessive pattern and results in no circulating plasma fibrinogen. Patients with afibrinogenemia have severe bleeding following trauma. Bleeding from the umbilical stump is common. Obstetrical complications occur in women with afibrinogenemia. Other bleeding symptoms include epistaxis, heavy menstrual bleeding and bruising. Intracranial hemorrhage has been reported rarely. Hypofibrinogenemia is defined as a fibrinogen level less than 150 mg/dL (4.4 µmol/L). Hypofibrinogenemia may be inherited or acquired. The inheritance pattern is autosomal dominant, and it is often seen in carrier mutations causing afibrinogenemia. Liver disease severe enough to compromise liver synthetic function is the most common cause of acquired hypofibrinogenemia. Hypofibrinogenemia may also result from disseminated intravascular coagulopathy, hemophagocytic lymphohistiocytosis, and medications that interfere with liver synthetic function such as antifibrinolytic agents, asparaginase, and valproic acid. Many individuals with hypofibrinogenemia are asymptomatic while others exhibit the phenotype of afibrinogenemia. Dysfibrinogenemia is a term that indicates a normal circulating level of functionally abnormal fibrinogen. Dysfibrinogenemia may be congenital or acquired. Transmission of most forms of dysfibrinogenemia is autosomal dominant, caused by heterozygosity for a missense mutation. Liver disease is the most common cause of acquired dysfibrinogenemia. Other causes include disseminated intravascular coagulopathy, hemophagocytic lymphohistiocytosis, a paraneoplastic syndrome associated with renal carcinoma, and the use of isotretinoin. Children with dysfibrinogenemia can have bleeding, thrombosis, or both, or may be asymptomatic. Hypodysfibrinogenemia indicates a decreased plasma level of dysfunctional fibrinogen. Hypodysfibrinogenemia is inherited or acquired similar to dysfibrinogenemia. Symptoms include bleeding, thrombosis, or both. Autoantibodies that inhibit specific functions of fibrinogen occur in conditions such as systemic lupus erythematosus, rheumatoid arthritis, ulcerative colitis, multiple myeloma, and mitochondrial myopathy, as well as with the use of medications (isoniazid). The laboratory evaluation of a fibrinogen disorder reveals prolongation of TT along with a low plasma fibrinogen level and/or abnormal fibrinogen function. The prothrombin time and activated partial thromboplastin time may be variably prolonged depending on the nature and severity of the fibrinogen disorder. Treatment to increase the level of functional fibrinogen to greater than 100 to 150 mg/dL (2.9-4.4 µmol/L) is indicated for individuals with afibrinogenemia, hypofibrinogenemia, or dysfibrinogenemia who have significant bleeding or require surgery. A target level of 150 to 200 mg/dL (4.4-5.9 µmol/L) is used for intracranial bleeding. Fibrinogen concentrate, cryoprecipitate, and fresh frozen plasma are used when fibrinogen replacement is required for afibrinogenemia and hypofibrinogenemia. Evidence-based management of dysfibrinogenemia is limited because of the heterogeneous phenotypes and increased risk of thrombosis. Fibrinogen concentrates have a lower risk of transfusion reactions and volume overload compared to cryoprecipitate and fresh frozen plasma, but most centers do not use fibrinogen concentrates because of cost and limited availability. The initial dosing is based on the following formula, with the correction factor (1.7-1.8) provided by the manufacturer: dose (mg/kg body weight) = (target fibrinogen level [in mg/dL] - measured fibrinogen level [in mg/dL]) ÷ correction factor (in mg/dL per mg/kg) Each unit of cryoprecipitate contains approximately 200 to 400 mg of fibrinogen in a volume of 10 to 20 mL. Dosing is 1 unit of cryoprecipitate per 5 kg of body weight. Risks include transfusion reaction and thrombosis. Fresh frozen plasma contains all of the coagulation factors present in a unit of blood. Dosing is 10 to 15 mL/kg. Risks include transfusion reaction and thrombosis. Due to volume considerations, cryoprecipitate or fibrinogen concentrate is preferred when available. Antifibrinolytic agents, tranexamic acid or epsilon-aminocaproic acid, may be used to treat or prevent mucosal bleeding. They should be used with caution in patients with a history of thrombosis. PREP Pearls Hypofibrinogenemia is defined as a fibrinogen level less than 150 mg/dL (4.4 µmol/L). Fibrinogen defects may be quantitative, qualitative, or both. Liver disease is the most common cause of acquired hypofibrinogenemia and dysfibrinogenemia. Children with fibrinogen disorders may have bleeding, thrombosis, or both. The laboratory evaluation of a fibrinogen disorder reveals prolongation of thrombin time along with a low plasma fibrinogen level and/or abnormal fibrinogen function.

A 4-week-old infant had an abnormal T-cell receptor excision circles (TRECs) newborn screening result. Flow cytometry documented severe T-cell lymphopenia with no CD3+ cells detected. CD19+ B cells were detected, but natural killer cells were absent. A mutation in IL2RG (X-linked severe combined immunodeficiency) is identified, and there is no maternal engraftment. The patient appears healthy without infection and has an older sister who is a 10/10 HLA-matched donor. Hematopoietic stem cell transplantation is planned. Of the following, the MOST appropriate preparative regimen for this infant prior to transplantation is A.anti-thymocyte globulin only B.busulfan and cyclophosphamide C.low-dose busulfan only D.no conditioning medications

The more widespread usage of the T-cell receptor excision circles (TRECs) assay as part of newborn screening has identified the majority of patients with severe combined immunodeficiency (SCID) prior to the onset of infections. This early identification has allowed SCID patients who are otherwise healthy to be evaluated for hematopoietic stem cell transplantation (HSCT) at very young ages. The infant in this vignette has a fully matched sibling donor and is infection-free without evidence of maternal engraftment; therefore, the infant may be successfully transplanted without a conditioning regimen. For patients who have an HLA-matched sibling donor, pre-HSCT conditioning regimens are generally not recommended because the risk of T-cell graft failure, graft rejection, and graft-vs-host disease is too low to warrant myeloablation (eg, busulfan alone or with cyclophosphamide) or immunotherapy (eg, anti-thymocyte globulin). There is significant variability in the HSCT approach to infants with SCID who lack an appropriate HLA-matched sibling donor. Historically, the goal of HSCT in patients with SCID has been focused on engraftment of the T-cell lineage to abate the severe T-cell deficiency while carefully balancing risks of infection, maternal engraftment, and/or radiosensitivity secondary to the underlying defect. As such, many patients have received minimal to no pre-HSCT conditioning treatment. A potential consequence of this approach is failure of B-cell engraftment whereby some patients may require lifelong intravenous immunoglobulin infusions. The future goals of HSCT may shift to include engraftment of both T cells and B cells. PREP Pearls The majority of patients with severe combined immunodeficiency who have an appropriate HLA-matched sibling donor do not require a conditioning regimen prior to hematopoietic stem cell transplantation. Hematopoietic stem cell transplantation for individuals with severe combined immunodeficiency who lack HLA-matched sibling donors remains controversial.

A full-term neonate has developed a renal vein thrombosis based on ultrasonographic findings for the evaluation of hematuria. She has significant purpuric and early necrotic skin lesions around her head and buttocks. Bilateral lower extremities are swollen. Laboratory data are shown: Laboratory Test Result White blood cell count Normal Platelet count 110 × 103/µL (110 × 109/L) Hemoglobin level Normal Prothrombin time 18 s (upper limit of normal, 16 s) Activated partial thromboplastin time 48 s (upper limit of normal, 43 s) D-dimers Elevated Protein C 5% Protein S 10% Antithrombin 40% In addition to starting anticoagulation, the MOST appropriate treatment at this time is A.cryoprecipitate B.factor VIIa C.fresh frozen plasma D.platelets

The neonate in this vignette has a clinical presentation consistent with neonatal purpura fulminans. This rare condition is often caused by a homozygous deficiency of protein C or protein S. However, it may present in patients with compound heterozygous deficiencies (as appears to be the case in this vignette), or it can be acquired in specific clinical situations. Neonatal purpura fulminans is a severe, life-threatening thrombotic condition that presents at birth with perivascular thrombosis, evidence of disseminated intravascular coagulopathy (as seen in this patient with elevated prothrombin time and activated partial thromboplastin time), and sometimes deep vein thrombosis. Treatment consists of anticoagulation and replacement of the missing anticoagulant protein through fresh frozen plasma infusion or specific factor concentrate, if available. Infusion of factor VIIa would worsen the thrombotic state, and cryoprecipitate does not contain proteins C or S. While transfusion of platelets may be indicated for thrombocytopenia with bleeding, it would not address the underlying pathology of the condition. Hypercoagulable states occur in children for a number of reasons (Table 1). Most often, they are acquired and may be due to sepsis, surgical interventions, or medications. However, inherited thrombophilia may be present and predispose a child to thrombosis, particularly when occurring in conjunction with acquired comorbidities. Hereditary thrombophilia may be due to the deficiency of a naturally occurring anticoagulant (protein C, protein S, or antithrombin), or it can be secondary to a gain-of-function mutation rendering a procoagulant enzyme more active or less able to be inhibited (factor V Leiden or prothrombin G20210 mutations). The risk of thrombosis in relation to hereditary thrombophilia varies based on the specific abnormality and the age of the patient (Table 2). Diagnosis of hereditary thrombophilia is made through genetic sequencing or measurement of plasma activity of the affected protein. Attention must be paid to the clinical setting to correctly interpret the results of protein activity assays. Normal ranges of protein activity assays are lower in newborns, and certain clinical states can result in levels lower than baseline . The normal ranges for prothrombin time and activated partial thromboplastin time are higher in newborns because the normal levels of many clotting factor proteins in newborns are lower than in adults. PREP Pearls Hypercoagulable states in children may be acquired or hereditary. Neonatal purpura fulminans is a life-threatening thrombotic condition that can be caused by homozygous protein C or protein S deficiency or a combined inherited thrombophilia. Diagnosis of a hereditary thrombophilia should take into account the current clinical state and age of the patient.

A female newborn with epicanthal folds and slanted palpebral fissures has been diagnosed with an atrioventricular canal defect. Peripheral blood cytogenetics confirms constitutional trisomy 21. Laboratory data are shown: Laboratory Test Result White blood cell count 51,000/µL (51 × 109/L) Hemoglobin 16 g/dL (160 g/L) Platelet count 90 × 103/µL (90 × 109/L) Alanine aminotransaminase Normal Aspartate aminotransaminase Normal Bilirubin Normal The peripheral blood smear shows that 80% of the white blood cells are large blasts with nuclei containing prominent nucleoli along with basophilic staining and blebbing of the cytoplasm. Flow cytometry of the peripheral blood reveals an abnormal CD45-dim population of blasts with expression of CD11B, CD33, CD34, and CD117; weak expression of CD4 and CD7; and partial expression of CD56, CD40, CD13, and platelet-associated antigens CD41, CD61, CD36, CD9, and glycophorin A. Of the following, the gene MOST likely to be mutated in the clonal blast cell population is A.COL18A1 B.DSCR1 C.ETS2 D.GATA1

The newborn in this vignette has Down syndrome (DS) with transient abnormal myelopoiesis (TAM), also known as transient myeloproliferative disorder. Somatic mutations in GATA1 have been detected in nearly all cases of TAM and acute megakaryocytic leukemia occurring in patients who have DS, but not in leukemia occurring in children who do not have DS. GATA1 mutations are linked to leukemogenesis. The diagnosis of TAM in infants with DS is based on the presence of megakaryoblasts in the peripheral blood or bone marrow with the same antigen expression as blast cells in acute megakaryocytic leukemia. Transient abnormal myelopoiesis also develops in infants and children with mosaicism for trisomy 21. COL18A1, DSCR-1, and ETS2 are located on chromosome 21. COL18A1 encodes endostatin, which inhibits angiogenesis and arrests tumor development in human and animal models. The DSCR-1 gene product inhibits intracellular calcineurin signaling and also blocks angiogenesis. The ETS 2 oncogene protects against the development of intestinal tumors in a DS mouse model. An extra copy of these genes in children with DS may offer a potential protective effect against the development of solid tumors. In the United States, the incidence of DS is 1 in 1,000 live births, with 3,000 to 5,000 affected children born each year. Children with DS have a 10- to 20-fold higher risk of developing acute lymphoblastic leukemia (ALL) and acute myeloid leukemia (AML) compared to children without DS. Approximately 1:100 to 1:150 children with DS will develop leukemia. Approximately 2% of children diagnosed with ALL have DS. The 2 most common subgroups of childhood B-precursor ALL involve qualitative or quantitative abnormalities of chromosome 21, which suggests a relationship between leukemogenesis and chromosome 21. Children with DS have a 400-fold increased risk of developing the AML M7 subtype, acute megakaryocytic leukemia. In newborns with DS, 4% to 10% have TAM, a potential precursor to acute megakaryocytic leukemia that often resolves without therapy. Approximately 20% to 25% of children with DS and a history of TAM eventually develop AML. Although children with DS have an increased risk of developing acute leukemia, there is no increased risk of solid tumors. In fact, large population-based studies have shown that solid tumors occur less frequently in children with DS compared to children without DS. PREP Pearls Children with Down syndrome have a 10- to 20-fold higher risk of developing leukemia compared to children without Down syndrome. Approximately 2% of children diagnosed with acute lymphoblastic leukemia have Down syndrome. Children with Down syndrome have a 400-fold increased risk of developing acute megakaryocytic leukemia compared to children without Down syndrome. Solid tumors occur less frequently in children with Down syndrome compared to children without Down syndrome.

A 2-year-old girl has been referred by her pediatrician for evaluation of an abdominal mass felt during her annual health supervision visit. Her medical history is significant for a prolonged neonatal intensive care unit stay for neonatal hypoglycemia and omphalocele requiring surgical correction. Today, her vital signs are significant for an elevated blood pressure of 122/84 mm Hg. A painless 7-cm mass is found on the left side of her abdomen. Her left upper and lower extremities appear larger than the right side. Of the following, the genetic syndrome that BEST explains the development of the tumor described in this patient is A.Beckwith-Wiedemann syndrome B.Denys-Drash syndrome C.Simpson-Golabi-Behmel syndrome D.WAGR syndrome

The patient in the vignette has Beckwith-Wiedemann syndrome (BWS) and associated Wilms tumor. An overgrowth and cancer predisposition disorder, BWS has a prevalence of 1 in 14,000. It is caused by dysregulation at chromosome 11p15.5 locus which is responsible for prenatal and childhood growth. Patients are often seen at birth with excessive birth weight (large for gestational age), large tongue (macroglossia), and enlargement of visceral organs (organomegaly); overgrowth (macrosomia) usually continues throughout childhood. Lateralized asymmetric overgrowth manifesting as enlargement of one side of the body can also be seen. Patients can also have neonatal hypoglycemia, likely due to overgrowth and excessive insulin secretion from pancreas and abdominal wall defects (umbilical hernia or omphalocele or diastasis recti). Patients with BWS are prone to renal abnormalities, such as enlarged kidneys, renal cysts, medullary dysplasia, and hydronephrosis. The risk of malignancy in patients with BWS is approximately 4% to 21%. These patients are at risk for developing embryonal tumors with Wilms tumor being the most common diagnosis, affecting 60% of patients with malignancy. Patients with BWS are usually recommended to undergo tumor surveillance through quarterly abdominal ultrasonography until the age of 7 years. Wilms tumor is one of the most common retroperitoneal tumors occurring in childhood, accounting for nearly 6% of childhood-onset cancers. The mean age at diagnosis for males and females with unilateral Wilms tumor is 42 months and 47 months, respectively. Clinically, patients present with abdominal mass with or without abdominal pain, hypertension, hematuria, fever, and malaise. While the majority of cases are sporadic, familial cases, congenital anomalies, and cancer predisposition syndromes account for up to 9% of Wilm tumor cases. The Table reviews genetic syndromes associated with Wilms tumor. Similar to BWS, Simpson-Golabi-Behmel syndrome is an overgrowth disorder but with an X-linked inheritance pattern. Patients with this condition can also have macrosomia, macrostomia, and macroglossia with abdominal wall defects. Other features include coarse facial features with ocular hypertelorism and skeletal and cardiovascular defects. The history of neonatal hypoglycemia in this vignette is consistent with the diagnosis of BWS. It is not consistent with WAGR syndrome either, which is a constellation of Wilms tumor, aniridia, genitourinary abnormalities such as ambiguous external genitalia, and intellectual disability. Similarly, Denys-Drash syndrome is a triad of Wilms tumor, genitourinary abnormalities in males, and nephropathy. Genitourinary abnormalities in these patients can range from mild hypospadias to pseudohermaphroditism. PREP Pearls Beckwith-Wiedemann syndrome is an overgrowth and cancer predisposition disorder with a higher risk of embryonal tumors, primarily Wilms tumor, and hepatoblastoma. Patients with Beckwith-Wiedemann syndrome present with macrosomia, hemihypertrophy, abdominal wall defects, and neonatal hypoglycemia. Patients with Beckwith-Wiedemann syndrome are recommended to undergo tumor surveillance through quarterly abdominal ultrasonography from diagnosis until the age of 7 years. ABP Content Specifications(s)/Content Area Know the congenital anomalies that are associated with an increased risk of Wilms tumor Suggested Readings Brioude F, Kalish JM, Mussa A, et al. Expert consensus document: Clinical and molecular diagnosis, screening and management of Beckwith-Wiedemann syndrome: an international consensus statement. Nat Rev Endocrinol. 2018;14(4):229-249. doi:10.1038/nrendo.2017.166 Scott RH, Stiller CA, Walker L, Rahman N. Syndromes and constitutional chromosomal abnormalities associated with Wilms tumour. J Med Genet. 2006;43(9):705-715. doi:10.1136/jmg.2006.041723

A 16-year-old female adolescent has had worsening shortness of breath and lightheadedness for 1 day. She has been taking a low-dose combination oral contraceptive pill for the past 3 months. Her weight is 110 kg. She has a respiratory rate of 36 breaths/min, heart rate of 50 beats/min, blood pressure of 88/50 mm Hg, and oxygen saturation of 90% on room air. Computed tomographic pulmonary angiography reveals a large filling defect in the bifurcation of the pulmonary arteries. Of the following, the BEST treatment for this patient is A.alteplase B.bivalirudin C.clopidogrel D.warfarin

The patient in this vignette has a massive pulmonary embolism (PE), which is designated by the American Heart Association as acute PE with sustained hypotension (defined as systolic blood pressure < 90 mm Hg for at least 15 minutes or requiring inotropic support, not due to a cause other than PE, such as arrhythmia, hypovolemia, sepsis, or left ventricular dysfunction), pulselessness, or persistent profound bradycardia (defined as heart rate < 40 beats/min with signs or symptoms of shock). Given the severity of her condition with hemodynamic instability, the treatment associated with the best outcome for this patient would be systemic or catheter-directed thrombolysis. Thrombolysis includes fibrinolytic agents, such as a tissue plasminogen activator (eg, alteplase), to convert the patient's plasminogen to plasmin, cleaving fibrin and generating fibrin-split products such as d-dimers. Antiplatelet agents, such as clopidogrel, are not a first-line treatment for venous thromboemboli such as PE. Anticoagulation with warfarin results in passive thrombus reduction and may be indicated after thrombolysis, but is not likely to immediately reduce thrombus burden. Bivalirudin is a direct thrombin inhibitor that is indicated in patients with suspected or confirmed heparin-induced thrombocytopenia and is not currently approved for patients younger than 18 years. The incidence of PE has been steadily increasing in children, perhaps due to the increased use of central venous catheters and longer survival of critically ill children with thrombophilic risk factors. Based on inpatient data, peaks in the incidence of pediatric PE occur in the infant/toddler age group and in adolescence. The presentation of PE in children may be subtle and less obvious than in adults. A high index of suspicion for PE is needed for patients with thrombophilic risk factors, such as obesity and estrogen therapy (as seen in the patient in this vignette). Given its speed and reliability, computed tomographic pulmonary angiography has become the primary imaging modality to diagnose PE, despite its disadvantages of ionizing radiation exposure and relative insensitivity in detecting small, subsegmental emboli. The clinical severity of PE in children ranges from an incidental finding in an asymptomatic patient to a massive PE with cardiovascular collapse. Once a PE is diagnosed, risk stratification, based on various models from the adult data, may help determine the best course of treatment. High-risk or massive PE presents with cardiovascular compromise such as hypotension (defined as systolic blood pressure < 90 mm Hg). Intermediate-risk or submassive PE is associated with normal blood pressure but with evidence of right-sided heart strain. Right-sided heart strain can be measured by: echocardiography as right ventricle dilatation or hypokinesis of the interventricular septum; electrocardiography as right-axis deviation, right bundle branch block, or ST and T-wave abnormalities; or laboratory evaluation with elevation of the biomarker levels cardiac troponin, brain-type natriuretic peptide, or heart-type fatty acid-binding protein. Low-risk PE lacks the aforementioned cardiopulmonary features. Other baseline laboratory studies should be obtained prior to starting treatment for PE, including a complete blood cell count, prothrombin time, activated partial thromboplastin time, fibrinogen level, liver function tests, and renal function tests. Checking plasminogen levels, particularly in neonates and young children, may be useful prior to systemic thrombolysis, to determine the need for supplementation with fresh frozen plasma. The recommended initial anticoagulation for acute PE is unfractionated heparin or low-molecular-weight heparin (enoxaparin or fondaparinux). For patients with high-risk PE, presenting with hypotension, bradycardia, hypoxia, and a large saddle PE, systemic or catheter-directed thrombolysis in addition to heparin anticoagulation may result in faster clot resolution and reduction of right ventricular strain, leading to improved survival. The decision to use systemic vs catheter-directed thrombolysis for critically ill patients, such as the patient in the vignette, may depend on the availability of institutional resources. Although the optimal dosing regimen for thrombolysis in pediatrics has not yet been established, there are 2 widely used dosing strategies. A higher dose of alteplase for a shorter duration of time may result in improved clot resolution as well as an increased risk for bleeding. A lower dose of alteplase for a longer duration has also shown efficacy with a lower risk for bleeding. Following thrombolysis, patients should continue anticoagulation with heparin or warfarin for at least 3 months. PREP Pearls The signs and symptoms of pulmonary embolism in children may be less obvious than in adults, therefore a high index of suspicion for pulmonary embolism in patients with thrombophilic risk factors is recommended. Initial anticoagulation for acute pulmonary embolism includes unfractionated heparin or low-molecular-weight heparin (enoxaparin or fondaparinux). For patients with high-risk or massive pulmonary embolism presenting with cardiovascular compromise, systemic or catheter-directed thrombolysis in addition to heparin anticoagulation may be required. Intermediate-risk or submassive pulmonary embolism is associated with normal blood pressure but evidence of right-sided heart strain on echocardiography, electrocardiography, or laboratory evaluation of serum biomarker levels.

A 4-year-old boy has a 2-week history of intermittent dyspnea, facial swelling, and enlarged cervical lymph nodes. One week ago his pediatrician prescribed prednisone for 5 days, and his parents noted a brief improvement in dyspnea and reduction in lymph node size. However, his breathing is currently labored, and he requires 3 pillows to sleep upright at night. He is mildly tachypneic sitting upright with facial plethora and swelling. He has an oxygen saturation of 100%. He has a right 3-cm anterior cervical node that is firm and fixed. His lungs are clear to auscultation. He does not have hepatosplenomegaly or other abdominal masses. Of the following, the BEST next test for this patient is A.arterial blood gas B.chest computed tomography C.chest radiography D.complete blood cell count

The patient in this vignette has enlarged lymph nodes, orthopnea, facial swelling, and dyspnea, which temporarily improved after treatment with corticosteroids. These obstructive airway symptoms raise concern for an anterior mediastinal mass causing tracheal or bronchial airway compression/compromise. His facial swelling is likely due to tumor compression resulting in superior vena cava syndrome (SVCS). The best next test would be chest radiography to confirm the extent of the mass. Although chest computed tomography would also demonstrate a mediastinal mass, laying flat for such imaging, particularly without medical supervision, may worsen airway obstruction. An arterial blood gas level would not help to determine the cause of the respiratory distress in this patient who is not hypoxic. A complete blood cell count is appropriate to determine the presence of anemia, leukocytosis, or blasts; however, these findings would not explain the facial swelling or orthopnea. Given the patient's age and quick but transient clinical improvement on steroids, the likely etiology of his symptoms would be a rapidly enlarging anterior mediastinal mass caused by T-lymphoblastic lymphoma/leukemia. Other non-Hodgkin as well as Hodgkin lymphomas may present in a similar fashion but tend to affect older children and adolescents or have a more gradual onset of symptoms. Neuroblastomas, germ cell tumors, and rarely rhabdomyosarcomas can also present as mediastinal masses compressing the main airways in infants and young children. The management of SVCS in a child is summarized by the algorithm in the Figure. The next step in SVCS management after chest radiography is to address tumor lysis syndrome by conducting a laboratory evaluation (complete blood cell count, renal and liver function tests, and levels of calcium, electrolytes, magnesium, phosphorus, and uric acid) in addition to starting aggressive intravenous hydration. If the patient is closely monitored for cardiopulmonary stability while lying supine, chest computed tomography and echocardiography (to evaluate for right atrial compression by the mass) may be performed. The patient in this vignette likely has T-lymphoblastic lymphoma and may have peripheral blood lymphoblasts (suggesting T-lymphoblastic leukemia) or a pleural effusion with leukemia cells, which could be assessed by flow cytometry. If indirect means of diagnosis are not possible, then various approaches to anesthesia may be used, depending on airway stability, so that a surgeon or interventional radiologist can obtain a lymph node or mediastinal tissue biopsy. If the patient is sufficiently stable for general anesthesia, a surgical biopsy should be promptly obtained. If the patient cannot tolerate general anesthesia, then local anesthesia or sedation should be considered because an "awake biopsy" can be difficult in young children. If the patient demonstrates cardiorespiratory instability and cannot tolerate tissue biopsy, then empiric chemotherapy can be considered. In this patient who has shown transient response to oral corticosteroids and most likely has T-lymphoblastic lymphoma, administering intravenous methylprednisolone in an intensive care unit while monitoring for tumor lysis or tumor swelling may be indicated. If the patient clinically improves after high-dose steroids for 24 hours, then tissue diagnosis, lumbar puncture, or bone marrow testing with anesthesia can be considered. If the patient remains too unstable for tissue diagnosis, the addition of cyclophosphamide and/or an anthracycline (eg, doxorubicin) or emergent radiation can be considered. Continuous assessment of cardiorespiratory stability while seeking opportunities for tissue diagnosis is essential, because empiric treatment may potentially confound the diagnosis and the opportunity for cure. PREP Pearls An anterior mediastinal mass compressing the trachea, bronchi, or superior vena cava is a common oncologic emergency resulting from the rapid progression of certain leukemias or lymphomas. In the rare instances of respiratory compromise that is life-threatening, emergent corticosteroids, chemotherapy, or radiation therapy can be administered prior to obtaining a mediastinal tissue biopsy for diagnosis.

A 14-year-old adolescent girl with menorrhagia is referred for evaluation. Laboratory Test Result Factor VIII activity 51% Ristocetin cofactor activity 27% von Willebrand factor antigen 29% Von Willebrand multimer analysis reveals a normal multimer pattern but in reduced concentration. Platelet count, prothrombin time, activated partial thromboplastin time, hemoglobin, and mean corpuscular volume are normal. Of the following, the treatment MOST likely to correct the hemostatic defect in this patient is A.activated prothrombin complex concentrate B.aminocaproic acid C.desmopressin D.recombinant factor VIII

The patient in this vignette has laboratory findings consistent with type 1 von Willebrand disease (vWD), which is a heterozygous quantitative deficiency of von Willebrand factor (vWF). Desmopressin (1-deamino-8-D-arginine vasopressin) is effective in the majority of patients with type 1 vWD, increasing vWF levels approximately 2 to 4 fold. Although an antifibrinolytic agent, such as aminocaproic acid or tranexamic acid, can be useful in controlling bleeding symptoms, its mechanism of action does not reverse the hemostatic defect of patients with vWD. Neither recombinant factor VIII nor activated prothrombin complex concentrate, which contains coagulation factors II, VII, IX, and X, would correct the deficiency of vWF found in this patient. Desmopressin is a synthetic analogue of vasopressin that stimulates release of vWF from the Weibel-Palade bodies in endothelial cells, increases plasma factor VIII levels, and enhances platelet adhesion to the vessel wall. Because tissue-type plasminogen activator is also stored in the Weibel-Palade bodies of endothelial cells, desmopressin also causes its release. Tissue-type plasminogen activator converts plasminogen to plasmin but is rapidly complexed to α2-antiplasmin; therefore, desmopressin administration does not lead to clinically significant fibrinolysis in the circulation. Desmopressin is an established therapy for patients with vWD and mild hemophilia A and an emerging therapy for platelet function disorders. It is most likely to be effective in type 1 and type 2N vWD where there are mild deficiencies. Desmopressin is relatively contraindicated in type 2B vWD where there is thrombocytopenia, because it may lead to increased platelet aggregation and clearance. It is unlikely to be effective in type 3 vWD (complete deficiency of vWF) and other forms of type 2 vWD (qualitative disorders). Desmopressin is available in intravenous, subcutaneous, and concentrated intranasal formulations. Of note, the more concentrated intranasal form of desmopressin (150 µg/spray) approved for use in patients with bleeding disorders differs from that used to treat diabetes insipidus and enuresis (10 µg/spray, 0.01% concentration). The National Heart, Lung, and Blood Institute recommends that patients be tested to document adequate response to desmopressin prior to a clinical event requiring hemostasis. A desmopressin challenge can be performed by checking plasma levels of the deficient coagulation factor (eg, vWF antigen or factor VIII) at baseline before desmopressin administration and then 30 to 90 minutes and sometimes 4 hours after administration to document a rise of the factor level ideally into the normal range as well as the duration of the rise in the factor level. The antidiuretic properties of desmopressin may put patients at risk for hyponatremic seizures, and it is therefore not recommended in infants younger than 11 months. Patients should be counseled to limit fluid intake for up to 24 hours after receiving desmopressin. PREP Pearls Desmopressin (1-deamino-8-D-arginine vasopressin) is a synthetic analogue of vasopressin that stimulates release of von Willebrand factor from the Weibel-Palade bodies in endothelial cells, increases plasma factor VIII levels, enhances platelet adhesion to the vessel wall, and causes the release of tissue-type plasminogen activator. Desmopressin can cause a 2- to 4-fold increase in von Willebrand factor and factor VIII levels and is an established therapy for patients with von Willebrand disease, types 1 and 2N, or mild hemophilia A. Patients with von Willebrand disease or mild hemophilia A should have a documented response to desmopressin, as evidenced by an adequate increase in von Willebrand antigen or factor VIII level, before relying on desmopressin to control excessive bleeding.

A 16-year-old adolescent girl has worsening cough, dyspnea when lying flat, and hoarseness. Computed tomography (Figure) shows a large mass spanning from the right hilum into the right supraclavicular area. The mass compresses the superior vena cava and elevates the right hemidiaphragm. She is clinically stable. Results of an open supraclavicular lymph node biopsy are diagnostic of primary mediastinal B-cell lymphoma. Of the following, the treatment modality that is MOST likely to be a part of her therapy is A.autologous stem cell transplant B.chemotherapy C.radiation D.surgical resection of her mediastinal mass

The patient in this vignette has primary mediastinal B-cell lymphoma (PMBL) that is compressing the airway and causing her to cough. She presents with hoarseness caused by compression of the recurrent laryngeal nerve as well as an elevated right hemidiaphragm from compression of the phrenic nerve. Primary mediastinal B-cell lymphoma is a distinct subtype of diffuse large B-cell lymphoma (DLBCL) that accounts for about 10% to 20% of all pediatric DLBCLs. Compared to other DLBCLs, PMBL can have an inferior prognosis and requires aggressive therapy for cure. Primary mediastinal B-cell lymphomas are relatively more resistant to chemotherapy than other non-Hodgkin lymphomas, so historically most patients received radiation therapy to the mediastinum. While often effective, radiation therapy has contributed to a significantly increased risk of late effects including cardiotoxicity, pulmonary toxicity, and secondary malignancy such as breast cancer. To eliminate radiation in most patients, efforts over the past decade have focused on intensifying chemotherapy. Trials of a dose-intense chemotherapy have been developed using etoposide, doxorubicin, and cyclophosphamide along with vincristine, prednisone, and rituximab (DA-EPOCH-R). Results of this chemotherapy regimen in an adult population were published in 2013 by Dunleavy et al and showed a 5-year event-free survival of 93% and an overall survival of 97%. Only 2 of the 51 patients did not go into a complete remission, and they both received radiation. A combined European/North American trial (COG ANHL1131) sought to validate this approach in an exclusively pediatric population. This study closed in 2016, and data are not yet published. Nevertheless, chemotherapy is the backbone of treatment for PMBL. Autologous stem cell transplant may be considered in certain refractory cases, but it is not required for most patients. Radiation therapy, as discussed above, was traditionally required for many patients. However, with newer more dose-intense chemotherapy regimens many patients can avoid radiation. Surgical resection of a mediastinal mass is not feasible and would not be recommended. PREP Pearls Primary mediastinal B-cell lymphomas are a subtype of diffuse large B-cell lymphomas that require intensive chemotherapy. Radiation therapy is required in primary mediastinal B-cell lymphomas that are refractory to chemotherapy.

After completing reinduction chemotherapy with a clofarabine-based regimen, a 4-year-old girl with relapsed acute lymphoblastic leukemia underwent matched-related hematopoietic stem cell transplantation 25 days ago. Her conditioning regimen included busulfan and cyclophosphamide. Palifermin was given to prevent mucositis. Over the last 3 days, her weight has increased by 4.5 kg (7% of body weight). Today, she has mild right upper quadrant discomfort. She has scleral icterus, diffuse abdominal tenderness, and an enlarged liver, 4 cm below the right costal border. Of the following, the MOST likely cause for the patient's symptoms is her recent exposure to A.busulfan B.clofarabine C.cyclophosphamide D.palifermin

The patient in this vignette has signs and symptoms consistent with sinusoidal obstructive syndrome (SOS), previously known as veno-occlusive disease. Although clofarabine has rarely been associated with the development of SOS, busulfan has been associated with a relatively greater risk of developing SOS. Cyclophosphamide and palifermin are not commonly associated with SOS. Busulfan is an alkylating agent that reacts with the N-7 position of guanosine and interferes with DNA replication and transcription of RNA. High-dose busulfan (4 mg/kg/dose, daily, for 4 days in patients who weigh more than 10 kg or 3.2 mg/kg/dose, daily, for 4 days in patients who weigh less than 10 kg) is commonly used as part of multiagent conditioning regimens, usually along with cyclophosphamide, for children undergoing hematopoietic stem cell transplantation for malignant and nonmalignant conditions. Lower doses of busulfan (0.8 mg/kg/dose every 6 hours) have been used as a palliative therapy option in patients with relapsed chronic myeloid leukemia. Busulfan has a very narrow therapeutic index. High doses are associated with the development of SOS whereas low doses increase the risk for cancer recurrence or hematopoietic stem cell graft failure. It is estimated that 21% of children exposed to busulfan will develop some degree of SOS. The most common adverse effect associated with busulfan therapy is myelosuppression. Gastrointestinal toxicities including nausea, vomiting, and mucositis are often seen in patients receiving high doses of busulfan. Busulfan-related, diffuse interstitial fibrosis or bronchopulmonary dysplasia presenting with cough and fever may progress to respiratory failure. High doses of busulfan have also been associated with the onset of seizures, for which a prophylactic anticonvulsant regimen is indicated. Female individuals who have received busulfan have a high incidence of ovarian failure. The oral formulation of busulfan has wide interpatient and intrapatient variability in plasma concentrations. This variability is mostly dependent on the patient's age and weight. Variability of plasma levels is more pronounced in small children because of their changing hepatic metabolism. For this reason, intravenous administration is preferred because it offers more predictive pharmacokinetics. Other factors accounting for drug variability include a patient's specific diagnosis, prior exposure to busulfan, as well as concomitant chemotherapy. Busulfan has a very short half-life of 2 to 3 hours and is mainly excreted through the urine. Glutathione conjugation is its primary route of elimination. As a result of more rapid glutathione conjugation in children younger than 5 years compared to older individuals, busulfan clearance in children is faster than in adults. Pharmacokinetic studies are used to monitor busulfan levels, and dose adjustments help to achieve and maintain adequate plasma levels. Target levels in children vary by protocol. PREP Pearls Busulfan is an alkylating agent that interferes with DNA replication and transcription of RNA. The oral formulation of busulfan has wide interpatient and intrapatient pharmacokinetic variability, primarily dependent on a patient's age and weight. The most common adverse effect associated with busulfan is myelosuppression. High doses of busulfan are associated with the development of hepatic sinusoidal obstruction syndrome, previously known as veno-occlusive disease, in up to 21% of treated children. ABP Content Specifications(s)/Content Area Understand the mechanism of action of busulfan Understand the metabolism of busulfan, including the role of age Know the toxicities associated with busulfan therapy

A 16-year-old female adolescent has dizziness, disorientation, and fever. She has scleral icterus, diffuse purpura and petechiae on her trunk and extremities, but no lymphadenopathy or hepatosplenomegaly. Laboratory Test Result White blood cell count 18,500/µL (18.5 × 109/L) Hemoglobin 5.4 g/dL (54 g/L) Mean corpuscular volume 82 fL Platelet count 18 × 103/µL (18 × 109/L) Reticulocytes 12% Direct Coombs test Negative Peripheral blood smear Schistocytes Prothrombin time 12 s INR 1.2 Activated partial thromboplastin time 30 s Creatinine 1.5 mg/dL (133 µmol/L) Indirect bilirubin 5.5 mg/dL (94 µmol/L) Lactate dehydrogenase 1,250 U/L Of the following, the BEST definitive treatment for this patient at this time is A.anti-D antibody (rh(D) immune globulin) B.intravenous immunoglobulin C.plasma exchange D.transfusion of fresh frozen plasma

The patient in this vignette with nonimmune microangiopathic hemolytic anemia and thrombocytopenia has a presentation consistent with a thrombotic microangiopathy, such as thrombotic thrombocytopenic purpura (TTP). The definitive treatment of choice for suspected TTP is plasma exchange, often accompanied by administration of glucocorticoids or rituximab. Anti-D antibody and intravenous immunoglobulin are used to raise platelet counts in immune thrombocytopenia and are not effective treatments for TTP. Transfusion of platelets, packed red blood cells, or fresh frozen plasma may be used in TTP if the patient needs these interventions for supportive care or as a temporizing measure until plasma exchange can be performed. In 2009, Swisher et al reported the results of a prospective study of patients with TTP who received platelet transfusions. They found no evidence of harm from platelet transfusions as was previously thought. In the absence of acute bleeding, prompt initiation of plasma exchange would be the best definitive treatment for the patient in this vignette with presumed TTP. Systemic platelet aggregation, organ ischemia, thrombocytopenia, and hemolysis characterize TTP. Platelet aggregates in the microcirculation cause erythrocyte fragmentation (ie, schistocytes) as the blood passes through the partially occluded vessels and eventually results in tissue ischemia. The classic pentad of thrombocytopenia, microangiopathic hemolytic anemia, neurologic abnormalities, renal disease, and fever is not required to make the diagnosis of TTP. Severe thrombocytopenia, a markedly elevated lactate dehydrogenase level, and schistocytes on the peripheral smear in the absence of a coagulopathy are adequate to support the diagnosis. Most cases of TTP are caused by decreased activity of ADAMTS13 (adisintegrinand metalloproteinase with 8 thrombospondin-1-like domains, member 13), a plasma von Willebrand factor (vWF)-cleaving metalloprotease. In acquired TTP, an IgG autoantibody leads to a marked decrease in ADAMTS13 and the accumulation of ultralarge vWF multimers in the plasma, which are highly adhesive to the vWF and fibrinogen receptors on platelets. This decreased ADAMTS activity can help distinguish TTP from other thrombotic microangiopathies such as "classical" hemolytic uremic syndrome (HUS) and bone marrow transplantation-associated thrombotic microangiopathy. Demonstration of the antibody or an inhibitor of ADAMTS13 confirms an acquired case. Familial, congenital, chronic relapsing TTP is caused by mutations in ADAMTS13 and often presents in infancy or childhood with an extremely low (often < 5%) ADAMTS13 level and no detectable antibody. Patients with acquired TTP may have a single episode or recurrent episodes of TTP. If a patient is suspected of having TTP, the ADAMTS13 assay should be sent before plasma product transfusion; however, treatment should not be delayed while awaiting the results. There is significant clinical overlap between atypical HUS and TTP. In general, atypical HUS is a renal thrombotic microangiopathy associated with bloody diarrhea and the triad of microangiopathic hemolytic anemia, renal failure, and thrombocytopenia (although less severe with platelet counts greater than 50 × 103/µL [50 × 109/L]). Atypical HUS and acute TTP may be associated with mortality rates as high as 25% and 20%, respectively, therefore prompt initiation of treatment is necessary. Plasma exchange replaces the patient's plasma with plasma containing ADAMTS13 while also removing the anti-ADAMTS13 IgG. Plasma exchange is continued daily but may need to be increased in frequency or plasma volume depending on the patient's response, as measured by improvement in blood cell counts. Concurrent treatment with high-dose steroids or rituximab is beneficial in acute TTP and may result in reduced plasma exchange and quicker response to therapy. PREP Pearls In thrombotic thrombocytopenic purpura, platelet aggregates in the microcirculation cause erythrocyte fragmentation (ie, schistocytes) as the blood passes through the partially occluded vessels which eventually results in tissue ischemia. The classic pentad of thrombocytopenia, microangiopathic hemolytic anemia, neurologic abnormalities, renal disease or dysfunction, and fever is not required to make the diagnosis of thrombotic thrombocytopenic purpura. Severe thrombocytopenia, markedly elevated lactate dehydrogenase level, and schistocytes on the peripheral smear are sufficient to initiate therapy, in the absence of an alternative diagnosis. Since thrombotic thrombocytopenic purpura is associated with a 20% mortality rate, treatment with plasma exchange should be promptly initiated.

A 2-year-old boy has had several weeks of worsening pallor, intermittent fevers, easy bruising, and rash. Hepatosplenomegaly, diffuse lymphadenopathy, a maculopapular rash, and multiple bruises are present. Laboratory data are shown: Laboratory Test Result White blood cell count 43,000/µL (43 × 109/L) Monocytes 15% Hemoglobin 6.8 g/dL (68 g/L) Platelet count 35 × 103/µL (35 × 109/L) There are no excess basophils or eosinophils. The peripheral blood smear is notable for nucleated red blood cells, abundant monocytes (some of which appear dysplastic), and decreased platelets. Rare blasts are seen as well as some immature myeloid elements (myelocytes and metamyelocytes). A bone marrow aspirate and biopsy reveals a hypercellular marrow with 12% blasts. Immunophenotyping is consistent with a myeloid clone featuring some monocytic differentiation. Of the following, the test that will BEST confirm this patient's diagnosis is A.colony assay to assess for granulocyte-macrophage colony stimulating factor hypersensitivity B.cytogenetic analysis C.DNA sequencing of genes associated with RAS/MAPK signaling pathway D.hemoglobin F percentage

The presentation in this vignette is classic for juvenile myelomonocytic leukemia (JMML), a rare hematologic malignancy with an incidence of 1.2 per million children per year. It typically affects young children (median age, 2 years) with a male predominance of 2:1 to 3:1, and it is classified as a myelodysplastic/myeloproliferative disorder according to the World Health Organization. In addition to the symptoms and signs described in the vignette, affected patients can exhibit bleeding, cough, diarrhea, abdominal distension, and failure to thrive. Excess proliferation of abnormal monocytes and immature granulocytes in the bone marrow and peripheral blood that can infiltrate the lungs, liver, spleen, and intestines characterizes JMML. Greater than 90% of cases feature a germline or somatic mutation in 1 of 5 genes (PTPN11, N-RAS, K-RAS, NF1, CBL) in the RAS/MAPK (mitogen-activated protein kinase) pathway that regulates cell-cycle control, survival, proliferation, and differentiation. Although the precise molecular mechanisms have yet to be elucidated, these mutations lead to RAS/MAPK pathway hyperactivation and consequent hypersensitivity of myeloid progenitor cells to granulocyte-macrophage colony stimulating factor (GM-CSF). Long considered a hallmark of the disorder, hypersensitivity to GM-CSF is currently a minor diagnostic criterion for JMML when a typical mutation has not been identified. Diagnostic criteria for JMML are shown in the Table . Over the last decade, key driver mutations in the RAS/MAPK pathway genes have been recognized as causal for JMML. Mutational analysis of specific RAS/MAPK genes can also inform prognosis and therapeutic decision-making. The presence of an oncogenic mutation has thus become a major diagnostic criterion for JMML. Furthermore, distinguishing between germline and somatic mutations, by testing both hematopoietic and nonhematopoietic cells (eg, both bone marrow and skin fibroblast cells) is critical in determining the presence of an underlying Ras-opathy or neuro-cardio-facio cutaneous syndrome that can predispose to cancer (eg, neurofibromatosis-1, Noonan syndrome [NS]). Physical features of these syndromes must be carefully sought when evaluating any patient with JMML. Neurofibromatosis-1 is caused by germline mutations in NF1 and confers a 200- to 500-fold increased risk for JMML. Between 10% to 15% of JMML cases arise in patients with neurofibromatosis-1. Noonan syndrome is most commonly due to germline mutations in PTPN11, but N-RAS and K-RAS mutations can also be causative. During infancy, patients with NS can exhibit clinical and laboratory findings suggestive of JMML. However, this clinical picture often spontaneously regresses over several months, leading to recognition of transient myeloproliferative disorder associated with NS (TMD-NS) as a diagnostic entity. Due to symptomatic organ infiltration by abnormal cells, some cases of TMD-NS may require treatment with mild cytoreductive chemotherapy (eg, 6-mercaptopurine), while others may evolve to classic, aggressive JMML. Thus, all cases of TMD-NS require careful clinical follow-up. Although monosomy 7 is the most commonly encountered cytogenetic abnormality in JMML, accounting for 25% of cases, it is considered a minor JMML diagnostic criterion (Table). A normal karyotype is the most typical cytogenetics at diagnosis. Colony assays to assess for hypersensitivity to GM-CSF leading to excess generation of CFU-GM have held an important historical role in evaluating for JMML. However, these assays are not sensitive, not specific, and not standardized amongst laboratories. Hypersensitivity to GM-CSF has also been associated with some human herpesvirus infections, such as cytomegalovirus and human herpesvirus-6, limiting the utility of colony assays to confirm a JMML diagnosis. Hemoglobin F is commonly increased for age in JMML, but is a minor diagnostic criterion. Significant hemoglobin F elevation has been cited by some as a negative prognostic factor. Disorders that have similar presenting clinical and laboratory findings as JMML include cytomegalovirus or human herpesvirus-6 infections, chronic myelogenous leukemia, hemophagocytic lymphohistiocytosis, and Wiskott-Aldrich syndrome. Allogeneic hematopoietic stem cell transplant presently offers the only curative treatment for JMML, conferring a 5-year event-free survival of 52%. However, the clinical course of JMML is heterogeneous, ranging from acute and aggressive to indolent, complicating decision-making around the urgency of transplantation. Involvement of germline mutations of CBL, PTPN11, KRAS, and NRAS can signify a JMML phenotype that includes spontaneous regression of hematologic abnormalities. Although controversial, a clinically vigilant "watch and wait" approach to the timing of transplantation has been advocated in some cases based on genetic testing results. PREP Pearls Cardinal features required to diagnosis juvenile myelomonocytic leukemia include splenomegaly, monocytosis, blast percentage less than 20%, and absence of the Philadelphia chromosome (BCR/ABL negativity). In this clinical setting, specific oncogenic mutations involving the RAS/MAPK pathway confirm the diagnosis. Driver mutations in juvenile myelomonocytic leukemia lead to hyperactivation of the RAS/MAPK pathway, which underlies hypersensitivity to granulocyte-macrophage colony stimulating factor in affected myeloid progenitor cells. Neurofibromatosis-1 and Noonan syndrome are examples of Ras-opathies or neuro-cardio-facio cutaneous syndromes caused by germline mutations affecting the RAS/MAPK pathway that confer markedly increased risk for juvenile myelomonocytic leukemia.

The utility of reticulocyte hemoglobin content as a marker for iron deficiency is being tested in a study population of adolescent, elite soccer players. The goal is to define an abnormal reticulocyte hemoglobin content that would have the sensitivity to detect more than 90% of patients with iron deficiency (defined as ferritin ≤ 12 ng/mL [27 pmol/L]) with greater than 90% specificity. The following data are collected from 58 subjects: Ferritin (ng/mL) Number of Patients Reticulocyte Hemoglobin Content (pg) < 20 20-27 28-35 ≤12 18 5 12 1 >13 40 0 4 36 Of the following, the statement that MOST accurately addresses the sensitivity or specificity of the data is A.sensitivity of a reticulocyte hemoglobin content between 20 to 27 pg in identifying patients with iron deficiency (ferritin ≤12 ng/mL) is greater than 90% B.sensitivity of a reticulocyte hemoglobin content ≤ 27 pg in identifying patients with iron deficiency (ferritin ≤12 ng/mL) is greater than 90% C.specificity of a reticulocyte hemoglobin content <20 pg is 0% D.specificity of a reticulocyte hemoglobin content of 20 to 27 pg is 100%

The sensitivity of a test is the number of true positives (patients correctly identified with the disease) divided by the total number of patients who have the disease (true positives plus false negatives). There are 18 subjects with iron deficiency (ferritin ≤12 ng/mL). One of these 18 subjects has a reticulocyte hemoglobin content >27 pg. A level of ≤27 pg would allow correct detection of 17 of the 18 patients with iron deficiency for a sensitivity of 94%. A reticulocyte hemoglobin content of 20 to 27 pg would detect 12 of the 18 patients with iron deficiency (ferritin ≤12 ng/mL), for a sensitivity of 67%. A reticulocyte hemoglobin content <20 pg would detect 5 of the 18 patients with iron deficiency (ferritin ≤12 ng/mL), for a sensitivity of 28% . Specificity is the number of true negatives (healthy individuals correctly identified without the disease) divided by the total number of individuals without the disease (the sum of the true negatives plus false positives). A reticulocyte hemoglobin content of <20 pg does not identify any individuals without iron deficiency (0 false positives), so its specificity is 100%. A reticulocyte hemoglobin content of ≤27 pg identifies 4 false positives (ferritin >13 ng/mL) amongst a population of 40 who are healthy and do not have iron deficiency for a specificity of 36/(36+4) or 36/40 or 90%. By determining the appropriate reticulocyte hemoglobin content level, one can improve sensitivity to avoid missing patients who are truly affected. By determining specificity, one may avoid false-positive results and correctly identify patients who do not have the condition. Reticulocyte hemoglobin content ≤27 pg may be used to screen for iron deficiency and to determine response to iron replacement therapy. However, reticulocyte hemoglobin content is also below normal in individuals with thalassemia trait. Reticulocyte hemoglobin content may be helpful in detecting iron deficiency in children and adults with thalassemia trait by using lower reticulocyte hemoglobin content cutoff levels (< 20 pg) to improve sensitivity. PREP Pearls Sensitivity reflects the ability of a diagnostic tool to accurately detect affected patients, avoiding false negatives. Specificity reflects the ability of a diagnostic tool to only detect the affected patients, avoiding false positives.

A 17-year-old adolescent girl has a 2-month history of worsening abdominal pain, jaundice, and weight loss. She immigrated from Taiwan 3 months ago. She reports no smoking, alcohol use, intravenous drug use, or unprotected intercourse. She has jaundice, icteric sclerae, and a firm, palpable liver edge. Chest, abdominal, and pelvic imaging studies reveal a 10-cm intrahepatic mass. Her serum liver transaminase levels are mildly elevated, and her α-fetoprotein level is elevated. The results of hepatitis B surface antigen and hepatitis B e antigen tests are positive. Findings from a complete blood cell count, HIV test, prothrombin time, activated partial thromboplastin time, stool guaiac test, and hepatitis A, C, and D tests are normal. Of the following, which is MOST commonly associated with this patient's hepatic tumor? A.Gardner syndrome B.Li-Fraumeni syndrome C.low birthweight D.viral hepatitis

This patient most likely has hepatocellular carcinoma (HCC), which needs to be confirmed by a biopsy. Children with viral hepatitis infection have an increased risk of developing HCC. Li-Fraumeni syndrome and Gardner syndrome are autosomal dominant cancer predisposition syndromes associated with the development of hepatoblastoma. Although not fully understood, there is a strong association between extremely low birth weight (< 1,000 g) and the development of hepatoblastoma. Liver tumors comprise approximately 1% of childhood malignant tumors, and between 100 and 150 children in the United States are diagnosed each year with primary liver cancer. The most common malignant liver tumors are hepatoblastoma and HCC. Less common malignant primary liver tumors include embryonal sarcoma of the liver, rhabdomyosarcoma, angiosarcoma, and choriocarcinoma. Hepatoblastoma usually occurs in young children and represents two-thirds of all malignant liver tumors. The mean age at diagnosis of hepatoblastoma is 19 months, and only 5% of cases occur in children older than 4 years. Hepatocellular carcinoma occurs more commonly in adolescents and adults and is usually associated with underlying liver disease, cancer predisposition syndromes, and cirrhosis. Disorders that increase the risk of HCC are shown in the Table. Viral hepatitis infection increases the risk of HCC, usually in the setting of chronic infection. Chronic hepatitis B infection is characterized by the persistence of hepatitis B surface antigen for at least 6 months (with or without concurrent hepatitis B e antigen). The risk of chronic hepatitis B infection increases as the age at which initial infection occurs decreases. Greater than 90% of infants infected with hepatitis B will develop chronic infection. Hepatocellular carcinoma is more common than hepatoblastoma in Asia and Africa, a consequence of the increased prevalence of hepatitis B infection. There are studies evaluating antiviral agents which may decrease the risk of progression to malignancy. With the introduction of the hepatitis B vaccine in the 1980s, the incidence of HCC has dropped significantly worldwide. Li-Fraumeni syndrome (LFS) is caused by a germline mutation of the p53 tumor suppressor gene. Children with this syndrome are at risk for developing cancer early in life and may develop multiple cancers over a lifetime. A variant of LFS results from a mutation in another tumor suppressor gene, CHK2, which regulates the action of p53. The most common cancers that arise in LFS are sarcoma, brain tumors (choroid plexus carcinoma and glioblastoma), breast cancer, adrenocortical carcinoma, leukemia, and lymphoma. Other malignancies reported to be linked with LFS include melanoma and Wilms tumor, as well as gonadal germ cell, pancreatic, gastric, colorectal, and prostate cancers. Gardner syndrome is caused by mutation in the adenomatous polyposis coli gene, APC, and is a subtype of familial adenomatous polyposis. Children with Gardner syndrome have an increased risk of colon cancer, thyroid cancer, epidermoid cysts, osteomas of the skull, fibromas, and desmoid tumors. PREP Pearls Hepatocellular carcinoma occurs more commonly in adolescents and adults and is associated with underlying liver disease, cancer predisposition syndromes, and cirrhosis. Chronic hepatitis B infection increases the risk for developing hepatocellular carcinoma. The incidence of hepatocellular carcinoma in the United States has decreased following the introduction of the hepatitis B vaccine. ABP Content Specifications(s)/Content Area Know the association between hepatitis viruses and hepatocellular carcinoma Suggested Readings Kelly D, Sharif K, Brown RM, Morland B. Hepatocellular carcinoma in children. Clin Liver Dis. 2015;19(2):433-447. doi: http://dx.doi.org/10.1016/j.cld.2015.01.010 Meyers RL, Trobaugh-Lotrario AD, Malogolowkin MH, Katzenstein HM, López-Terrada DH, Finegold MJ. Liver tumors. In: Pizzo PA, Poplack DG, eds. Principles and Practice of Pediatric Oncology. 7th ed. Philadelphia, PA: Wolters Kluwer; 2016:725-751. Rapkin LB, Olson TA. Hepatic tumors. In: Lanzkowsky PL, Lipton JM, Fish JD, eds. Lanzkowsky's Manual of Pediatric Hematology and Oncology. 6th ed. London, UK: Elsevier; 2016:569-576. Wen WH, Chang MH, Hsu HY, Ni YH, Chen HL. The development of hepatocellular carcinoma among prospectively followed children with chronic hepatitis B virus infection. J Pediatr. 2004;144(3):397-399. doi: http://dx.doi.org/10.1016/j.jpeds.2003.11.022 Zhang XF, Liu XM, Wei T, et al. Clinical characteristics and outcome of hepatocellular carcinoma in children and adolescents. Pediatr Surg Int. 2013;29(8):763-770. doi: http://dx.doi.org/10.1007/s00383-013-3334-4

A 13-year-old adolescent boy is referred for evaluation of chronic, refractory cytopenias. He first presented at age 5 years with lymphadenopathy and splenomegaly and subsequently developed autoimmune thrombocytopenia and neutropenia at age 9 years. He has received intermittent treatment with corticosteroids, intravenous immunoglobulin, and rituximab with some improvement, but his cytopenias are now worsening. A lymph node biopsied at age 6 years showed paracortical expansion and follicular hyperplasia but no evidence of malignancy or infection. His lymphadenopathy has now nearly resolved. Of the following, the study MOST likely to yield a definitive diagnosis is A.antineutrophil antibodies B.B- and T-lymphocyte subsets C.bone marrow aspiration and biopsy D.direct antiglobulin test

This patient's clinical presentation of chronic nonmalignant lymphadenopathy that improves with age, splenomegaly, and multilineage autoimmune cytopenias suggests a diagnosis of autoimmune lymphoproliferative syndrome (ALPS), a rare disorder of lymphocyte homeostasis. In ALPS, lymphocyte apoptosis does not occur normally, allowing accumulation of lymphoid tissue and development of autoimmunity. The cell-surface receptor protein FAS (a member of the tumor necrosis factor receptor superfamily of proteins) plays a critical role in lymphocyte homeostasis and prevention of autoimmunity by mediating lymphocyte apoptosis. High levels of FAS are expressed on activated B and T lymphocytes and on other cells in the body. Binding with the FAS ligand results in intracellular changes that ultimately lead to formation of the "death-inducing signaling complex" that incites caspase activation culminating in the death of cells. In ALPS, the lymphocyte apoptosis does not occur normally and uncontrolled proliferation results, manifesting as chronic nonmalignant lymphadenopathy, hepatosplenomegaly, and recurring cytopenias. The initial presentation is often chronic lymphadenopathy or splenomegaly in an otherwise well-appearing child. However, patients may also present initially with signs and symptoms of hemolytic anemia, such as fatigue, pallor, and icterus, or cytopenias associated with bruising, mucocutaneous bleeding, and infections. Multilineage cytopenias occur due to both autoimmune complications and hypersplenism. Symptomatic cytopenias are usually most severe in early childhood and often improve during adolescence and young adulthood. On evaluation of these patients, the most common laboratory findings are cytopenias, with anemia being almost always present due to autoimmunity or hypersplenism. Multiple autoantibodies, such as antineutrophil antibodies, and a positive direct Coombs test or direct antiglobulin test are frequently present, but these are nonspecific and can be seen in other autoimmune conditions. Bone marrow evaluation may be helpful to rule out malignancy and other causes of cytopenias, but the laboratory picture usually suggests autoimmune destruction rather than decreased production of hematopoietic cells. An increase in a rare population of T-cell receptor (TCR) -αβ+ cells that lacks expression of CD4 and CD8, known as double-negative T cells (DNT cells), is a diagnostic hallmark of ALPS, although the role of these cells is not well understood. Because DNT cells are a common reactive feature in many disorders, a diagnosis of ALPS requires a minimum DNT population of 1.5% of total lymphocytes or 2.5% of T lymphocytes, in the setting of normal or elevated total lymphocyte counts. When DNT cells comprise greater than 3% of total lymphocytes or greater than 5% of T lymphocytes, it is highly suggestive of ALPS and is generally not seen in other disorders. Lymph node pathology in ALPS is characterized by paracortical expansion because of infiltration by polyclonal DNT cells, follicular hyperplasia, and polyclonal plasmacytosis. The diagnosis of ALPS is made from a set of diagnostic criteria developed by consensus at the First International ALPS Workshop in 2009. The 2 required criteria are: Chronic lymphadenopathy and/or splenomegaly Elevated circulating TCR-αβ+ DNT cells Additional criteria consist of: Primary criteria: Abnormal lymphocyte apoptosis assay (only offered in few specialized centers) Pathogenic mutations in genes of the FAS pathway (FAS, FASLG, or CASP10) Secondary criteria: Elevated biomarker levels (soluble FAS ligand, plasma IL-10 or IL-18, or vitamin B12) Typical histopathology Family history of nonmalignant lymphoproliferation Autoimmune cytopenias with elevated IgG levels For a definitive diagnosis of ALPS, both of the required criteria and one of the primary criteria must be present. For a probable diagnosis of ALPS, the required criteria and any one of the secondary criteria must be present. The majority of ALPS cases are associated with autosomal dominant inheritance of heterozygous germline FAS mutations. Variable penetrance and expressivity are seen in both affected families and animal models. The risk of Hodgkin lymphoma is increased fiftyfold and the risk of non-Hodgkin lymphoma is increased fourteenfold in patients with ALPS, as compared to the general population. Approximately 90% of ALPS patients with lymphoma have germline heterozygous mutations of FAS affecting the intracellular portion of the protein. These patients are generally responsive to conventional lymphoma therapy. Treatment for ALPS includes management of cytopenias, surveillance for lymphoma, and genetic counseling. The management of autoimmune cytopenias in ALPS mirrors that seen in other autoimmune cytopenias, including corticosteroids, intravenous immunoglobulin, intermittent granulocyte colony stimulating factor, rituximab, mycophenolate, and sirolimus. The risks and benefits of ongoing immunosuppression must be weighed carefully in these patients. Corticosteroids and most immunosuppressants are not effective in reducing lymphadenopathy or splenomegaly. Sirolimus can reduce adenopathy and spleen size, but the primary indication for therapy is to provide relief from refractory cytopenias related to hypersplenism. Many patients with ALPS undergo splenectomy to manage their chronic cytopenias, often prior to the diagnosis of ALPS. However, these patients have a high frequency of recurrent cytopenias and sepsis after splenectomy. They may be more susceptible to sepsis due to decreased circulating memory B lymphocytes, and the current recommendation is to avoid splenectomy when possible. The overall prognosis of ALPS is good, as the chronic cytopenias generally improve as patients age. There is a 5% mortality, usually related to postsplenectomy sepsis or malignancy. Hematopoietic stem cell transplantation does not have a role in this disease for most patients. However, patients need lifelong monitoring, particularly those patients with the genetic mutation conferring increased lymphoma risk. PREP Pearls Autoimmune lymphoproliferative syndrome is a disorder of lymphocyte homeostasis in which lymphocyte apoptosis does not occur normally, allowing accumulation of lymphoid tissue and development of autoimmunity. Autoimmune lymphoproliferative syndrome presents with chronic lymphadenopathy, splenomegaly, and multilineage cytopenias. The hallmark of autoimmune lymphoproliferative syndrome is an increase in T-cell receptor (TCR)-αβ+ cells that lack expression of CD4 and CD8, known as double-negative T (DNT) cells.

A 4-year-old girl with relapsed stage IV Wilms tumor is scheduled to receive a second chemotherapy cycle of ifosfamide, carboplatin, and etoposide. Of the following, the BEST antiemetic medication to prevent acute antineoplastic-induced nausea and vomiting in this patient is A.aprepitant B.lorazepam C.palonosetron D.promethazine

f the antiemetic medications listed, the 5-HT3 (serotonin) receptor antagonist palonosetron is the best choice. Palonosetron has not been associated with significant QTc interval prolongation compared to other 5-HT3 receptor antagonists (ondansetron, granisetron). Aprepitant, a neurokinin-1 receptor antagonist, is a moderate inhibitor of the cytochrome P450 3A4 enzyme. Since ifosfamide and etoposide are CYP3A4 substrates whose dose intensity may be altered when given with aprepitant, aprepitant is not an appropriate choice for the chemotherapy regimen in this vignette. Lorazepam and promethazine are typically not used to prevent acute antineoplastic-induced nausea and vomiting, but both can be used for breakthrough nausea and vomiting. Lorazepam is also indicated for anticipatory nausea, while promethazine has been used to prevent motion sickness and nausea after surgery. The best treatment of acute antineoplastic-induced nausea and vomiting is prevention. Optimal control is defined as: no vomiting, no retching, no nausea, no use of antiemetics other than preventative antiemetics, and no nausea-related change in the child's appetite and diet on each day of antineoplastic medication administration and 24 hours after last dose of antineoplastic medication for that therapy block. To choose appropriate preventative antiemetic therapy, the risk of emesis must be determined. Emetogenicity of antineoplastic therapy is determined by the frequency of emesis in the absence of prophylaxis. Emetogenicity is categorized as high (> 90%), moderate (30%-90%), low (10% to < 30%), or minimal (< 10%). Examples of chemotherapeutic agents associated with a given emetogenicity risk category are shown: High: carboplatin, cyclophosphamide ≥ 1 g/m2, methotrexate ≥ 12 g/m2 Moderate: doxorubicin, ifosfamide, intrathecal (methotrexate, hydrocortisone, cytarabine) Low: etoposide, 5-fluorouracil, topotecan Minimal: asparaginase, bevacizumab, vincristine A complete list of the acute emetogenic potential of antineoplastic agents used in pediatric cancer patients is found in Suggested Reading 2. Rarely is one antineoplastic medication given in isolation. If more than one chemotherapy agent is given in one day, the highest emetogenicity risk category is used to determine prophylaxis. If antineoplastic therapy is given over subsequent days, the highest emetogenicity risk category of all medications in that chemotherapy block is used. Some combinations of antineoplastic medications will increase the emetogenicity potential from a lower to a higher risk category. For example, although cytarabine has moderate emetogenicity alone and etoposide has low emetogenicity alone, the coadministration of these agents results in high risk of emetogenicity. Recommendations for antiemetic prophylaxis are based on emetogenicity risk, patient age, and whether the antineoplastic medication is known or suspected to interact with aprepitant metabolism. Children 6 months of age and older who receive high emetogenicity therapy that is not suspected to interact with aprepitant should receive prophylaxis with 5-HT3 receptor antagonist (ondansetron, granisetron, or palonosetron) plus dexamethasone plus aprepitant. If dexamethasone cannot be given, aprepitant plus palonosetron is preferred. Aprepitant is not recommended for children younger than 6 months. Palonosetron can be used for antiemetic prophylaxis in a child younger than 6 months who receives high emetogenicity antineoplastic therapy and who cannot receive dexamethasone. Similarly, for children 6 months of age and older who cannot receive aprepitant (because of interaction with chemotherapy) or dexamethasone, palonosetron is recommended. In general, for children 6 months of age and older who receive moderate emetogenicity therapy, prophylaxis with 5-HT3 receptor antagonist (ondansetron, granisetron, or palonosetron) plus dexamethasone is recommended. If dexamethasone cannot be given, aprepitant can be added to the regimen. Palonosetron can be used for antiemetic prophylaxis in a child younger than 6 months who receives a moderately emetogenic antineoplastic and who cannot receive dexamethasone. Low emetogenic risk therapy can be prophylaxed with 5-HT3 receptor antagonist alone. Minimal emetogenic risk therapy requires no routine prophylaxis. In addition to acute antineoplastic-induced nausea and vomiting, high dose cisplatin (> 70 mg/m2) or coadministration of anthracycline and cyclophosphamide results in delayed emesis. Delayed emesis is defined as occurring more than 24 hours after chemotherapy completion. Prophylaxis for delayed emesis includes aprepitant and corticosteroids. In adults, olanzapine, a second-generation antipsychotic that blocks serotonin 5-hydroxytryptamine (5-HT2) receptors and dopamine D2 receptors, is recommended, but currently there is a paucity of data for its use in children. Additional medications to treat delayed emesis include metoclopramide and 5-HT3 receptor antagonists. If the patient experiences nausea and/or vomiting despite a prophylactic regimen, diphenhydramine and promethazine (not recommended in children younger than 2 years) are commonly used as rescue medications. If this occurs with low or moderate emetogenic risk therapy, prophylaxis with the next cycle of therapy should be escalated to the next higher risk category. Although not listed in recommended prophylaxis, a scopolamine patch may also be used in pediatric patients. Lorazepam is recommended to reduce the occurrence of anticipatory nausea, a learned response to chemotherapy. Anticipatory nausea is more likely to occur if there is a history of motion sickness, anxiety, or poor control of antineoplastic-induced nausea and vomiting. Nonpharmacologic interventions for nausea and emesis (eg, guided imagery, music therapy, and acupuncture) may also be helpful, although there is little evidence to support these interventions. PREP Pearls Aprepitant, a moderate inhibitor of the cytochrome P450 3A4 enzyme, is known or suspected to interact with some antineoplastic medications that are CYP3A4 substrates and should be avoided in patients who receive these chemotherapy agents. When receiving multiple antineoplastic medications in one day or on multiple days, the chemotherapy with the highest emetogenicity potential should be used to to determine emetogenic potential of the therapy block and the appropriate antiemetic regimen. Children younger than 6 months should not receive aprepitant.


Ensembles d'études connexes

Example and Application Practice - Midterm 1 PSYC 309A

View Set

Chapter 38: Caring for Clients with Cerebrovascular Disorders

View Set

FRL 301 - CHAPTER 10 - basic knowledge terms

View Set

EMT Chapter 37: Transport Operations

View Set